You are on page 1of 729

Join us on Telegram :

Click here : @eduwaves360

Unlocked the Medical premiums

Click here : www.eduwaves360.com

Medical Courses : https://t.me/usmle_study_materials_2

Discussion Group : @usmle_discussion_group

www.eduwaves360.com | Telegram : @eduwaves360


Question # 1

A 62-year-old man comes to the physician because of a swollen and painful right knee for the last
3 days. He has no history of joint disease. His vital signs are within normal limits. Examination shows
erythema and swelling of the right knee, with limited range of motion due to pain. Arthrocentesis of the
right knee joint yields 7 mL of cloudy fluid with a leukocyte count of 29,000/mm3 (97% segmented
neutrophils). Compensated polarized light microscopy of the aspirate is shown. Which of the following
is the most likely underlying mechanism of this patient's knee pain?

Answer Image

A Bacterial infection of the joint

B Calcium pyrophosphate deposition

C Mechanical stress and trauma

www.eduwaves360.com | Telegram : @eduwaves360


Answer Image

Immune complex-mediated cartilage


D
destruction

E Monosodium urate deposition

www.eduwaves360.com | Telegram : @eduwaves360


Hint

The arthrocentesis aspirate shows rhomboid-shaped crystals that are positively birefringent.

www.eduwaves360.com | Telegram : @eduwaves360


Correct Answer

A - Bacterial infection of the joint

Explanation Why

The presence of bacteria in synovial fluid would be consistent with septic arthritis, of which the most
common causative organisms include S. aureus, Streptococcus, and Neisseria gonorrhoeae. Septic
arthritis is characterized by inflammation both locally and systemically, and classically results in
purulent synovial fluid (> 50,000/mm3 WBC). This patient's lack of fever and the presence of
crystals in the synovial fluid analysis suggest another diagnosis.

B - Calcium pyrophosphate deposition

Explanation But

Initial treatment is similar as for gout; NSAIDs and colchicine are used in acute inflammation. If
only 1–2 joints are affected, an intraarticular glucocorticoid injection can be considered.

Explanation Why

The presence of positively birefringent (blue under parallel light) rods and rhomboid crystals in
synovial fluid supports a diagnosis of calcium pyrophosphate crystal deposition disease (CPPD). The
deposition of calcium pyrophosphate crystals causes synovial inflammation, which manifests with
joint pain and swelling. CPPD most commonly affects the knee joint and is usually idiopathic, but
can also be caused by trauma, hyperparathyroidism, and hemochromatosis.

C - Mechanical stress and trauma

Explanation Why

Repetitive mechanical wear and tear or trauma is the underlying mechanism of osteoarthritis, a
chronic and degenerative form of arthritis. This patient's acute knee pain is not consistent with

www.eduwaves360.com | Telegram : @eduwaves360


osteoarthritis and crystals would not be expected on synovial fluid analysis of an osteoarthritic joint.

D - Immune complex-mediated cartilage destruction

Explanation Why

Immune complex formation and deposition in the synovium is the underlying mechanism of
autoimmune arthritis (e.g., as seen in SLE, RA). The deposition of immune complex in joint spaces
leads to the recruitment of leukocytes and activation of complement, which in turn leads to both local
and systemic inflammation. Aspiration of these joints would reveal sterile fluid with elevated
leukocytes (5,000–50,0000 mm3) but no crystals.

E - Monosodium urate deposition

Image

www.eduwaves360.com | Telegram : @eduwaves360


Explanation Why

Articular deposition of monosodium urate crystals occurs in patients with gout, which can also
present with articular pain and swelling. Gout is due to overproduction or under excretion of uric
acid, and most commonly affects the smaller lower extremity joints, such as the toes and ankles.
Microscopically, monosodium urate crystals appear needle-shaped and negatively birefringent
(yellow under parallel light), not rod or rhomboid-like as seen in this patient's synovial fluid.

www.eduwaves360.com | Telegram : @eduwaves360


Question # 2

A 36-year-old woman is brought to the emergency department after a high-speed motor vehicle
collision. Her temperature is 36.5°C (97.7°F), pulse is 120/min, respirations are 24/min, and blood
pressure is 100/65 mm Hg. Examination shows second and third-degree burns covering 30% of the
surface area of her body. Intravenous fluids are administered. 30 minutes later, the patient develops
respiratory distress and requires intubation. Administration of succinylcholine during the procedure is
most likely to increase this patient's risk of developing which of the following laboratory abnormalities?

Answer Image

A Hyperphosphatemia

B Hypermagnesemia

C Hyperkalemia

D Hypernatremia

E Hyperglycemia

www.eduwaves360.com | Telegram : @eduwaves360


Hint

Succinylcholine is a depolarizing paralytic agent that can worsen metabolic imbalances associated with
cellular destruction (e.g., large second- and third-degree burns, crush injuries, etc.).

www.eduwaves360.com | Telegram : @eduwaves360


Correct Answer

A - Hyperphosphatemia

Explanation Why

Succinylcholine has no significant effect on phosphate metabolism. Furthermore, hyperphosphatemia


is not a complication of burn injuries, unlike hypophosphatemia, which may occur as a late
complication (> 48 hours after injury).

B - Hypermagnesemia

Explanation Why

Succinylcholine has no effect on magnesium metabolism. Moreover, hypermagnesemia is not a


complication of burn injuries, unlike hypomagnesemia, which may occur as a late complication (> 48
hours after injury).

www.eduwaves360.com | Telegram : @eduwaves360


C - Hyperkalemia

Image

Explanation But

Malignant hyperthermia is a rare complication of succinylcholine administration that occurs in


patients with a predisposing genetic mutation.

Explanation Why

Acute hyperkalemia is a dangerous complication found in patients with extensive second- and third-
degree burns. Potassium is stored primarily in the intracellular space. Therefore, any cause of
significant tissue breakdown (e.g., significant burn or crush injuries), releases excess potassium into
the extracellular space. Succinylcholine causes persistent depolarization of the motor end plate,
resulting in further increased extracellular shift of potassium.

www.eduwaves360.com | Telegram : @eduwaves360


D - Hypernatremia

Explanation Why

Succinylcholine has no effect on sodium metabolism. However, hypernatremia may occur as a late
complication of second- and third-degree burns after the initial resuscitation period (> 36 hours) as a
result of increased insensible, hypotonic fluid loss despite large-volume fluid resuscitation.

E - Hyperglycemia

Explanation Why

Succinylcholine has no significant effect on glucose metabolism. However, hyperglycemia can


present as a complication of moderate-to-severe burns due to increased insulin resistance,
gluconeogenesis, and glycogenolysis.

www.eduwaves360.com | Telegram : @eduwaves360


Question # 3

A 76-year-old woman is brought to the physician because of lesions on her left arm. She first noticed
them 3 months ago and they have grown larger since that time. She has not had any pain or pruritus in
the area. She has a history of invasive ductal carcinoma of the left breast, which was treated with
mastectomy and radiation therapy 27 years ago. Since that time, she has had lymphedema of the left
arm. Physical examination shows extensive edema of the left arm. There are four coalescing, firm,
purple-blue nodules on the left lateral axillary region and swelling of the surrounding skin. Which of the
following is the most likely diagnosis?

Answer Image

A Cellulitis

B Thrombophlebitis

C Lichen planus

D Angiosarcoma

www.eduwaves360.com | Telegram : @eduwaves360


Answer Image

E Melanoma

F Kaposi sarcoma

www.eduwaves360.com | Telegram : @eduwaves360


Hint

A biopsy would likely show collections of slit-like anastomosing vascular channels that are lined by
endothelial cells.

www.eduwaves360.com | Telegram : @eduwaves360


Correct Answer

A - Cellulitis

Image

Explanation Why

Chronic lymphedema is an important risk factor for development of cellulitis and other soft tissue
infections, and this patient is certainly at ongoing risk of infection because of her long-standing
edema. However, she lacks typical features of cellulitis (e.g., fever, pain, erythema, warmth, or
tenderness), which makes an infection very unlikely. Additionally, untreated cellulitis would have
progressed more quickly and the timeline of her symptoms is not consistent with this diagnosis.

www.eduwaves360.com | Telegram : @eduwaves360


B - Thrombophlebitis

Explanation Why

Patients with thrombophlebitis of a superficial vein typically present with erythema and tenderness
along the vein, and the condition can cause congestion of venous vasculature distal to the thrombus.
In the upper extremities, it is usually associated with intravenous catheter use. While the nodular
appearance and the distribution of this patient's lesions is potentially consistent with a
thrombophlebitis, pain and erythema would also be expected. Her history of chronic lymphedema
suggests a different diagnosis.

C - Lichen planus

Image

Explanation Why

Cutaneous lichen planus typically manifests as purple, polygonal, pruritic papules or plaques on the

www.eduwaves360.com | Telegram : @eduwaves360


lower back, wrist, and/or ankles. Patients also present with a lattice-like network of white stripes
called Wickham striae. This patient is missing common features of lichen planus, e.g., pruritus and
polygonal shape. Moreover, the distribution of her lesions and the fact that they appear in
conjunction with chronic lymphedema suggest a different diagnosis.

D - Angiosarcoma

Explanation But

Due to their locally aggressive nature and tendency to metastasize early, angiosarcomas of the breast
have a poor prognosis.

Explanation Why

Angiosarcoma of the breast is a rare malignancy that can occur secondary to chronic lymphedema. It
classically appears in women who have undergone radiation therapy with subsequent lymph node
sclerosis and/or mastectomy with extensive axillary lymphadenectomy, in which case it is also
referred to as Stewart-Treves syndrome. Angiosarcomas typically manifest as multiple violaceous
macules, papules, and/or subcutaneous nodules that develop in the area of the breast, chest wall, or
upper extremity.

www.eduwaves360.com | Telegram : @eduwaves360


E - Melanoma

Image

Explanation Why

Melanoma should always be considered in the differential for a growing, asymmetric,


hyperpigmented skin lesion. The amelanotic subtype can present as pink or even skin-colored
nodules. While this patient is at increased risk (age, previous skin cancer) for development of
melanoma, the presence of chronic lymphedema suggests a different diagnosis.

www.eduwaves360.com | Telegram : @eduwaves360


F - Kaposi sarcoma

Image

Explanation Why

Human herpesvirus 8 can cause Kaposi sarcoma, which manifests as purple-colored or brownish skin
plaques of the extremities that can progress to deeper, more nodular, and, occasionally, ulcerative,
lesions. However, the lesions occur in patients who are significantly immunocompromised (e.g.,
AIDS, transplant recipients). The absence of severe immunocompromise in this woman makes this
diagnosis highly unlikely.

www.eduwaves360.com | Telegram : @eduwaves360


Question # 4

A 69-year-old man comes to the physician because of a 2-month history of severe right hip pain. The
pain is worse at night. He has chronic headaches and back pain for which he takes vitamin D, calcium
supplements, and ibuprofen. Examination shows hip tenderness and mild sensorineural hearing loss.
X-ray of the hip shows a radiolucent lesion in the ilium with a moth-eaten appearance, wide transition
zone, and an aggressive periosteal reaction. Wide excision of the lesion is performed. A
photomicrograph of a section of the lesion is shown. Which of the following is the most likely
diagnosis?

Answer Image

A Multiple myeloma

www.eduwaves360.com | Telegram : @eduwaves360


Answer Image

B Chondrosarcoma

C Osteosarcoma

D Ewing sarcoma

E Giant cell tumor

www.eduwaves360.com | Telegram : @eduwaves360


Hint

This patient likely has Paget disease of the bone (PDB), which, when symptomatic, manifests with
nonspecific features such as bone pain. The sensorineural hearing loss is likely due to bony compression
of the vestibulocochlear nerve. Chronically increased bone turnover in patients with long-standing PDB
increases the risk of a certain bone malignancy.

www.eduwaves360.com | Telegram : @eduwaves360


Correct Answer

A - Multiple myeloma

Image

Explanation Why

Multiple myeloma commonly affects the axial skeleton (including the pelvis), has a peak incidence at
50–70 years, and would cause radiolucent lesions on x-ray. This patient's history of chronic back
pain, chronic headaches (skull involvement), and hearing loss (SNHL) are also consistent with the
diagnosis of multiple myeloma. However, multiple myeloma would appear as well-defined (narrow
transition zone) lesions on x-ray with a punched-out appearance and minimal periosteal reaction.
Furthermore, biopsy of a multiple myeloma lesion would show numerous plasma cells, which are not
seen here.

www.eduwaves360.com | Telegram : @eduwaves360


B - Chondrosarcoma

Image

Explanation Why

Chondrosarcoma is associated with PDB, often affects the pelvis, can appear as an ill-defined (wide
transition zone), radiolucent lesion with a moth-eaten appearance and aggressive periosteal reaction,
and show eosinophilic cartilaginous matrix on biopsy, as seen here. However, the biopsy of a
chondrosarcoma would also show nests of small, round, anaplastic cartilage cells with a
pathognomonic chicken wire mesh appearance, and an x-ray would typically reveal ring-and-arc or
popcorn-like calcification within the tumor. These features are not seen in this patient.

C - Osteosarcoma

Explanation But

Secondary osteosarcomas, which develop in diseased bone, typically manifest in the elderly, as seen

www.eduwaves360.com | Telegram : @eduwaves360


here. In contrast, primary osteosarcomas, which develop in previously normal bone, have a peak
incidence at 10–20 years of age, with metaphyses of long bones (typically the distal femur, proximal
tibia) more commonly affected than flat bones (e.g., the ilium).

Explanation Why

Osteosarcoma is the most common bone malignancy associated with PDB. Osteosarcomas have
immature osteoid, which appears as eosinophilic, glassy material around pleomorphic tumor cells.
Destruction of normal bone results in an osteolytic (radiolucent) lesion with an ill-defined border
(wide transition zone) and a moth-eaten appearance. Aggressive periosteal reactions (e.g., sunburst
appearance, Codman triangle) indicate lifting of the periosteum following rapid tumor growth. Other
risk factors for developing osteosarcomas are bone infarcts, radiation, retinoblastoma gene
mutations, and Li-Fraumeni syndrome.

D - Ewing sarcoma

Image

Explanation Why

Ewing sarcoma can occur in flat bones such as the ilium and appear as an ill-defined (wide transition
zone), radiolucent lesion with a moth-eaten appearance and an aggressive periosteal reaction (e.g., a

www.eduwaves360.com | Telegram : @eduwaves360


lamellated “onion-skin” reaction). However, a biopsy would have shown nests of anaplastic small
blue cells (primitive neuroectodermal cells) arranged around capillaries (pseudorosettes). Moreover,
Ewing sarcoma is not a complication of PDB, which this patient has.

E - Giant cell tumor

Image

Explanation Why

A giant cell tumor of the bone (osteoclastoma), which is a rare complication of PDB, can present as a
radiolucent lesion and show multinucleated cells on biopsy, as seen here. However, the
multinucleated cells would be larger (giant cells) and contain many more nuclei (typically > 50), and
other pathognomic features such as a highly vascular stroma as well as sheets of mononuclear cells
would be present. Moreover, osteoclastomas typically affect the epiphyseal regions of long bones,
rather than flat bones like the ilium, and an x-ray would show a well-defined (narrow transition zone)
lesion with a non-aggressive periosteal reaction and a soap-bubble appearance.

www.eduwaves360.com | Telegram : @eduwaves360


Question # 5

A 22-year-old woman is brought to the emergency department after being struck by a car while crossing
the street. She has major depressive disorder with psychosis. Current medications include sertraline and
haloperidol. Vital signs are within normal limits. X-ray of the lower extremity shows a mid-shaft femur
fracture. The patient is taken to the operating room for surgical repair of the fracture. As the surgeon
begins the internal fixation, the patient shows muscle rigidity and profuse diaphoresis. Her temperature
is 39°C (102.2°F), pulse is 130/min, respirations are 24/min, and blood pressure is 146/70 mm Hg. The
pupils are equal and reactive to light. The end tidal CO2 is 85 mm Hg. Which of the following is the
most appropriate treatment for this patient's condition?

Answer Image

Dantrolene
A
therapy

Fat
B
embolectomy

Cyproheptadine
C
therapy

Bromocriptine
D
therapy

Propranolol
E
therapy

www.eduwaves360.com | Telegram : @eduwaves360


Hint

After receiving volatile anesthetics and/or depolarizing neuromuscular blocking drugs (e.g.,
succinylcholine) as part of general anesthesia, patients with mutations affecting the ryanodine receptor
(RYR) channels can develop malignant hyperthermia.

www.eduwaves360.com | Telegram : @eduwaves360


Correct Answer

A - Dantrolene therapy

Image

Explanation Why

The features of malignant hyperthermia (e.g., fever, muscle rigidity, tachycardia, tachypnea,
increased CO2 production) are caused by severe, generalized muscle contraction due to increased
Ca2+ release from the sarcoplasmic reticulum via the RYR channels. Dantrolene, an RYR antagonist,
would prevent this Ca2+ release.

www.eduwaves360.com | Telegram : @eduwaves360


B - Fat embolectomy

Explanation Why

A femoral fracture is a risk factor for pulmonary fat embolism. In an anesthetized patient, pulmonary
embolism may manifest with tachycardia. However, the end-tidal CO2 would be decreased because
of impaired pulmonary perfusion and muscle rigidity would not be expected.

C - Cyproheptadine therapy

Explanation Why

Cyproheptadine is the antidote of choice to treat severe serotonin syndrome. Serotonin syndrome
may be a complication of therapy with antidepressants, such as sertraline. Although serotonin
syndrome may also present with hyperthermia, diaphoresis, and muscle hypertonia, it usually occurs
after a recent increase in the dose or frequency of antidepressants, which has not been mentioned
here. The absence of other typical features of serotonin syndrome, such as clonus and mydriasis, and
the marked increase in end-tidal CO2 suggest a different diagnosis.

D - Bromocriptine therapy

Explanation Why

Bromocriptine are used to treat neuroleptic malignant syndrome (NMS), not malignant hyperthermia.
This patient is taking haloperidol, which may predispose her to develop NMS. While patients with
NMS also present with fever, hypertension, tachycardia, and muscle rigidity, the onset of NMS is not
hyperacute, as seen here, but evolves gradually over 1–3 days. Furthermore, it usually occurs within
2 weeks of initiating antipsychotic therapy or increasing dosage.

www.eduwaves360.com | Telegram : @eduwaves360


E - Propranolol therapy

Explanation Why

Propranolol is the treatment of choice for a thyrotoxic crisis. In patients with untreated
hyperthyroidism, acute stress (e.g., surgery) may precipitate a thyrotoxic crisis. In an anesthetized
patient, thyrotoxic crisis presents with fever, diaphoresis, tachycardia, and/or hypertension. Muscle
rigidity, however, is not a typical feature of a thyrotoxic crisis. Additionally, there were no
preoperative clinical features to suggest hyperthyroidism in this patient.

www.eduwaves360.com | Telegram : @eduwaves360


Question # 6

A 67-year-old man presents to his physician's office for evaluation of a 9-month history of pain in the
midline of his lower back. Treatment with over-the-counter analgesics has not been effective. Physical
examination shows point tenderness over the lumbar vertebrae. X-ray of the lumbosacral spine shows
coarse trabecular bone with cortical thickening of the L3 vertebra consistent with a “picture-frame
appearance.” The L3 vertebra appears slightly larger than L4 on lateral view. Further evaluation is most
likely to show which of the following sets of laboratory findings?

Alkaline phosphatase Parathyroid hormone


Calcium Phosphate
(ALP) (PTH)

A Increased decreased increased increased

B Normal normal normal normal

C Normal normal increased normal

D Decreased decreased increased increased

E Decreased increased increased increased

F Increased increased normal decreased

Answer Image

A A

B B

www.eduwaves360.com | Telegram : @eduwaves360


Answer Image

C C

D D

E E

F F

www.eduwaves360.com | Telegram : @eduwaves360


Hint

This patient's x-ray shows coarse trabecular bone and cortical thickening in a “picture-frame”
appearance, which is diagnostic of Paget disease of bone (PDB).

www.eduwaves360.com | Telegram : @eduwaves360


Correct Answer

A-A

Explanation Why

Elevated PTH, calcium, and ALP, along with decreased phosphate is consistent with primary
hyperparathyroidism. PTH increases serum calcium by stimulating bone turnover and renal excretion
of phosphate. X-ray findings in primary hyperparathyroidism include rugger-jersey spine and the
tumor-like appearance of osteitis fibrosa cystica. This patient's x-ray findings are inconsistent with
primary hyperparathyroidism.

B-B

Image

www.eduwaves360.com | Telegram : @eduwaves360


Explanation Why

Normal calcium, phosphate, ALP, and PTH is most consistent with osteoporosis and/or
osteopetrosis. In osteoporosis, there is a decrease in both osteoblastic and osteoclastic activity;
however, osteoclastic activity predominates, which causes the bone to demineralize. This patient's x-
ray findings are inconsistent with both osteoporosis and osteopetrosis.

C-C

Image

Explanation Why

PDB is characterized by an accelerated rate of bone remodeling, which can cause bone pain and
pathologic fractures. Increased bone remodeling leads to coarse trabecular bone and sclerosis of the
vertebral margins (i.e., “picture-frame” appearance) and can be seen on x-ray, as in this patient.
Laboratory findings in PDB show normal calcium, phosphate, and PTH, along with elevated ALP
levels. Calcium and phosphate are not affected because of the balance of increased osteoblastic and
osteoclastic activity.

www.eduwaves360.com | Telegram : @eduwaves360


D-D

Explanation Why

Decreased calcium and phosphate, along with increased ALP and PTH is consistent with secondary
hyperparathyroidism due to vitamin D deficiency. Phosphate is decreased due to deficient intestinal
absorption from the vitamin D deficiency and increased renal excretion from high PTH levels. X-ray
findings in secondary hyperparathyroidism include rugger-jersey spine and the tumor-like
appearance of osteitis fibrosa cystica. This patient's x-ray findings are inconsistent with secondary
hyperparathyroidism.

E-E

Explanation Why

Decreased calcium, along with increased phosphate, ALP, and PTH is consistent with secondary
hyperparathyroidism due to chronic kidney disease. X-ray findings in secondary
hyperparathyroidism include rugger-jersey spine and the tumor-like appearance of osteitis fibrosa
cystica. This patient's x-ray findings are inconsistent with secondary hyperparathyroidism.

F-F

Explanation Why

Elevated calcium and phosphate, normal ALP, and decreased PTH occurs with hypervitaminosis D
or granulomatous diseases, such as sarcoidosis. Clinical features of hypervitaminosis D are mainly
caused by hypercalcemia and include nausea, vomiting, diarrhea, constipation, polyuria, and
polydipsia. In granulomatous diseases, calcium is elevated due to increased expression of 1α-
hydroxylase in macrophages, which activates vitamin D. This patient lacks evidence of systemic
disease.

www.eduwaves360.com | Telegram : @eduwaves360


Question # 7

A 32-year-old woman comes to the physician because of pain and stiffness in both of her hands for the
past 3 weeks. The pain is most severe early in the day and does not respond to ibuprofen. She has no
history of serious illness and takes no medications. Vital signs are within normal limits. Examination
shows swelling and tenderness of the wrists and metacarpophalangeal joints bilaterally. Range of motion
is decreased due to pain. There are subcutaneous, nontender, firm, mobile nodules on the extensor
surface of the forearm. Which of the following is the most appropriate pharmacotherapy for this patient's
current symptoms?

Answer Image

A Methotrexate

B Adalimumab

C Colchicine

D Sulfasalazine

E Prednisone

www.eduwaves360.com | Telegram : @eduwaves360


Hint

This patient has classic signs of rheumatoid arthritis (RA), including morning stiffness, subcutaneous
rheumatoid nodules, and swelling of the wrist and metacarpophalangeal joints. Her current symptoms
are likely due to an acute attack of the disease.

www.eduwaves360.com | Telegram : @eduwaves360


Correct Answer

A - Methotrexate

Explanation Why

Methotrexate is the first-line disease-modifying antirheumatic drug (DMARD) for patients with
rheumatoid arthritis (RA). It slows progression of the disease, preserves joint function, and limits
complications, and therefore, should be initiated as soon as a diagnosis of RA is made. However, it
takes ≥ 6 weeks to take effect. This patient requires fast-acting symptomatic relief for her acute flare.

B - Adalimumab

Explanation Why

The biologic adalimumab is used in patients with rheumatoid arthritis who still have moderate or
severe disease activity after 3 months of DMARD therapy. It would not be used as the initial
management of an acute flare.

C - Colchicine

Explanation Why

Colchicine is used to treat gout and pseudogout, both of which can present with acute flares of joint
pain. However, these conditions generally present with monoarthritis and/or asymmetrical joint
involvement. Gout, which is often triggered by a large, purine-rich meal, or alcohol consumption,
typically affects the metatarsophalangeal joint of the great toe, the knee, and the thumb. Pseudogout
typically affects the knee joint in elderly patients, and is associated with conditions such as
hemochromatosis and hyperparathyroidism.

www.eduwaves360.com | Telegram : @eduwaves360


D - Sulfasalazine

Explanation Why

Sulfasalazine is a disease-modifying antirheumatic drug (DMARD) that is typically used in pregnant


patients with rheumatoid arthritis (RA). However, it takes ≥ 6 weeks to take effect. This patient
requires fast-acting symptomatic relief for her acute flare.

E - Prednisone

Image

Explanation Why

Glucocorticoids like prednisone act rapidly to reduce symptoms of severe acute flares of rheumatoid
arthritis. A short course of glucocorticoids has been shown to be more effective than NSAID therapy
in reducing joint tenderness and pain and can be used to bridge the period until DMARDs, which are
prescribed on a long-term basis to control disease activity, take effect.

www.eduwaves360.com | Telegram : @eduwaves360


www.eduwaves360.com | Telegram : @eduwaves360
Question # 8

Seven hours after undergoing left hip arthroplasty for chronic hip pain, a 67-year-old woman reports a
prickling sensation in her left anteromedial thigh and lower leg. Neurologic examination shows left leg
strength 3/5 on hip flexion and 2/5 on knee extension. Patellar reflex is decreased on the left. Sensation
to pinprick and light touch are decreased on the anteromedial left thigh as well as medial lower leg.
Which of the following is the most likely underlying cause of this patient's symptoms?

Answer Image

A Sural nerve injury

B Femoral nerve injury

C Fibular nerve injury

D Obturator nerve injury

E S1 radiculopathy

F L5 radiculopathy

www.eduwaves360.com | Telegram : @eduwaves360


Hint

Postoperative anteromedial thigh and lower leg paresthesias with concurrent decreased patellar reflex,
hip flexion, and knee extension are consistent with damage to a peripheral nerve of the anterior lower
extremity.

www.eduwaves360.com | Telegram : @eduwaves360


Correct Answer

A - Sural nerve injury

Explanation Why

Sural nerve injury is usually caused by Achilles tendon rupture, entrapment, or ganglion lipomas.
The sural nerve is a purely sensory nerve, and injury to it typically manifests with decreased
sensation on the posterolateral lower leg and the lateral border of the foot. It would not explain the
loss of sensation on the anteromedial thigh and decreased strength in knee extension seen in this
patient.

B - Femoral nerve injury

Explanation Why

This patient has evidence of femoral nerve injury, likely from prolonged compression during surgery.
Injury of the femoral nerve typically results in loss of sensation in the anteromedial thigh and medial
lower leg (anterior cutaneous branches), as well as decreased strength in hip flexion (iliopsoas
muscle) and knee extension (quadriceps femoris muscle).

C - Fibular nerve injury

Explanation Why

Fibular nerve injury is usually caused by injuries in the lower leg (e.g., fracture of the fibular head or
tight-fitting casts of the lower leg). It typically presents with foot drop (weakness of foot
dorsiflexors) and a loss of sensation between the first and second toe (deep peroneal nerve branch)
and the back of the feet and toes (superficial peroneal nerve branch). Fibular nerve injury would not
cause loss of sensation in the anteromedial thigh or the decreased knee extension seen in this patient.

www.eduwaves360.com | Telegram : @eduwaves360


D - Obturator nerve injury

Explanation Why

Obturator nerve injury usually results from pelvic ring fractures or entrapment. It typically manifests
with loss of sensation of the medial thigh (cutaneous branch of the obturator nerve) and decreased
strength in the hip adductors. It would not explain this patient's loss of sensation in the anterior thigh
or her decreased strength in knee extension.

E - S1 radiculopathy

Explanation Why

S1 radiculopathy typically causes back pain that radiates down the posterior leg into the foot, with
sensory loss along the lateral foot, weakness in plantar flexion and eversion, and a decreased
Achilles tendon reflex. This patient's symptoms are localized to different areas and do not follow a
radicular pattern.

F - L5 radiculopathy

Explanation Why

L5 radiculopathy typically causes back pain that radiates down the lateral aspect of the leg into the
foot, with sensory loss along the lateral aspect of the lower leg and dorsum of the foot and strength
deficits in foot dorsiflexion and eversion. This patient's symptoms are localized to different areas and
do not follow a radicular pattern.

www.eduwaves360.com | Telegram : @eduwaves360


Question # 9

A 13-year-old boy is brought to the emergency room 30 minutes after being hit in the face with a
baseball at high velocity. Examination shows left periorbital swelling, posterior displacement of the left
globe, and tenderness to palpation over the left infraorbital rim. There is limited left upward gaze and
normal horizontal eye movement. Further evaluation is most likely to show which of the following as a
result of this patient's trauma?

Answer Image

A Lacrimal duct system injury

B Clouding of maxillary sinus

C Pneumatization of frontal sinus

D Cerebrospinal fluid leak

E Disruption of medial canthal ligament

www.eduwaves360.com | Telegram : @eduwaves360


Hint

The patient's findings of enophthalmos (posteriorly displaced eye) and infraorbital rim tenderness are
suggestive of an orbital floor fracture (orbital blowout fracture). The mechanism of this injury can lead
to downward displacement of orbital fat and inferior rectus muscle entrapment, which in turn can cause
impaired upward gaze.

www.eduwaves360.com | Telegram : @eduwaves360


Correct Answer

A - Lacrimal duct system injury

Explanation Why

Injury to the lacrimal duct system is associated with nasoethmoid fractures. Fracture of the maxillary
bone in this portion of the medial orbital rim can also result in disruption of the medial canthal
ligament, causing eyelid displacement and entrapment of the medial rectus muscle, which would
prevent horizontal eye movement. Nasoethmoid fractures are the second most common orbital
fractures after orbital floor fractures, but this patient has none of the accompanying signs, and such a
fracture would not explain his limited upward gaze or inferior orbital rim tenderness.

B - Clouding of maxillary sinus

Image

www.eduwaves360.com | Telegram : @eduwaves360


Explanation Why

Clouding of the maxillary sinus is a radiological sign known as the “tear drop sign.” It will most
likely be seen on further evaluation due to the inferior rectus muscle and orbital fat herniating
through the fractured orbital floor. Other symptoms may include diplopia, crepitus, ecchymosis, and
loss of sensation in the area innervated by the infraorbital nerve (midface from the lower eyelid to the
upper lip).

C - Pneumatization of frontal sinus

Explanation Why

Pneumatization of the frontal sinus is a part of normal development as bone marrow in the frontal
sinuses is slowly replaced by air starting at 2 years of age until adolescence. Pneumatization of the
frontal sinuses may also occur as a result of orbital trauma in cases of orbital roof fractures. These
fractures are rare in adults because of the strength of the orbital roof. They are more common in
children, who have a higher cranium-to-midface ratio relative to adults. Orbital roof fractures result
in CSF leaks and mucocele formation, none of which are present in this patient.

D - Cerebrospinal fluid leak

Explanation Why

Cerebrospinal fluid leak is seldom associated with orbital fractures. When CSF leak does occur, it is
typically due to an orbital roof fracture with extension to the frontal sinuses. Orbital roof fracture is
inconsistent with this patient's inferior orbital rim tenderness and limitation of upward gaze
(signifying inferior rectus muscle entrapment).

E - Disruption of medial canthal ligament

Explanation Why

Disruption of the medial canthal ligament is associated with nasoethmoid fractures. Fracture of the

www.eduwaves360.com | Telegram : @eduwaves360


maxillary bone in this portion of the orbital rim could also result in disruption of the lacrimal duct
system and entrapment of the medial rectus muscle, which would prevent horizontal eye movement.
While these nasoethmoid fractures are the second most common orbital fractures after orbital floor
fractures, this patient has normal horizontal eye movement, and such a fracture would not explain his
limited upward gaze or inferior orbital rim tenderness.

www.eduwaves360.com | Telegram : @eduwaves360


Question # 10

A 45-year-old man comes to the physician because of intermittent lower back pain for 1 week. His
symptoms began shortly after lifting heavy boxes at work. He has not had any fever, chills, or weight
loss. He has a history of peptic ulcer disease. He does not smoke or drink alcohol. His vital signs are
within normal limits. Examination shows mild paraspinal lumbar tenderness. Neurologic examination
shows no focal findings. An x-ray of the spine shows no abnormalities. Which of the following is the
most appropriate initial pharmacotherapy?

Answer Image

A Aspirin

B Acetaminophen

C Oxycodone

D Naproxen

E Ibuprofen

www.eduwaves360.com | Telegram : @eduwaves360


Hint

Prostaglandins protect the gastric epithelium by increasing gastric mucosal secretion and decreasing
gastric acid secretion. Prostaglandins that protect the gastric epithelium are predominantly produced by
COX-1, while prostaglandins that cause pain and inflammation are predominantly produced by COX-2.

www.eduwaves360.com | Telegram : @eduwaves360


Correct Answer

A - Aspirin

Explanation Why

Aspirin is an agent in the NSAID class. NSAIDs would be contraindicated in this patient with a
history of PUD as they reduce production of protective mucosal prostaglandins. Use of NSAIDs in
patients with PUD can increase the risk of complications including gastrointestinal bleeding and
perforation.

B - Acetaminophen

Explanation Why

NSAIDs (e.g., ibuprofen, naproxen, aspirin), which inhibit both COX-1 and COX-2 peripherally, are
contraindicated in this patient with PUD because they reduce synthesis of gastroprotective
prostaglandins. Acetaminophen, which is not an NSAID, has analgesic effects because it inhibits
COX in the CNS but it is inactivated peripherally and therefore does not cause gastric side effects.
Another treatment option for this patient would be selective COX-2 inhibitors (e.g., celecoxib),
which spare COX-1 in the gastric mucosa and, hence, spare mucosal prostaglandins.

C - Oxycodone

Explanation Why

Oxycodone as well as other opioids would not be indicated as a first-line therapy for a patient with
only mild musculoskeletal back pain. Opioids are reserved for treatment of severe or refractory back
pain (e.g., tumor pain).

www.eduwaves360.com | Telegram : @eduwaves360


D - Naproxen

Explanation Why

Naproxen is an agent in the NSAID class. NSAIDs would be contraindicated in this patient with a
history of PUD as they reduce production of the protective mucosal prostaglandins. Use of NSAIDs
in patients with PUD can increase the risk of complications including gastrointestinal bleeding and
perforation.

E - Ibuprofen

Explanation Why

Ibuprofen is an agent in the NSAID class. NSAIDs would be contraindicated in this patient with a
history of PUD as they reduce production of the protective mucosal prostaglandins. Use of NSAIDs
in patients with PUD can increase the risk of complications such as gastrointestinal bleeding and
perforation.

www.eduwaves360.com | Telegram : @eduwaves360


Question # 11

A 65-year-old man with chronic myelogenous leukemia comes to the physician because of severe pain
and swelling in both knees for the past day. He finished a cycle of chemotherapy 1 week ago. His
temperature is 37.4°C (99.4°F). Physical examination shows swelling and erythema of both knees and
the base of his left big toe. Laboratory studies show:

Leukocyte count 13,000/mm3

Serum

Creatinine 2.2 mg/dL

Calcium 8.2 mg/dL

Phosphorus 7.2 mg/dL

Arthrocentesis of the involved joints is most likely to show which of the following?

Answer Image

Granulocytes with cytoplasmic


A
inclusion bodies

B Monosodium urate crystals

www.eduwaves360.com | Telegram : @eduwaves360


Answer Image

C Calcium pyrophosphate crystals

D Calcium phosphate crystals

E Gram-negative diplococci

F Gram-positive cocci in clusters

www.eduwaves360.com | Telegram : @eduwaves360


Hint

This patient presents with elevated creatinine, hyperphosphatemia, and hypocalcemia one week after
chemotherapy. This is consistent with tumor lysis syndrome.

www.eduwaves360.com | Telegram : @eduwaves360


Correct Answer

A - Granulocytes with cytoplasmic inclusion bodies

Explanation Why

Granulocytes with cytoplasmic inclusion bodies, also known as ragocytes, would be consistent with
autoimmune-related arthropathies, including rheumatoid arthritis and SLE. They are not seen in
tumor lysis syndrome.

B - Monosodium urate crystals

Image

Explanation Why

Tumor lysis syndrome occurs when there is widespread lysis of cancer cells (e.g., from
chemotherapy), resulting in the release of large amounts of nucleic acid, which is further broken

www.eduwaves360.com | Telegram : @eduwaves360


down to monosodium urate (uric acid). The subsequent hyperuricemia can lead to the precipitation of
monosodium urate crystals in the renal tubules and joints, causing acute kidney injury and triggering
gout, as seen in this patient. Arthrocentesis of the involved joints would reveal needle-shaped,
negatively birefringent crystals.

C - Calcium pyrophosphate crystals

Image

Explanation Why

Deposition of calcium pyrophosphate crystals within the joint space is the pathomechanism of
calcium pyrophosphate deposition disease. Arthritis in calcium pyrophosphate deposition disease
commonly affects only one joint (usually the knee), and arthroscopy typically reveals rhomboid-
shaped, positively birefringent crystals. The disease is usually idiopathic, but can be caused by
hemochromatosis, hyperparathyroidism, or trauma to the joint. Calcium pyrophosphate crystal
deposition is not a sequela of tumor lysis syndrome.

www.eduwaves360.com | Telegram : @eduwaves360


D - Calcium phosphate crystals

Explanation Why

Calcium phosphate crystals are non-birefringent, globular crystals associated with degenerative joint
diseases, such as osteoarthritis. Patients with this condition have chronic joint disease of the weight-
bearing joints and morning stiffness lasting less than 30 minutes. While tumor lysis syndrome can
lead to deposition of calcium phosphate crystals in the kidneys (predisposing to acute kidney injury),
deposition of these crystals in the joints is not a sequela of tumor lysis syndrome.

E - Gram-negative diplococci

Explanation Why

Gram-negative diplococci in the synovial fluid culture would be concerning for gonococcal arthritis,
a type of oligoarthritis commonly seen in sexually active adults. It presents either as monoarthritis
(e.g., of the knee), or as polyarthralgia with dermatitis and tenosynovitis. Tumor lysis syndrome does
not predispose to gonococcal arthritis.

F - Gram-positive cocci in clusters

Explanation Why

Gram-positive cocci in clusters in the joint space would be consistent with septic arthritis due to
Staphylococcus aureus. Patients typically present with acute monoarticular joint pain, swelling, and
fever. Risk factors include prosthetic joints, skin infection, or IV drug abuse. Tumor lysis syndrome
does not predispose to septic arthritis.

www.eduwaves360.com | Telegram : @eduwaves360


Question # 12

A 56-year-old woman with rheumatoid arthritis comes to the physician for a follow-up examination. She
has no other history of serious illness. Menopause occurred 1 year ago. Current medications include
antirheumatic drugs and hormone replacement therapy. She exercises regularly. A DEXA scan shows a
T-score of -1.80, indicating decreased bone density. Which of the following drugs is most likely
involved in the pathogenesis of this finding?

Answer Image

A Sulfasalazine

B Medroxyprogesterone acetate

C Naproxen

D Prednisone

E Adalimumab

www.eduwaves360.com | Telegram : @eduwaves360


Hint

This patient's DEXA scan indicates osteopenia. Other adverse effects of the drug responsible for this
patient's osteopenia include hyperglycemia, peptic ulcers, and fat redistribution.

www.eduwaves360.com | Telegram : @eduwaves360


Correct Answer

A - Sulfasalazine

Explanation Why

Sulfasalazine is a nonbiologic DMARD used to treat a variety of inflammatory and rheumatological


conditions, including maintenance therapy for rheumatoid arthritis. Common side effects of
sulfasalazine include headache, rash, and GI disturbances. Sulfasalazine is not known to decrease
bone mineral density.

B - Medroxyprogesterone acetate

Explanation Why

Medroxyprogesterone acetate is a medication most commonly used as a contraceptive agent. It


mimics progesterone and can cause reversible bone mineral loss at high doses. This patient is
postmenopausal, making it unlikely that medroxyprogesterone acetate is the etiology of her
decreased bone mineral density.

C - Naproxen

Explanation Why

Gastrointestinal ulceration and renal insufficiency are the most common adverse effects seen with
chronic NSAID use (e.g., naproxen, ibuprofen). While NSAIDs are commonly used in patients with
rheumatoid arthritis, they are not known to decrease bone mineral density.

www.eduwaves360.com | Telegram : @eduwaves360


D - Prednisone

Explanation But

Bisphosphonates and denosumab (anti-RANKL antibody) are used to treat glucocorticoid-induced


osteoporosis.

Explanation Why

Patients with refractory rheumatoid arthritis are frequently prescribed glucocorticoids (e.g.,
prednisone), which induce bone resorption and increase the risk of osteopenia and osteoporosis.
Glucocorticoid-induced bone resorption is primarily mediated via the RANKL signaling pathway, a
cascade that upregulates osteoclast activity and induces apoptosis of osteoblasts. Chronic
glucocorticoid use can also induce mild hypocalcemia, which leads to PTH-mediated bone
resorption. In addition, decreased estrogen levels following menopause increase this patient's risk for
osteoporosis and osteopenia.

E - Adalimumab

Explanation Why

Adalimumab is a biologic drug used to inhibit TNF-α in patients with rheumatoid arthritis. TNF-α
inhibition has immunosuppressive effects and is commonly used for refractory RA. Common side
effects of TNF-α inhibitors include dermatitis, GI disturbances, and opportunistic infections.
Adalimumab is not known to decrease bone mineral density.

www.eduwaves360.com | Telegram : @eduwaves360


Question # 13

A 66-year-old woman comes to the physician for a routine health maintenance examination. She has no
history of serious medical illness and takes no medications. A screening DEXA scan shows a T-score of
-1.5 at the femur. Which of the following is the strongest predisposing factor for osteopenia?

Answer Image

A Hypoparathyroidism

B NSAID use

C Obesity

D Smoking

E Type 2 diabetes mellitus

www.eduwaves360.com | Telegram : @eduwaves360


Hint

This patient has been diagnosed with osteopenia, a disease characterized by decreased bone mass and
increased susceptibility to fractures.

www.eduwaves360.com | Telegram : @eduwaves360


Correct Answer

A - Hypoparathyroidism

Explanation Why

Hypoparathyroidism is characterized by low PTH that leads to impaired calcium homeostasis in the
bone, kidneys, and intestines. In the bone, deficient PTH levels result in reduced osteoclastic activity
and therefore higher bone mineral density rather than osteopenia. Chronic hyperparathyroidism, on
the other hand, is a risk factor for osteopenia.

B - NSAID use

Explanation Why

NSAID use is not associated with increased risk of osteopenia and osteoporosis. Instead, low-dose
NSAID use is associated with increased bone mineral density. This is thought to be due to COX-2
inhibition, which may decrease osteoclastic activity.

C - Obesity

Explanation Why

Obesity is not associated with decreased bone mineral density. Low BMI (< 21 kg/m2) and
malnutrition, on the other hand, are risk factors for osteopenia and osteoporosis.

www.eduwaves360.com | Telegram : @eduwaves360


D - Smoking

Explanation Why

Smoking is a major risk factor for both osteopenia and osteoporosis. In postmenopausal women who
smoke, an additional 2% of bone density loss per decade of life can be attributed to smoking. This
equates to a significantly higher incidence of osteoporosis and pathologic fractures in
postmenopausal women who smoke compared to nonsmokers. Other major risk factors include
advanced age, positive family history, alcohol abuse, sedentary lifestyle, low body weight, vitamin D
deficiency, and cortisone use/hypercortisolism.

E - Type 2 diabetes mellitus

Explanation Why

Type 2 diabetes mellitus is not a risk factor for decreased bone mass density and osteoporosis. T2DM
can, however, increase fracture risk in elderly adults due to peripheral neuropathy, which increases
the risk of falling. It is therefore an important comorbidity to consider in patients with osteopenia and
osteoporosis.

www.eduwaves360.com | Telegram : @eduwaves360


Question # 14

A 72-year-old man comes to the emergency room because of a 4-day history of progressively worsening
pain and swelling on the right side of his face. The patient was diagnosed with multiple myeloma
8 months ago and is currently undergoing treatment. His vital signs are within normal limits. Physical
examination shows erythema and swelling over the right cheek and mandible. An orofacial fistula is
present. Which of the following best describes the mechanism of action of the drug that is most likely
responsible for this patient's symptoms?

Answer Image

A Inhibition of angiogenesis

B Cross-linking DNA at guanine

C Inhibition of proteasome activity

D Stimulation of PTH receptor

E Binding to hydroxyapatite

www.eduwaves360.com | Telegram : @eduwaves360


Hint

This patient's painful swelling, erythema, and orofacial fistula indicate osteonecrosis of the jaw.

www.eduwaves360.com | Telegram : @eduwaves360


Correct Answer

A - Inhibition of angiogenesis

Explanation But

Inhibition of angiogenesis is a mechanism of action of lenalidomide, a drug regularly used in the


treatment of multiple myeloma (in combination with low-dose dexamethasone). Significant adverse
effects include venous thromboembolism, hematologic toxicity, and dermatologic reactions (e.g.,
Stevens-Johnson syndrome). Lenalidomide is not associated with osteonecrosis of the jaw.

Explanation Why

Inhibition of angiogenesis is the mechanism of action of bevacizumab, a drug that is typically used to
treat solid tumors (e.g., colorectal cancer, renal cell carcinoma) and wet age-related macular
degeneration. Significant adverse effects include hemorrhage, blood clots, and impaired wound
healing. Bevacizumab is not used to treat multiple myeloma and is not associated with osteonecrosis
of the jaw.

B - Cross-linking DNA at guanine

Explanation Why

Cross-linking DNA at guanine is the mechanism of action of cyclophosphamide, a drug regularly


used in the treatment of multiple myeloma. Major adverse effects include hemorrhagic cystitis and
SIADH. Cyclophosphamide is not associated with osteonecrosis of the jaw.

C - Inhibition of proteasome activity

Explanation Why

Inhibition of proteasome activity is the mechanism of action of bortezomib, a cytostatic drug that
arrests cell growth at the G2-M phase and causes apoptosis. It is first-line treatment for multiple

www.eduwaves360.com | Telegram : @eduwaves360


myeloma and is associated with polyneuropathy and reactivation of herpes zoster. It does not cause
osteonecrosis of the jaw.

D - Stimulation of PTH receptor

Explanation Why

Stimulation of PTH receptors is the mechanism of action of teriparatide. Teriparatide can enhance
bone resorption by stimulating osteoclastic activity, but the net increase in bone formation through
upregulation of osteoblastic activity predominates. Teriparatide is associated with the development of
osteosarcoma and may cause transient hypercalcemia, but it does not cause osteonecrosis of the jaw.

E - Binding to hydroxyapatite

Explanation Why

Binding to hydroxyapatite on the surface of bone tissue is the mechanism of action of


bisphosphonates, a group of drugs used in the treatment of multiple myeloma. Bisphosphonates are
pyrophosphate analogs that decrease bone resorption by inhibiting osteoclast activity. The
pathophysiology of bisphosphonate-induced osteonecrosis of the jaw is not yet fully understood.
Trauma (e.g., during dental procedures) may act as a trigger. Other important adverse effects of
bisphosphonate therapy include hypocalcemia, renal impairment, and esophagitis.

www.eduwaves360.com | Telegram : @eduwaves360


Question # 15

A 13-year-old girl is brought to the physician by her father because of a 1-month history of pain in her
right knee. She is a competitive volleyball player and has missed several games recently due to pain.
Examination shows swelling distal to the right knee joint on the anterior surface of the proximal tibia;
there is no overlying warmth or deformity. Extension of the right knee against resistance is painful.
Which of the following structures is attached to the affected anterior tibial area?

Answer Image

A Anterior cruciate ligament

B Patellar ligament

C Quadriceps tendon

D Iliotibial band

E Pes anserinus tendon

www.eduwaves360.com | Telegram : @eduwaves360


Hint

This patient's presentation is consistent with Osgood-Schlatter disease, a common cause of anterior knee
pain in growing adolescents, especially those involved in running and jumping sports.

www.eduwaves360.com | Telegram : @eduwaves360


Correct Answer

A - Anterior cruciate ligament

Explanation Why

The anterior cruciate ligament (ACL) connects the lateral femoral condyle and the anteromedial tibia
within the knee joint. Sprains and tears of the ACL occur suddenly following injuries with a twisting
mechanism and present with knee pain and instability. Examination typically shows anterior knee
laxity (positive Lachman test).

B - Patellar ligament

Image

Explanation Why

The patellar ligament connects the patella to the tibial tuberosity and allows for full extension of the

www.eduwaves360.com | Telegram : @eduwaves360


knee via the quadriceps muscle. Osgood-Schlatter disease arises due to overuse of the quadriceps
muscle during a period of growth (adolescence), which puts excessive strain on the attachment of the
patellar ligament at the tibial tuberosity, resulting in painful inflammation and avascular necrosis.
Patients present with pain during quadriceps contraction.

C - Quadriceps tendon

Explanation Why

Although injury to the quadriceps tendon can also present with anterior knee pain that is worse
during extension of the knee against resistance, the tendon connects the quadriceps femoris muscle to
the superior aspect of the patella, not the tibial tuberosity.

D - Iliotibial band

Explanation Why

Inflammation of the iliotibial band (ITB) is a common cause of knee pain secondary to excessive
running, jumping, or overstretching. However, it typically causes sharp pain along the lateral side of
the knee, as the ITB inserts at the lateral condyle of the tibia, not the tibial tuberosity.

E - Pes anserinus tendon

Explanation Why

Chronic overuse of the pes anserinus tendon, which connects the sartorius, gracilis, and
semitendinosus muscles to the anteromedial aspect of the tibia, can cause pes anserine bursitis.
Patients present with medial knee pain and swelling. Extension of the knee against resistance would
not elicit pain.

www.eduwaves360.com | Telegram : @eduwaves360


Question # 16

A 28-year-old man comes to the physician because of a 3-month history of pain in his left shoulder. He
is physically active and plays baseball twice a week. The pain is reproduced when the shoulder is
externally rotated against resistance. Injury of which of the following tendons is most likely in this
patient?

Answer Image

A Teres major

Pectoralis
B
major

C Infraspinatus

www.eduwaves360.com | Telegram : @eduwaves360


Answer Image

D Supraspinatus

E Subscapularis

www.eduwaves360.com | Telegram : @eduwaves360


Hint

The rotator cuff tendons, which help to stabilize the glenohumeral joint through its broad range of
motion, are commonly injured in overhead throwing athletes.

www.eduwaves360.com | Telegram : @eduwaves360


Correct Answer

A - Teres major

Image

Explanation Why

The teres major contributes to internal rotation, extension, and adduction of the humerus. This
muscle is not part of the rotator cuff. Tendinopathy of the teres major is rare; such cases would
present with pain that occurs during resisted internal rotation of the humerus. The teres minor is an
external rotator of the humerus and could be affected in this patient.

www.eduwaves360.com | Telegram : @eduwaves360


B - Pectoralis major

Image

Explanation Why

The pectoralis major is a large muscle on the anterior chest that is responsible for adduction, internal
rotation, and flexion of the humerus. Tendon avulsion, or rupture of the pectoralis major, can occur
in bodybuilders as a result of excessive muscle loading and is accompanied by a sharp, tearing
sensation, weakness, swelling, and ecchymosis. This patient has pain during external rotation of the
shoulder, which suggests the injury of a different tendon.

www.eduwaves360.com | Telegram : @eduwaves360


C - Infraspinatus

Image

Explanation Why

Overhead throwing activities (e.g., baseball pitching) place supraphysiological loads on the tendons
of the rotator cuff. Through eccentric contraction, this type of activity especially strains the posterior
rotator cuff when slowing down after releasing the ball. Tendon overload can cause microscopic tears
called microinjuries. The relatively avascular nature of tendons means that these microinjuries have
limited healing potential and can propagate into partial or full-thickness tears, or tendinopathy. The
infraspinatus muscle, along with the teres minor muscle, is responsible for the external rotation of the
humerus. This patient's shoulder pain during external rotation suggests infraspinatus tendinopathy.

www.eduwaves360.com | Telegram : @eduwaves360


D - Supraspinatus

Image

Explanation Why

The supraspinatus is a rotator cuff muscle that is responsible for shoulder abduction in the scapular
plane (30° of forward shoulder flexion). The supraspinatus is the most commonly affected tendon in
rotator cuff tears. However, reproduction of this patient's pain during shoulder external rotation
suggests the involvement of a different tendon, as the supraspinatus does not perform external
rotation.

www.eduwaves360.com | Telegram : @eduwaves360


E - Subscapularis

Image

Explanation Why

The subscapularis is a rotator cuff muscle that is responsible for internal rotation of the humerus.
Injury of this tendon would present with pain on resisted internal rotation during a bear hug or belly-
press test. This patient has pain during external rotation, which suggests the injury of a different
tendon.

www.eduwaves360.com | Telegram : @eduwaves360


Question # 17

A 4-year-old girl is brought to the emergency department after falling out from a chair and injuring her
right leg. During the past 2 years, she has had two long bone fractures. She is at the 5th percentile for
height and 20th percentile for weight. Her right lower leg is diffusely erythematous. The patient
withdraws and yells when her lower leg is touched. A photograph of her face is shown. An x-ray of the
right lower leg shows a transverse mid-tibial fracture with diffusely decreased bone density. Which of
the following is the most likely cause of this patient's symptoms?

Answer Image

A Type 3 collagen defect

B Type 5 collagen defect

www.eduwaves360.com | Telegram : @eduwaves360


Answer Image

C Type 1 collagen defect

D Type 4 collagen defect

E Type 2 collagen defect

www.eduwaves360.com | Telegram : @eduwaves360


Hint

This patient presents with growth retardation, recurrent bone fractures, blue sclera, and osteopenia, all of
which are consistent with the diagnosis of osteogenesis imperfecta.

www.eduwaves360.com | Telegram : @eduwaves360


Correct Answer

A - Type 3 collagen defect

Explanation Why

The vascular type of Ehlers-Danlos syndrome (type 4) arises from an autosomal mutation in collagen
type 3. This condition manifests predominantly in the cardiovascular system and often goes
undiagnosed until it becomes apparent through some sort of vascular damage/injury (e.g., aortic
dissection, ruptures arterial aneurysm, subarachnoid hemorrhage). The child presents with easy
bruising, as evidenced by the ecchymosis, which is also a feature of Ehlers-Danlos type 4. However,
the child's history of frequent fractures after minor trauma, the blue sclerae, and growth retardation
are not consistent with this condition.

B - Type 5 collagen defect

Explanation Why

Type 5 collagen is a regulator of type 1 collagen fibrillogenesis. Type 5 collagen defects usually
manifest as classic Ehlers-Danlos syndrome, which is characterized by chronic joint and bone issues.
Rather than frequent bone fractures, the major symptoms are joint instability, early onset of
osteoarthritis, hyperflexible joints, and scoliosis.

www.eduwaves360.com | Telegram : @eduwaves360


C - Type 1 collagen defect

Image

Explanation But

The prognosis depends heavily on the type of osteogenesis imperfecta. People with type 1 often have
a normal life expectancy, no long-bone deformities, and few childhood fractures. Type 2
osteogenesis imperfecta is the most severe form, with most affected children dying in utero or within
the first year of life due to respiratory failure secondary to underdeveloped lungs or intracerebral
hemorrhage.

Explanation Why

A mutation in type 1 collagen causes osteogenesis imperfecta type 1, the mildest and most common
form of osteogenesis imperfecta. In fact, some patients with osteogenesis imperfecta type 1 appear
normal and without obvious deformities to the casual observer despite requiring special
accommodations for their bone fragility. This child presents with blue sclera, short stature, and
growth retardation, recurrent fractures from minor falls, and decreased bone density – all classic
symptoms of osteogenesis imperfecta.

www.eduwaves360.com | Telegram : @eduwaves360


D - Type 4 collagen defect

Explanation Why

Mutations to the genes encoding type 4 collagen can present as Alport syndrome. This disorder
usually affects boys, who present with intermittent gross hematuria, hearing loss, and ocular
disturbances, none of which are seen here. Recurrent bone fractures, growth retardation, and blue
sclerae are not features of Alport syndrome.

E - Type 2 collagen defect

Explanation Why

Type 2 collagen defects predominantly affect tissues such as cartilage and the vitreous humor.
Various conditions are associated with type 2 collagen defects (e.g., hypochondrogenesis, Kniest
dysplasia, or osteoarthritis) and these differ substantially in presentation and age of onset. While
some conditions could explain the growth retardation observed in this patient (e.g.,
hypochondrogenesis, Kniest dysplasia), blue sclerae and recurrent bone fractures in an otherwise
healthy child would not be consistent with any type 2 collagen defect.

www.eduwaves360.com | Telegram : @eduwaves360


Question # 18

A 16-year-old boy comes to the physician because of muscle weakness and cramps for 5 months. He
becomes easily fatigued and has severe muscle pain and swelling after 15 minutes of playing basketball
with his friends. The symptoms improve after a brief period of rest. After playing, he sometimes also has
episodes of reddish-brown urine. There is no family history of serious illness. Serum creatine kinase
concentration is 950 U/L. Urinalysis shows:

Blood 2+

Protein negative

Glucose negative

RBC negative

WBC 1–2/hpf

Which of the following is the most likely underlying cause of this patient's symptoms?

Answer Image

A CTG repeat in the DMPK gene

B Low levels of triiodothyronine and thyroxine

C Dystrophin gene mutation

D Acid maltase deficiency

E Medium-chain acyl-CoA dehydrogenase deficiency

www.eduwaves360.com | Telegram : @eduwaves360


Answer Image

F Myophosphorylase deficiency

www.eduwaves360.com | Telegram : @eduwaves360


Hint

This patient's muscle cramps and pain after physical activity, second wind phenomenon, myoglobinuria
(red-brown urine with positive blood but no RBCs), elevated serum creatine kinase are indicative of
McArdle's disease.

www.eduwaves360.com | Telegram : @eduwaves360


Correct Answer

A - CTG repeat in the DMPK gene

Explanation Why

CTG repeats in the DMPK gene cause myotonic dystrophy. While myotonic dystrophy can be
associated with muscle pain and weakness as in this patient, it causes symptoms at rest and is
associated with delayed muscle relaxation following contraction (myotonia). Additionally, myotonic
dystrophy is associated with arrhythmias, cataracts, testicular atrophy, and insulin resistance.

B - Low levels of triiodothyronine and thyroxine

Explanation Why

Low thyroid hormone levels cause hypothyroidism, which can present with myopathy as seen in this
patient. However, these symptoms would be present at rest rather than improve with rest.
Additionally, hypothyroidism would likely present with other general symptoms such as fatigue, cold
intolerance, cold skin, weight gain, constipation, delayed reflexes, or myxedema. Finally,
hypothyroidism is uncommon in otherwise healthy teenage boys (5–8 times more common in
women).

C - Dystrophin gene mutation

Explanation Why

Dystrophin gene mutations cause Duchenne muscular dystrophy or Becker muscular dystrophy,
depending on the type of mutation. While these disorders can result in myalgias and elevated creatine
kinase, as in this patient, they are usually progressive conditions that would demonstrate symptoms
both at rest and with exercise. Additionally, symptoms in dystrophinopathies would not improve so
significantly with rest. Lastly, Duchenne muscular dystrophy typically presents before age 5 and
usually leads to ambulatory inability before age 13.

www.eduwaves360.com | Telegram : @eduwaves360


D - Acid maltase deficiency

Explanation Why

Lysosomal acid maltase deficiency (Pompe disease) is a glycogen storage disease that can present
with myopathy and exercise intolerance as in this patient. However, Pompe disease is usually
diagnosed in infants and young children who present with failure to thrive along with myopathy and,
sometimes, hypertrophic cardiomyopathy and/or macroglossia. Moreover, the rapid improvement of
this patient's symptoms following rest is characteristic of another disease.

E - Medium-chain acyl-CoA dehydrogenase deficiency

Explanation Why

MCAD deficiency can present with exercise intolerance and elevated serum creatinine kinase due to
muscle catabolism. However, this weakness would develop with more prolonged exercise, not within
a few minutes. Additionally, patients with MCAD deficiency would not experience the rapid
symptom improvement seen in this patient.

F - Myophosphorylase deficiency

Explanation Why

Myophosphorylase deficiency (McArdle's disease) is a glycogen storage disease that presents with
myalgias, rhabdomyolysis, myoglobinuria, and early fatigue from exercise due to inability to break
down glycogen for use as energy in muscle tissues. Many patients experience the second wind
phenomenon, in which symptoms improve after a brief period of rest. A hallmark of McArdle's
disease is a flat venous lactate curve with exaggerated elevations in blood ammonia during exercise.
Treatment often involves aerobic exercise programs and nutritional supplementation, although
evidence of benefit is limited.

www.eduwaves360.com | Telegram : @eduwaves360


Question # 19

A previously healthy 2-year-old boy is brought to the emergency room by his mother because of
persistent crying and refusal to move his right arm. The episode began 30 minutes ago after the mother
lifted him up by the arms. He appears distressed and is inconsolable. On examination, his right arm is
held close to his body in a flexed and pronated position. Which of the following is the most likely
diagnosis?

Answer Image

A Proximal ulnar fracture

Anterior shoulder
B
dislocation

www.eduwaves360.com | Telegram : @eduwaves360


Answer Image

Supracondylar fracture of
C
the humerus

D Olecranon fracture

E Radial head subluxation

www.eduwaves360.com | Telegram : @eduwaves360


Hint

This patient presents with the most common elbow injury under 5 years of age. It is caused by sudden
axial traction of the pronated and extended forearm.

www.eduwaves360.com | Telegram : @eduwaves360


Correct Answer

A - Proximal ulnar fracture

Image

Explanation Why

A proximal ulnar fracture can occur alongside a radial head subluxation and is called a Monteggia
fracture. This fracture is typically caused by either a fall on an outstretched arm or a direct forceful
blow to the elbow. Considering the history of this patient, this fracture is unlikely. The clinical
presentation would also be more severe.

www.eduwaves360.com | Telegram : @eduwaves360


B - Anterior shoulder dislocation

Image

Explanation Why

An anterior shoulder dislocation causes severe shoulder pain. Unlike in this patient, it usually occurs
when falling on an outstretched arm, and the arm is typically held in external rotation and slight
abduction. Common complications are axillary nerve and rotator cuff injury.

www.eduwaves360.com | Telegram : @eduwaves360


C - Supracondylar fracture of the humerus

Image

Explanation Why

Supracondylar humerus fractures are the most common pediatric fractures. They mainly occur in the
age group of 5–8 years. This fracture is unlikely to occur from pure traction on an arm. Additionally,
we would expect local swelling and potentially a lesion of the median nerve with loss of wrist
flexion, thumb opposition, and loss of thenar sensation.

D - Olecranon fracture

Explanation Why

An olecranon fracture occurs through direct hits to the elbow or falls, which did not happen to this
patient. Examination would show local elbow swelling and maybe dislocation of the olecranon.

www.eduwaves360.com | Telegram : @eduwaves360


E - Radial head subluxation

Explanation Why

Radial head subluxation describes the slipping of the annular ligament over the radial head. Patients
classically present with the arm held in pronated and flexed position, as in this scenario. It usually
occurs when adults pull on children's arms to hold them back or to swing them around. The radial
head is relocated by hyperpronation or supination of the flexed forearm.

www.eduwaves360.com | Telegram : @eduwaves360


Question # 20

A 14-year-old boy is brought to the physician by his mother because of a 1-month history of pain in his
right leg. His mother has been giving him ketorolac at night to improve his sleep, but the pain has not
improved. Physical examination shows marked tenderness along the right mid-femur. An x-ray of the
right lower extremity shows several lytic lesions in the diaphysis of the femur and a surrounding cortex
covered by several layers of new bone. A biopsy of the right femur shows small round blue cells. Which
of the following is the most likely diagnosis?

Answer Image

A Ewing sarcoma

B Chondroblastoma

www.eduwaves360.com | Telegram : @eduwaves360


Answer Image

C Osteochondroma

D Osteosarcoma

E Chondrosarcoma

www.eduwaves360.com | Telegram : @eduwaves360


Answer Image

F Osteoid osteoma

www.eduwaves360.com | Telegram : @eduwaves360


Hint

Further diagnostics will likely reveal a t(11;22) chromosomal translocation.

www.eduwaves360.com | Telegram : @eduwaves360


Correct Answer

A - Ewing sarcoma

Image

Explanation Why

Ewing sarcoma typically occurs in patients < 20 years of age and involves the diaphysis of long
bones. X-ray usually reveals a tumor with multiple lytic lesions that may extend into the surrounding
soft tissue and an aggressive lamellated periosteal reaction. A biopsy shows small blue cells with a
high nuclear-cytoplasmic ratio. Systemic symptoms, as seen in this patient, are common. A t(11;22)
chromosomal translocation, resulting in expression of fusion protein EWS-FLI 1, is found in approx.
90% of cases of Ewing sarcoma.

www.eduwaves360.com | Telegram : @eduwaves360


B - Chondroblastoma

Image

Explanation Why

While this patient's age, x-ray findings, and local symptoms would fit the diagnosis of
chondroblastoma, this condition almost always occurs in the epiphysis and manifests as a single lytic
lesion with well-defined margins. Symptoms are less severe and progress more slowly (over a period
of 1–2 years) than those experienced by this patient. A biopsy would show a bluish chondroid matrix
and a characteristic chicken-wire calcification pattern.

www.eduwaves360.com | Telegram : @eduwaves360


C - Osteochondroma

Image

Explanation Why

An osteochondroma typically presents as a painless bone swelling in the metaphyses of long bones,
rather than the diaphysis as seen here. X-ray would reveal a metaphyseal bony protrusion with
variable degrees of sclerosis and a cartilaginous cap. Unlike in this patient, a biopsy of
osteochondroma would typically show abundant cartilage matrix with irregular lobules and fibrous
band formation.

www.eduwaves360.com | Telegram : @eduwaves360


D - Osteosarcoma

Image

Explanation Why

Osteosarcomas typically occur in the long bones of adolescents between 10–20 years of age and
present with painful swelling and progressive symptoms over the course of a few weeks to months
similar to those experienced by this patient. However, they usually arise at the metaphysis, have a
mixed lytic-sclerotic appearance, and involve an aggressive periosteal reaction with perpendicular
new bone growth and irregular cortical destruction. A biopsy would reveal poorly formed trabecular
bone.

www.eduwaves360.com | Telegram : @eduwaves360


E - Chondrosarcoma

Image

Explanation Why

A chondrosarcoma appears as a focal osteolytic lesion on x-ray with characteristic ring-and-arc


calcifications and may present with painful swelling. However, chondrosarcomas are extremely rare
in patients < 30 years of age and would classically involve the pelvis, not the diaphysis of the femur.
A biopsy would reveal hyaline cartilage with a chondroid matrix and atypical binucleate cells.

www.eduwaves360.com | Telegram : @eduwaves360


F - Osteoid osteoma

Image

Explanation Why

Osteoid osteomas typically affect patients < 30 years old in the diaphysis of a long bone and manifest
with nocturnal pain, as seen here. However, x-ray would usually show a single lytic nidus surrounded
by a dense sclerotic reaction. Unlike in this patient, the pain of osteoid osteoma would be responsive
to NSAIDs such as ketorolac.

www.eduwaves360.com | Telegram : @eduwaves360


Question # 21

A 61-year-old woman comes to the physician because of a 6-month history of left knee pain and
stiffness. Examination of the left knee shows tenderness to palpation along the joint line; there is
crepitus with full flexion and extension. An x-ray of the knee shows osteophytes with joint-space
narrowing. Arthrocentesis of the knee joint yields clear fluid with a leukocyte count of 120/mm3.
Treatment with ibuprofen during the next week significantly improves her condition. The beneficial
effect of this drug is most likely due to inhibition of which of the following?

Answer Image

A Conversion of dihydroorotate to orotate

B Conversion of hypoxanthine to urate

C Conversion of prostaglandin H2 to thromboxane A2

D Conversion of arachidonic acid to prostaglandin G2

E Conversion of phospholipids to arachidonic acid

www.eduwaves360.com | Telegram : @eduwaves360


Hint

Ibuprofen is a reversible NSAID that inhibits both COX-1 and COX-2 enzymes.

www.eduwaves360.com | Telegram : @eduwaves360


Correct Answer

A - Conversion of dihydroorotate to orotate

Explanation Why

Conversion of dihydroorotate to orotate is catalyzed by dihydroorotate dehydrogenase, a pyrimidine


synthesis enzyme inhibited by antiproliferative and anti-inflammatory drugs like leflunomide, not
NSAIDs. Leflunomide is used in the treatment of rheumatoid arthritis and psoriatic arthritis.

B - Conversion of hypoxanthine to urate

Explanation Why

Conversion of hypoxanthine to urate is catalyzed by xanthine oxidase, an enzyme inhibited by


allopurinol, not NSAIDs. Allopurinol is used in the treatment of chronic gout and hyperuricemia. It
does not possess intrinsic anti-inflammatory properties.

C - Conversion of prostaglandin H2 to thromboxane A2

Explanation Why

Conversion of prostaglandin H2 to thromboxane A2 is catalyzed by thromboxane synthase, a platelet


enzyme inhibited by some antithrombotic drugs, not NSAIDs. Moreover, reducing thromboxane A2
synthesis would not result in an analgesic or anti-inflammatory effect.

www.eduwaves360.com | Telegram : @eduwaves360


D - Conversion of arachidonic acid to prostaglandin G2

Explanation Why

Cyclooxygenases (COX-1 and COX-2) catalyze the conversion of arachidonic acid to prostaglandins
(e.g., prostaglandin G2), prostacyclin, and thromboxane A2. Because prostaglandins are involved in
regulating pain receptor sensitivity, body temperature, and inflammatory processes, blocking their
synthesis results in analgesic, anti-inflammatory, and antipyretic effects.

E - Conversion of phospholipids to arachidonic acid

Explanation Why

Conversion of cell membrane phospholipids to arachidonic acid is catalyzed by phospholipase A2, an


enzyme inhibited by glucocorticoids, not NSAIDs. Reduction of all arachidonic acid derivatives
attenuates inflammatory processes and intra-articular glucocorticoid injections can be used as a
second-line short-term treatment of osteoarthritis.

www.eduwaves360.com | Telegram : @eduwaves360


Question # 22

A 42-year-old woman comes to the physician because of a 10-month history of joint pain and stiffness in
her wrists and fingers. The symptoms are worse in the morning and improve with activity. Physical
examination shows swelling and warmth over the MCP and wrist joints in both hands. An x-ray of the
hands is shown. Synovial biopsy from an affected joint would most likely show which of the following?

Answer Image

Noninflammatory superficial
A
fibrin deposits

B Monosodium urate crystals

Calcium pyrophosphate
C
crystals

www.eduwaves360.com | Telegram : @eduwaves360


Answer Image

D Noncaseating granulomas

Proliferation of granulation
E
tissue

www.eduwaves360.com | Telegram : @eduwaves360


Hint

The presence of morning stiffness and joint pain affecting the wrists and fingers, symmetrical swelling
and warmth over the joints, and an x-ray showing a generalized decrease of bone radiodensity and
narrowing of the joint spaces of metacarpophalangeal, proximal interphalangeal, and wrist joints
indicate rheumatoid arthritis in this patient.

www.eduwaves360.com | Telegram : @eduwaves360


Correct Answer

A - Noninflammatory superficial fibrin deposits

Explanation Why

Noninflammatory superficial fibrin deposits can be seen on biopsy of joint specimens in patients
with scleroderma. While arthritis can be an initial presenting symptom in scleroderma, it does not
lead to the joint destruction as observed on this patient's x-ray.

B - Monosodium urate crystals

Explanation Why

Monosodium urate crystals, which are either found in synovial fluid obtained by arthrocentesis or
tissue samples of a tophus, are typically seen in gout. Gout most commonly causes acute episodes of
monoarticular pain (classically podagra). However, this patient presents with symptoms and imaging
findings that are highly suggestive of rheumatoid arthritis.

C - Calcium pyrophosphate crystals

Explanation Why

Synovial fluid containing calcium pyrophosphate crystals is seen in pseudogout and osteoarthritis,
which also cause joint pain. However, this patient has had joint symptoms for several months,
making an acute arthropathy like pseudogout very unlikely. Moreover, the imaging findings are
highly suggestive of rheumatoid arthritis.

www.eduwaves360.com | Telegram : @eduwaves360


D - Noncaseating granulomas

Explanation Why

Noncaseating granulomas are seen in sarcoidosis. While arthritis is a common feature in sarcoidosis,
it most often presents as oligoarthritis (2–4 joints) with ankle involvement (> 85% of cases).
Furthermore, sarcoid arthropathy is not typically a destructive condition; joint space narrowing and
marginal erosions seen on x-ray would be very unusual.

E - Proliferation of granulation tissue

Image

Explanation Why

This patient has evidence of advanced rheumatoid arthritis, based on the findings of marginal
erosions on imaging. Microscopic evaluation of a synovial biopsy would most likely show
granulation tissue with mononuclear cells, also referred to as pannus. Synovial fluid analysis in this

www.eduwaves360.com | Telegram : @eduwaves360


patient would be expected to show neutrophilic infiltrate, increased proteins, reduced viscosity, and
possibly rheumatoid factor.

www.eduwaves360.com | Telegram : @eduwaves360


Question # 23

A 49-year-old woman comes to the physician because of a 4-month history of fatigue and recurrent pain
in both of her wrists and her fingers. During this time, she has also had stiffness of her joints for about
80 minutes after waking up in the morning. Examination shows swelling and tenderness of the wrists
and metacarpophalangeal joints bilaterally. Her serum erythrocyte sedimentation rate is 42 mm/h and
rheumatoid factor is positive. Treatment is begun with a drug that results in decreased synthesis of
deoxythymidine monophosphate. This mechanism is most similar to the mechanism of action of which
of the following drugs?

Answer Image

A Sulfamethoxazole

B Doxycycline

C Trimethoprim

D Gentamicin

www.eduwaves360.com | Telegram : @eduwaves360


Answer Image

E Ciprofloxacin

F Azithromycin

www.eduwaves360.com | Telegram : @eduwaves360


Hint

This patient presents with signs and symptoms of rheumatoid arthritis, including fatigue, symmetric
polyarthritis of the small joints, prolonged morning stiffness, elevated ESR, and positive rheumatoid
factor. The drug she was prescribed is methotrexate, the disease-modifying antirheumatic drug of
choice, which targets the enzyme dihydrofolate reductase.

www.eduwaves360.com | Telegram : @eduwaves360


Correct Answer

A - Sulfamethoxazole

Explanation Why

Sulfamethoxazole inhibits dihydropteroate synthase (DHPS), which is a key enzyme in the folate
pathway of bacteria. Inhibition of DHPS results in impaired bacterial growth due to decreased
synthesis of dihydrofolic acid. However, methotrexate has a different mechanism of action.

B - Doxycycline

Explanation Why

Tetracyclines such as doxycycline inhibit bacterial protein synthesis by binding to the 30S subunit of
the ribosome, thus preventing the attachment of aminoacyl-tRNA to the A site on the ribosome. This
prevents new amino acids from being introduced to the growing peptide chain. However,
methotrexate has a different mechanism of action.

www.eduwaves360.com | Telegram : @eduwaves360


C - Trimethoprim

Image

Explanation Why

Trimethoprim (TMP) inhibits dihydrofolate reductase, a mechanism of action shared with other
drugs such as methotrexate and pyrimethamine. Dihydrofolate reductase reduces dihydrofolic acid to
tetrahydrofolic acid (THF), which can subsequently be converted to methylene-THF. Methylene-
THF is an important cofactor for thymidylate synthase, which catalyzes the conversion of
deoxyuridine monophosphate (dUMP) to deoxythymidine monophosphate (dTMP). Inhibition of
dihydrofolate reductase results in decreased synthesis of dTMP, impaired de novo synthesis of
pyrimidines, and, consequently, decreased bacterial proliferation.

www.eduwaves360.com | Telegram : @eduwaves360


D - Gentamicin

Image

Explanation Why

Aminoglycosides (including gentamicin) inhibit bacterial protein synthesis by binding to the 30S
subunit of the bacterial ribosome and subsequently inhibit the translation initiation complex. They
can also inhibit the ribosomal translocation process, which causes misreading of the mRNA.
However, methotrexate has a different mechanism of action.

E - Ciprofloxacin

Explanation Why

Fluoroquinolones such as ciprofloxacin directly inhibit bacterial replication by blocking the


prokaryotic enzymes topoisomerase II (DNA gyrase) and topoisomerase IV, which results in
impaired unwinding and duplication of the bacterial DNA. However, methotrexate has a different

www.eduwaves360.com | Telegram : @eduwaves360


mechanism of action.

F - Azithromycin

Image

Explanation Why

Macrolides (including azithromycin) inhibit bacterial protein synthesis by binding to the 50S subunit
of the ribosomes, which inhibits the ribosomal translocation process, resulting in premature
detachment of incomplete polypeptide chains. However, methotrexate has a different mechanism of
action.

www.eduwaves360.com | Telegram : @eduwaves360


Question # 24

A 15-year-old boy is brought to the physician because of progressive left leg pain for the past 2 months.
The pain is worse while running and at night. Examination of the left leg shows swelling and tenderness
proximal to the knee. Laboratory studies show an alkaline phosphatase level of 200 U/L. An x-ray of the
left leg shows sclerosis, cortical destruction, and new bone formation in the soft tissues around the distal
femur. There are multiple spiculae radiating perpendicular to the bone. This patient's malignancy is most
likely derived from cells in which of the following structures?

Answer Image

A Periosteum

B Bone marrow

C Cartilage

D Epiphyseal plate

www.eduwaves360.com | Telegram : @eduwaves360


Answer Image

E Neural crest

www.eduwaves360.com | Telegram : @eduwaves360


Hint

This boy presents with a tender swelling of his knee, elevated alkaline phosphatase levels, and an x-ray
showing a Codman triangle (protrusion that extends from the primary site) and sunburst appearance
(formation of spicules). Given his age, he most likely has osteosarcoma.

www.eduwaves360.com | Telegram : @eduwaves360


Correct Answer

A - Periosteum

Image

Explanation Why

Osteosarcomas are caused by malignant osteoblasts that arise from mesenchymal stem cells located
within the periosteum. In patients with osteosarcoma, malignant osteoblast activity results in
increased production of osteoid matrix and formation of poorly organized bone networks.
Osteosarcomas can destroy nearby structures and cause periosteal reactions, which may manifest
with the characteristic Codman triangle and/or sunburst appearance on imaging.

www.eduwaves360.com | Telegram : @eduwaves360


B - Bone marrow

Explanation Why

Malignancies arising from the bone marrow can originate from various hematopoietic cells to form
distinct neoplastic diseases (e.g., acute and chronic leukemia, multiple myeloma). The most common
pediatric malignancy, ALL, can cause insidious bone pain similar to this patient's symptoms.
However, leukemias would not present with the radiographic findings seen in this patient. Moreover,
this boy lacks other common findings of acute leukemia, such as splenomegaly, anemia, hemorrhage,
and/or history of recurrent infections.

C - Cartilage

Image

Explanation Why

Tumors arising from chondroblasts of cartilage tissue are known as enchondromas (benign) and

www.eduwaves360.com | Telegram : @eduwaves360


chondrosarcomas (malignant). Both types of cartilage-derived tumors can present with bone pain and
local swelling. However, they would not present with elevated alkaline phosphatase levels or
periosteal reactions on x-ray. Moreover, chondrosarcomas commonly present with osteolytic lesions
involving the pelvis, spine, or shoulder girdle and mainly occur in patients over the age of 50 years.

D - Epiphyseal plate

Explanation Why

Giant cell tumors of bone are benign, locally aggressive tumors that arise from osteoclastic,
multinucleated giant cells (due to overactivity in the RANK signaling pathway) in the epiphysis of
long bones. Giant cell tumors usually arise around the knee and present with local pain and swelling,
as seen here. However, unlike in this 15-year-old boy, these tumors typically occur in patients older
than 20 years and appear on x-ray as a well-defined osteolytic lesion with a soap-bubble appearance.

E - Neural crest

Explanation Why

Neuroblastoma are malignant, neuroendocrine tumors that originate from neural crest cells and most
commonly present in early childhood. Neuroblastomas can occur anywhere along the sympathetic
tract and can metastasize to the bone, causing localized pain and swelling. Unlike the findings in this
patient, these metastases would appear as lytic lesions on x-ray. Moreover, this patient lacks other
common findings of neuroblastoma, such as abdominal distention and hypertension.

www.eduwaves360.com | Telegram : @eduwaves360


Question # 25

A 31-year-old man comes to the physician because of several months of recurrent abdominal pain and
diarrhea. Six months ago, he traveled to Lake Superior for a fishing trip with his friends, during which
they often ate their day's catch for dinner. Physical examination shows pallor. Laboratory studies show
macrocytic anemia with eosinophilia. A peripheral blood smear shows hypochromic red blood cells with
megaloblasts and hypersegmented neutrophils. A cestode infection is suspected and a drug is prescribed
that kills cestodes by inducing uncontrollable muscle spasm in the parasite. The drug prescribed for this
patient most likely acts by which of the following mechanisms of action?

Answer Image

Blockade of myosin binding


A
sites

Increased calcium influx into the


B
sarcoplasm

Increased sodium efflux from


C
the sarcoplasm

Blockade of L-type calcium


D
channels

Increased potassium efflux from


E
the sarcoplasm

www.eduwaves360.com | Telegram : @eduwaves360


Answer Image

Phosphorylation of adenosine
F
diphosphate

www.eduwaves360.com | Telegram : @eduwaves360


Hint

This patient shows signs of chronic enteritis associated with megaloblastic anemia. A history of eating
freshwater fish suggests a helminthic infection by Diphyllobothrium latum.

www.eduwaves360.com | Telegram : @eduwaves360


Correct Answer

A - Blockade of myosin binding sites

Explanation Why

Blockade of myosin binding sites on actin filaments would prevent the interaction of myosin and
actin, which is essential for cross-bridge formation. Blocking myosin binding sites would inhibit
muscle contraction, rather than causing muscle contraction or spasm.

B - Increased calcium influx into the sarcoplasm

Image

Explanation Why

This patient is most likely treated with the antihelminthic medication praziquantel, which causes
contracted muscle paralysis in parasites by increasing calcium influx into the sarcoplasm of the

www.eduwaves360.com | Telegram : @eduwaves360


parasite. Calcium binds to troponin C and results in a conformational change that exposes
myosin-binding sites on actin filaments. According to the sliding filament theory of muscle
contraction, this enables actin and myosin to form cross-bridges. Contracted muscle paralysis of the
parasites caused by treatment with praziquantel facilitates their destruction by the host immune
system.

C - Increased sodium efflux from the sarcoplasm

Explanation Why

An increased sodium influx into the sarcoplasm (not an increased sodium efflux from the
sarcoplasm) is generated when acetylcholine binds to ligand-gated receptors of the neuromuscular
endplate. This induces an excitatory postsynaptic potential that causes opening of further voltage-
dependent sodium channels, generating an action potential, which eventually leads to muscle
contraction. Increased sodium efflux from the sarcoplasm does not cause muscle contraction or
spasm.

D - Blockade of L-type calcium channels

Explanation Why

Blockade of L-type calcium channels is the mechanism of action of calcium channel blockers, such
as nifedipine and amlodipine. These medications block the flow of calcium into vascular smooth
muscle cells, thereby producing a vasodilatory effect. Therefore, their effect on muscle tissue is
opposite from that of the medication used in this patient. Moreover, these drugs are not used for the
treatment of cestode infection.

E - Increased potassium efflux from the sarcoplasm

Explanation Why

Potassium efflux from the sarcoplasm is part of the physiological process of cell membrane
repolarization. Increased potassium efflux from the sarcoplasm helps to restore the resting membrane
potential in myocytes, which would lead to disruption of muscle contraction, rather than causing

www.eduwaves360.com | Telegram : @eduwaves360


muscle contraction or spasm.

F - Phosphorylation of adenosine diphosphate

Explanation Why

Binding of ATP, which is generated by phosphorylation of adenosine diphosphate (ADP), to myosin


heads leads to detachment of myosin from actin filaments, which disrupts the cross-bridge cycle and
ends muscle contraction. However, hydrolysis of myosin-bound ATP to ADP and Pi alters the
myosin head conformation to a cocked state, which primes the myosin for cross-bridge formation
with actin to generate muscle contraction.

www.eduwaves360.com | Telegram : @eduwaves360


Question # 26

A 54-year-old woman comes to the physician because of lower back pain, night sweats, and a 5-kg
(11-lb) weight loss during the past 4 weeks. She has rheumatoid arthritis treated with adalimumab. Her
temperature is 38°C (100.4°F). Physical examination shows tenderness over the T10 and L1 spinous
processes. Passive extension of the right hip causes pain in the right lower quadrant. The patient's
symptoms are most likely caused by an organism with which of the following virulence factors?

Answer Image

A Polysaccharide capsule that prevents phagocytosis

B Surface glycolipids that prevent phagolysosome fusion

C Polypeptides that inactivate elongation factor 2

D Proteins that bind to the Fc region of immunoglobulin G

E Protease that cleaves immunoglobulin A

www.eduwaves360.com | Telegram : @eduwaves360


Hint

Fever, lower back pain, focal spinal tenderness, and a positive psoas sign are characteristic of vertebral
osteomyelitis. Constitutional symptoms such as weight loss and fever as well as the history of treatment
with a TNF-alpha inhibitor (adalimumab) suggest Mycobacterium tuberculosis infection.

www.eduwaves360.com | Telegram : @eduwaves360


Correct Answer

A - Polysaccharide capsule that prevents phagocytosis

Explanation Why

A polysaccharide capsule that prevents phagocytosis is found in P. aeruginosa and Salmonella spp.,
which can cause osteomyelitis. However, pseudomonal osteomyelitis typically affects intravenous
drug users and Salmonella osteomyelitis typically occurs in patients with sickle cell disease. These
risk factors are not reported in this patient. The presence of constitutional symptoms such as night
sweats and weight loss as well as the history of treatment with a TNF-alpha inhibitor suggest a
different pathogen.

B - Surface glycolipids that prevent phagolysosome fusion

Explanation But

TNF-α plays an important role in the sequestration of M. tuberculosis in granulomas; inhibition of


TNF-α results in the breakdown of granulomas and the release of mycobacteria, causing reactivation
of latent TB. Patients receiving treatment with TNF-alpha inhibitors should be tested for infection
with M. tuberculosis.

Explanation Why

The cord factor is a glycolipid in the cell wall of M. tuberculosis that inhibits phagosome-lysosome
fusion and prevents lysis of phagocytosed mycobacteria. It has a synergistic effect with sulfatides, a
sulfolipid virulence factor in M. tuberculosis that inhibits phagosome-lysosome fusion. This allows
M. tuberculosis to survive in the phagosomes of macrophages. Cord factor also causes virulent M.
tuberculosis bacilli to grow in a serpentine cord-like pattern and promotes the formation of
granulomas by stimulating the release of TNF-alpha.

www.eduwaves360.com | Telegram : @eduwaves360


C - Polypeptides that inactivate elongation factor 2

Explanation Why

The virulence factor exotoxin A, which is produced by Pseudomonas aeruginosa, impairs protein
synthesis by inactivating EEF2. P. aeruginosa is a common cause of osteomyelitis in intravenous
drug users. The presence of constitutional symptoms such as night sweats and weight loss as well as
the history of treatment with a TNF-alpha inhibitor suggest a different pathogen.

D - Proteins that bind to the Fc region of immunoglobulin G

Explanation Why

The virulence factor protein A, which is produced by Staphylococcus aureus, binds to the Fc region
of immunoglobulin G, thereby preventing opsonization and phagocytosis. S. aureus is the most
common cause of osteomyelitis. However, the presence of constitutional symptoms such as night
sweats and weight loss as well as the history of treatment with a TNF-alpha inhibitor suggest a
different pathogen.

E - Protease that cleaves immunoglobulin A

Explanation Why

IgA protease is a virulence factor that cleaves immunoglobulin A, which allows pathogens to adhere
to mucous membranes. It is produced by various bacteria, including S. pneumoniae, H. influenzae,
and Neisseria species. Infection with N. gonorrhoeae can lead to septic arthritis and osteomyelitis of
the spine, causing hip or lower back pain. However, this patient's constitutional symptoms following
treatment with a TNF-alpha inhibitor suggest infection with a different microorganism.

www.eduwaves360.com | Telegram : @eduwaves360


Question # 27

A 3-year-old boy is brought to the emergency department because of nausea and vomiting for 1 day. His
maternal uncle had a seizure disorder and died in childhood. He appears fatigued. Respirations are 32/
min. Examination shows diffuse weakness in the extremities. Serum studies show a low pH, elevated
lactate concentration, and normal blood glucose. A metabolic condition characterized by a defect in
oxidative phosphorylation is suspected. Microscopic examination of a muscle biopsy specimen of this
patient is most likely to show which of the following findings?

Answer Image

Fibrofatty replacement of normal muscle


A
fibers

Muscle atrophy with perimysial


B
inflammation

Intermyofibrillar accumulation of
C
glycogen

Endomysial inflammation with T-cell


D
infiltration

www.eduwaves360.com | Telegram : @eduwaves360


Answer Image

Subsarcolemmal accumulation of
E
mitochondria

www.eduwaves360.com | Telegram : @eduwaves360


Hint

This patient's presentation of metabolic acidosis (manifesting as tachypnea), elevated serum lactate, and
diffuse muscle weakness is most likely due to lactic acidosis.

www.eduwaves360.com | Telegram : @eduwaves360


Correct Answer

A - Fibrofatty replacement of normal muscle fibers

Image

Explanation Why

Fibrofatty replacement of normal muscle fibers can be seen in Duchenne muscular dystrophy and
Becker muscular dystrophy. These X-linked disorders most commonly occur in boys 2–5 years of
age but are caused by a defect in the dystrophin gene, not in oxidative phosphorylation. Early
manifestations typically include paresis and atrophy of the proximal lower limbs. Lactic acidosis is
not associated with muscular dystrophies.

www.eduwaves360.com | Telegram : @eduwaves360


B - Muscle atrophy with perimysial inflammation

Image

Explanation Why

Muscle atrophy with perimysial inflammation is seen in dermatomyositis, which results from a
CD4+ T cell-mediated immunologic response, not from defective oxidative phosphorylation.
Dermatomyositis manifests with muscle weakness, as in this patient, but is typically limited to
proximal muscles (most commonly the pelvic and shoulder girdle muscles). Serum studies show
elevated inflammatory markers (e.g., ESR) and muscle enzymes (creatinine kinase), but lactic
acidosis is not associated with the condition. Moreover, this patient does not have characteristic
cutaneous manifestations such as heliotrope rash and/or Gottron papules.

www.eduwaves360.com | Telegram : @eduwaves360


C - Intermyofibrillar accumulation of glycogen

Explanation Why

Intermyofibrillar accumulation of glycogen is characteristic of McArdle disease, a disorder in which


the muscle is unable to break down glycogen. It commonly manifests in adolescence or early
adulthood with muscle cramps, weakness, and myoglobinuria following exercise. Due to the lack of
myophosphorylase, affected patients show a flat increase in lactate level during exercise testing,
unlike the elevated lactate levels in this patient. Oxidative phosphorylation is unaffected by McArdle
disease.

D - Endomysial inflammation with T-cell infiltration

Explanation Why

Endomysial inflammation with CD8+ T-cell infiltration is a typical finding in polymyositis (PM),
which is presumably caused by an autoimmune reaction against skeletal muscle antigens, not by
defective oxidative phosphorylation. PM presents with muscle weakness, as in this patient, but is
typically limited to proximal muscles (most commonly the pelvic and shoulder girdle muscles).
Serum studies show elevated inflammatory markers and muscle enzymes, but lactate acidosis is not
associated with PM.

E - Subsarcolemmal accumulation of mitochondria

Explanation Why

Subsarcolemmal and intermyofibrillar accumulation of mitochondria in muscle fibers is a hallmark


of mitochondrial diseases, which are characterized by defective oxidative phosphorylation and
subsequent lack of energy. The compensatory proliferation of mitochondria presents as characteristic
ragged-red fibers on Gomori trichrome stain. This patient's lactic acidosis and diffuse muscle
weakness in combination with the family history of CNS disease and childhood death suggest a form
of mitochondrial myopathy, most likely MELAS syndrome.

www.eduwaves360.com | Telegram : @eduwaves360


Question # 28

A 72-year-old woman is brought to the emergency department because of severe pain in her left hip after
a fall this morning. She has smoked one pack of cigarettes daily for 45 years. Her only medication is a
vitamin D supplement. Physical examination shows that her left leg is externally rotated and appears
shorter than her right leg. An x-ray of the pelvis shows a fracture of the neck of the left femur. Which of
the following changes in bone architecture is the most likely underlying cause of this patient's
symptoms?

Answer Image

A Overgrowth of cortical bone and reduced marrow space

B Loss of cortical bone mass and thinning of trabeculae

C Subperiosteal bone resorption and cystic degeneration

D Accumulation of inadequately mineralized osteoid

E Formation of multiple sclerotic lesions in bony cortex

F Deposition of lamellar bone interspersed with woven bone

www.eduwaves360.com | Telegram : @eduwaves360


Hint

The most likely cause of femoral neck fracture in a postmenopausal woman with a history of smoking is
osteoporosis.

www.eduwaves360.com | Telegram : @eduwaves360


Correct Answer

A - Overgrowth of cortical bone and reduced marrow space

Explanation Why

Overgrowth of cortical bone and reduced marrow space are found in osteopetrosis, which is caused
by defective osteoclast activity. This results in thickened cortical bone that nevertheless breaks easily.
Typical clinical manifestations are cranial nerve palsies, and pancytopenia, none of which is seen in
this patient. Furthermore, this patient's significant risk for osteoporosis indicates that she has a
different change in bone architecture.

B - Loss of cortical bone mass and thinning of trabeculae

Explanation But

Postmenopausal women are especially prone to osteoporotic fractures due to increased bone
resorption. This is caused by decreased estrogen levels, as estrogen stimulates osteoblasts and
inhibits osteoclasts.

Explanation Why

Loss of cortical bone mass and thinning of trabecular bone are the microscopic hallmarks of
osteoporosis. This patient has numerous risk factors for osteoporosis including female gender,
advanced age, postmenopausal state, and tobacco use. In patients with osteoporosis, relatively minor
trauma can cause fractures.

C - Subperiosteal bone resorption and cystic degeneration

Explanation Why

Subperiosteal bone resorption and cystic degeneration are seen in hyperparathyroidism.


Pathologically elevated parathyroid hormone levels inappropriately stimulate osteoclasts, causing

www.eduwaves360.com | Telegram : @eduwaves360


bone catabolism, which can lead to fractures. However, hyperparathyroidism would be unlikely to
present with an isolated fracture and no other manifestations such as nephrolithiasis, nausea, or
constipation. Furthermore, this patient's significant risk for osteoporosis indicates that she has a
different change in bone architecture.

D - Accumulation of inadequately mineralized osteoid

Explanation Why

Accumulation of inadequately mineralized osteoid is the pathophysiology of osteomalacia (in adults)


and rickets (in children). In an adult patient, osteomalacia would most likely result from a vitamin D
deficiency due to malnutrition, malabsorption, or chronic kidney disease. This patient, however,
takes a vitamin D supplement, making a vitamin D deficiency unlikely.

E - Formation of multiple sclerotic lesions in bony cortex

Explanation Why

Sclerotic bony lesions may occur due to bone metastases, vascular disease, or infection, and can
predispose patients to femur fractures. However, this patient's significant risk for osteoporosis
indicates that she has a different change in bone architecture.

F - Deposition of lamellar bone interspersed with woven bone

Explanation Why

Deposition of lamellar bone interspersed with woven bone creates a 'mosaic' pattern that is classic for
Paget disease of bone (PDB). PDB can present with pain and deformities in the affected bone but
most commonly involves the skull (e.g., increased hat size, impaired hearing). Although Paget
disease of bone can predispose patients to femur fractures, this patient's significant risk for
osteoporosis indicates that she has a different change in bone architecture.

www.eduwaves360.com | Telegram : @eduwaves360


Question # 29

A previously healthy 5-year-old boy is brought to the emergency department because of fever,
irritability, malaise, and left knee pain for 4 days. Four days ago, he fell off his bike and scraped his
elbow. His temperature is 39.1°C (102.4°F). The patient walks with a limp. Examination shows swelling
and point tenderness over the medial aspect of the left knee. An MRI of the left knee shows edema of
the bone marrow and destruction of the medial metaphysis of the tibia. Which of the following is the
most likely causal organism?

Answer Image

A Staphylococcus epidermidis

B Brucella melitensis

C Staphylococcus aureus

D Pseudomonas aeruginosa

E Pasteurella multocida

F Salmonella enterica

G Streptococcus pyogenes

www.eduwaves360.com | Telegram : @eduwaves360


Hint

This patient has osteomyelitis of the lower extremity. Since there is no evidence of direct inoculation
(e.g., trauma to the knee) or contiguous spread (e.g., from a foot ulcer), the most likely route of infection
is via hematogenous spread. The most common causal pathogen of hematogenous osteomyelitis in
children has several virulence factors, including protein A.

www.eduwaves360.com | Telegram : @eduwaves360


Correct Answer

A - Staphylococcus epidermidis

Explanation Why

Staphylococcus epidermidis can cause osteomyelitis in patients with diabetic ulcers of the feet and in
patients with prosthetic implants or indwelling vascular catheters. It is a rare cause of osteomyelitis
in children who are otherwise healthy.

B - Brucella melitensis

Explanation Why

Brucella melitensis can cause osteomyelitis in children who live in or travel to areas where
consumption of unpasteurized dairy products is common. It is a rare cause of osteomyelitis in
children who live in developed countries.

C - Staphylococcus aureus

Explanation But

If feasible, bone biopsy or aspiration should be performed before administering antibiotics. However,
empiric antibiotic therapy (e.g., IV nafcillin or vancomycin) should not be delayed in this patient, as
osteomyelitis can have detrimental effects on bone development that result in severe, long-term
complications.

Explanation Why

Staphylococcus aureus accounts for the majority of acute osteomyelitis cases in both children and
adults. Hematogenous osteomyelitis occurs more frequently in children than adults; > 50% of
patients are 5 years of age or younger. In children, the metaphysis of long bones is most often
affected, whereas in adults, the vertebrae are the most common site. Pediatric risk factors include

www.eduwaves360.com | Telegram : @eduwaves360


sickle cell disease, immunodeficiency disorders, sepsis, indwelling vascular catheters, and minor
trauma.

D - Pseudomonas aeruginosa

Explanation Why

Pseudomonas aeruginosa is an uncommon cause of hematogenous osteomyelitis and is associated


with IV drug use and nail puncture wounds to the foot. Since this patient has no history of such an
injury, another causal pathogen is more likely.

E - Pasteurella multocida

Explanation Why

Pasteurella multocida is an extremely rare cause of osteomyelitis that can be caused by direct
inoculation from cat or dog bites. Since this patient has no history of such an injury, another causal
pathogen is more likely.

F - Salmonella enterica

Explanation Why

Salmonella enterica is the most common cause of hematogenous osteomyelitis in patients with sickle
cell disease in developed countries, but it is an exceedingly rare cause of osteomyelitis in children
who are otherwise healthy. Since there is no evidence of sickle cell disease in this patient, another
causal pathogen is more likely.

www.eduwaves360.com | Telegram : @eduwaves360


G - Streptococcus pyogenes

Explanation Why

Streptococcus pyogenes causes up to 10% of cases of hematogenous osteomyelitis in children;


however, it is not the most common causal pathogen. Osteomyelitis from Streptococcus pyogenes
can occur as a complication of varicella-zoster virus infection.

www.eduwaves360.com | Telegram : @eduwaves360


Question # 30

A 28-year-old male comes to the physician for worsening back pain. The pain began 10 months ago, is
worse in the morning, and improves with activity. He has also had bilateral hip pain and difficulty
bending forward during exercise for the past 3 months. He has celiac disease and eats a gluten-free diet.
Examination shows a limited range of spinal flexion. Flexion, abduction, and external rotation of both
hips produces pain. Further evaluation of this patient is most likely to show which of the following
laboratory findings?

Answer Image

High levels of creatine


A
phosphokinase

Presence of anti-dsDNA
B
antibodies

Presence of anti-Ro and anti-La


C
antibodies

High levels of rheumatoid


D
factor

E HLA-B27 positive genotype

www.eduwaves360.com | Telegram : @eduwaves360


Answer Image

F HLA-DR3-positive genotype

www.eduwaves360.com | Telegram : @eduwaves360


Hint

This patient's history of progressively worsening back pain that is worse in the morning, impaired spinal
flexion, and bilateral hip pain are consistent with ankylosing spondylitis.

www.eduwaves360.com | Telegram : @eduwaves360


Correct Answer

A - High levels of creatine phosphokinase

Explanation Why

Creatine phosphokinase is elevated following brain or muscle damage (e.g., vigorous exercise,
myositis, rhabdomyolysis). Lumbar muscle damage is a common cause of acute lower back pain, but
unlike our patient, symptoms usually worsen with muscle use, and pain resolves within 2 months for
90% of patients.

B - Presence of anti-dsDNA antibodies

Explanation Why

Anti-dsDNA antibodies are diagnostic of systemic lupus erythematosus (SLE). Although this patient
's history of celiac disease (an autoimmune disease) and joint pain are associated with SLE, he has no
other evidence of the disease (e.g., malar rash, photosensitivity).

C - Presence of anti-Ro and anti-La antibodies

Explanation Why

The presence of both anti-Ro and anti-La antibodies is suggestive of either Sjogren's syndrome or
systemic lupus erythematosus (SLE), for which this patient is at increased risk (history of Celiac
disease). While both diseases frequently cause joint pain, this patient lacks associated symptoms
(e.g., dry eye/mouth for Sjogren's syndrome or rash for SLE).

www.eduwaves360.com | Telegram : @eduwaves360


D - High levels of rheumatoid factor

Explanation Why

Although rheumatoid factor is a nonspecific finding, it is classically associated with rheumatoid


arthritis (RA) and absent in seronegative spondyloarthropathies. Unlike this patient, RA typically
causes a symmetric polyarthritis (usually the PIP and MCP joints of the hands) in women of
childbearing age and rarely affects the thoracic or lumbar spine.

E - HLA-B27 positive genotype

Image

Explanation But

Although most patients with ankylosing spondylitis are HLA-B27 positive, fewer than 5% of people
who are HLA-B27 positive develop the disease. Isolated HLA-B27 positivity has little clinical
significance. Gene positivity only supports the diagnosis in patients with clinical suspicion of an

www.eduwaves360.com | Telegram : @eduwaves360


associated disease (e.g., chronic lower back pain and morning stiffness).

Explanation Why

90–95% of patients with ankylosing spondylitis are HLA-B27 positive, compared to 6% of the
general population. HLA-B27 testing is indicated to diagnose ankylosing spondylitis in patients with
an inconclusive clinical presentation and pelvic x-ray.

F - HLA-DR3-positive genotype

Explanation Why

HLA-DR3 is positive in some autoimmune diseases, including systemic lupus erythematosus (SLE).
Although this patient 's history of celiac disease (an autoimmune disease) and joint pain are
associated with SLE, this patient has no other evidence of the disease (e.g., malar rash,
photosensitivity).

www.eduwaves360.com | Telegram : @eduwaves360


Question # 31

A 42-year-old woman comes to the physician because of a 2-month history of progressive muscular
weakness. She has had difficulty climbing stairs, getting up from chairs, and brushing her hair. Her vital
signs are within normal limits. Muscle strength is 2/5 with flexion of the hips and 3/5 with abduction of
the shoulders. She is unable to stand up from her chair without the use of her arms for support.
Laboratory studies show elevations in leukocyte count, erythrocyte sedimentation rate, and creatine
kinase concentration. Histological evaluation of a biopsy specimen of the deltoid muscle is most likely
to show which of the following?

Answer Image

Muscle fiber necrosis with rare


A
inflammatory cells

Sarcolemmal MHC-I overexpression


B
with CD8+ lymphocytic infiltration

Sarcoplasmic rimmed vacuoles with


C
CD8+ lymphocytic infiltration

www.eduwaves360.com | Telegram : @eduwaves360


Answer Image

Relative atrophy of type II muscle


D fibers with hypertrophy of type I
muscle fiber

Perimysial inflammation with


E perivascular CD4+ T lymphocytic
infiltration

www.eduwaves360.com | Telegram : @eduwaves360


Hint

This patient has symmetric proximal muscle weakness with elevated muscle and inflammatory markers
without dermatologic findings, which is consistent with a diagnosis of polymyositis.

www.eduwaves360.com | Telegram : @eduwaves360


Correct Answer

A - Muscle fiber necrosis with rare inflammatory cells

Explanation But

Unlike statin-induced myopathy, NAM does not improve after discontinuing the medication.

Explanation Why

Necrotic muscle fibers with only rare inflammatory cells would be consistent with necrotizing
autoimmune myopathy (NAM), a rare disease that is associated with paraneoplastic disorders,
connective tissue disorders, and statin use. It can also manifest with proximal muscle weakness and
elevated creatine kinase, but this patient doesn't have any risk factors for NAM and a different
diagnosis is more likely.

www.eduwaves360.com | Telegram : @eduwaves360


B - Sarcolemmal MHC-I overexpression with CD8+ lymphocytic infiltration

Image

Explanation Why

Histological examination of skeletal muscles affected by polymyositis typically shows endomysial


infiltration of CD8+ lymphocytic T cells and macrophages, with areas of muscle necrosis and
regeneration of muscle fibers. In addition, overexpression of MHC-I on the sarcolemma is an
immunohistochemical finding that is classic for polymyositis. Muscle atrophy is not a prominent
feature of this disorder, in contrast to dermatomyositis. Polymyositis is associated with positive ANA
(30–60% of cases), positive anti-Jo-1 (25% of cases), and malignancy (15–20% of cases).

www.eduwaves360.com | Telegram : @eduwaves360


C - Sarcoplasmic rimmed vacuoles with CD8+ lymphocytic infiltration

Image

Explanation Why

A skeletal muscle biopsy showing sarcoplasmic rimmed vacuoles and infiltration of CD8+ T
lymphocytes and dendritic cells would be consistent with inclusion body myositis (IBM), which is
also characterized by necrosis and atrophy of muscle fibers. Although IBM also manifests with
proximal muscle weakness, the weakness is usually asymmetric and progresses over years, rather
than months. In addition, the condition is most common in older age (> 50 years old) and is more
common in men.

www.eduwaves360.com | Telegram : @eduwaves360


D-

Relative atrophy of type II muscle fibers with hypertrophy of type I muscle


fiber

Explanation Why

Hypothyroid myopathy is characterized by noninflammatory type II muscle fiber atrophy with


relative type I muscle fiber hypertrophy. This condition may also manifest with proximal muscle
weakness and elevated creatine kinase, as seen in this patient. However, patients with hypothyroid
myopathy typically have other symptoms of hypothyroidism (e.g., fatigue, weight gain, constipation,
and delayed deep tendon reflex), which are not seen in this patient.

E-

Perimysial inflammation with perivascular CD4+ T lymphocytic infiltration

Image

www.eduwaves360.com | Telegram : @eduwaves360


Explanation Why

Perimysial inflammation with perivascular infiltration of CD4+ T lymphocytes would be consistent


with dermatomyositis, which is also characterized by capillary damage and muscle atrophy. It also
manifests with progressive symmetrical proximal muscle weakness but is also defined by skin
involvement (e.g., Gottron's papules and heliotrope rash), which is lacking in this patient.

www.eduwaves360.com | Telegram : @eduwaves360


Question # 32

A 65-year-old woman is brought to the emergency department because of left wrist pain and swelling
that began after she fell from a seated position. Menopause occurred 15 years ago. Her serum
parathyroid hormone level is within normal limits. An x-ray of the left wrist shows a nondisplaced
fracture of the distal radial metaphysis and decreased bone mineral density. The patient would likely
benefit from an agent with a structure analogous to which of the following substances?

Answer Image

A Inositol

B Keratan sulfate

C Hydroxyapatite

D Nitric oxide

E Pyrophosphate

www.eduwaves360.com | Telegram : @eduwaves360


Hint

A postmenopausal woman presenting with a fracture on a background of decreased bone mineral density
suggests osteoporosis. Bisphosphonates are indicated.

www.eduwaves360.com | Telegram : @eduwaves360


Correct Answer

A - Inositol

Explanation Why

Inositol is a supplement given to women in the treatment of polycystic ovary syndrome (PCOS).
Studies have shown that inositol supplementation can increase insulin sensitivity and indirectly
improve bone density through the management of metabolic syndrome. However, inositol is not
structurally analogous to bisphosphonates.

B - Keratan sulfate

Explanation Why

Proteoglycans (PGs), such as keratan sulfate, are macromolecules consisting of a protein core with
50 to 100 unbranched glycosaminoglycans. Keratan sulfate forms part of the bone extracellular
matrix and functions to support and organize bone tissue. However, PGs are not structurally
analogous to bisphosphonates.

C - Hydroxyapatite

Explanation Why

Hydroxyapatite crystals are the inorganic component of the bone matrix and are sites for
bisphosphonate binding. Hydroxyapatite crystals coated with bisphosphonates are absorbed by
osteoclasts, whereby the bisphosphonates impair metabolism in the cells, inducing apoptosis and
decreasing bone resorption. However, hydroxyapatite crystals are not structurally analogous to
bisphosphonates.

www.eduwaves360.com | Telegram : @eduwaves360


D - Nitric oxide

Explanation Why

Nitric oxide (NO) is the metabolite of several anti-anginal and antihypertensive drugs and acts as a
vasodilator in the cardiovascular system. While endogenous NO acts as a local hormonal regulator of
bone homeostasis, the effects of orally-taken NO precursors (e.g., nitroglycerin) on bone metabolism
have not been established. NO compounds are not structurally analogous to bisphosphonates.

E - Pyrophosphate

Explanation Why

Inorganic pyrophosphates are compounds formed by the linkage of two phosphate ions and are found
endogenously in all cells. Bisphosphonates are structurally analogous to inorganic pyrophosphates
and attach to binding sites on hydroxyapatite, where pyrophosphate would normally bind. When
these bisphosphonate-coated hydroxyapatite crystals are taken up by osteoclasts during bone
resorption, the bisphosphonate interferes with osteoclast functions, thereby reducing bone resorption.
Bisphosphonates also reduce osteoclast activity by inducing apoptosis.

www.eduwaves360.com | Telegram : @eduwaves360


Question # 33

A 3-year-old boy is brought to the physician because of a 3-day history of fever and chills. The mother
reports that he has also been limping for 2 days. He has no history of trauma to this region. His
temperature is 38.9°C (102°F). Physical examination shows dull tenderness over his left lower
extremity. The range of motion of the leg is also limited by pain. While walking, he avoids putting
weight on his left leg. Laboratory studies show an erythrocyte sedimentation rate of 67 mm/h. An MRI
is most likely to show abnormalities in which of the following regions?

Answer Image

A Lumbar vertebral body

B Proximal metaphysis of the femur

C Proximal epiphysis of the femur

D Diaphysis of the tibia

E Acetabulum of the ilium

www.eduwaves360.com | Telegram : @eduwaves360


Hint

Fever, chills, limping, local tenderness, and the elevated ESR in this young boy raise concerns for
osteomyelitis. Acute osteomyelitis in children is usually caused by hematogenous spread; a particular
region of the bone is especially susceptible to the deposition of bacteria.

www.eduwaves360.com | Telegram : @eduwaves360


Correct Answer

A - Lumbar vertebral body

Explanation Why

Lumbar vertebral body can be affected by hematogenous osteomyelitis. However, pain would be
localized in the spine, not in the leg. Hematogenous osteomyelitis would also be more commonly
found in adults than in children. This is due to the fact that, as aging occurs, metaphysis metabolism
slows down, blood flow decreases, and phagocytic activity increases. The vertebrae become more
vascular with maturation, which increases the possibility of bacteria seeding to vertebral bodies.

B - Proximal metaphysis of the femur

Explanation Why

Hematogenous osteomyelitis is most common in children and adolescents (> 50% of cases occur in
children ≤ 5 years). The metaphyseal blood supply is unique, in that the nutrient artery supplying the
metaphysis makes a sharp loop before it empties into sinusoidal spaces, where the rate of blood flow
is decreased. This creates a medium for bacteria to proliferate. The focus of infection in the
metaphysis leads to cellulitis in the bone marrow. Inflammation in the marrow causes increased
intramedullary pressure, which forces the exudate into the cortex, where it can rupture through the
periosteum. More than 80% of cases of hematogenous osteomyelitis occur in the long bones, the
femur and tibia being the most common, and cases usually originate in the metaphyses.

C - Proximal epiphysis of the femur

Explanation Why

The proximal epiphysis of the femur is often affected in slipped capital femoral epiphysis, which
presents with pain, limping, and a restricted range of motion. However, the pain would be located in
the groin or hip area, and patients might hold their hip in a passive external rotation. Furthermore,
fever, chills, and an elevated ESR would be highly unlikely. The epiphysis derives its blood supply
from peri-articular vascular anastomoses, which enter the epiphysis through vascular foramina. After

www.eduwaves360.com | Telegram : @eduwaves360


18 months of age, the number and size of these foramina decrease, thereby reducing the chance of
hematogenous seeding.

D - Diaphysis of the tibia

Explanation Why

The tibial diaphysis is often affected in cancerous diseases such as Ewing sarcoma, which presents
with localized pain, fever, chills (B symptoms), and an elevated ESR. However, in Ewing sarcoma,
the pain would worsen at night, and peak incidence would be between 10–15 years of age. The
diaphysis derives its blood supply from the overlying periosteum via a high-flow periosteal capillary
network. As a result, pathogens less commonly lodge in the diaphysis.

E - Acetabulum of the ilium

Explanation Why

The acetabulum of the ilium can be affected by Paget disease, which is characterized by an increased
rate of bone remodeling and presents with pain and an elevated ESR. However, Paget disease occurs
more commonly in adults > 55 years of age than in children, and fever or chills would not be typical.
The acetabulum, like most flat bones, is not commonly affected by osteomyelitis. This is because the
articular surface of the acetabulum is made of pure hyaline cartilage and has no blood vessels,
resulting in immunity to hematogenous seeding.

www.eduwaves360.com | Telegram : @eduwaves360


Question # 34

A 25-year-old man comes to the physician because of right wrist pain after a fall from a ladder. Physical
examination shows decreased grip strength and tenderness between the tendons of extensor pollicis
longus and extensor pollicis brevis. X-ray of the right wrist shows no abnormalities. This patient is at
increased risk for which of the following complications?

Answer Image

Paralysis of the abductor


A
pollicis brevis muscle

Osteoarthritis of the
B
radiocarpal joint

Avascular necrosis of the


C
lunate bone

Hypesthesia of the
D
hypothenar eminence

www.eduwaves360.com | Telegram : @eduwaves360


Answer Image

Contracture of the palmar


E
aponeurosis

www.eduwaves360.com | Telegram : @eduwaves360


Hint

The region between the tendons of extensor pollicis longus and extensor pollicis brevis is called the
anatomical snuff box.

www.eduwaves360.com | Telegram : @eduwaves360


Correct Answer

A - Paralysis of the abductor pollicis brevis muscle

Explanation Why

Paralysis of the abductor pollicis brevis is a feature of median nerve palsy, which can occur
following a fall on the hand due to compression of the median nerve within the carpal tunnel by a
dislocated lunate. However, lunate dislocation would cause swelling in the wrist with restriction of
the range of motion, and the dislocated lunate would be clearly visible on x-ray. This patient with
pain in the anatomical snuffbox and a normal initial x-ray most likely has a different injury.

B - Osteoarthritis of the radiocarpal joint

Image

www.eduwaves360.com | Telegram : @eduwaves360


Explanation Why

Decreased grip strength with pain in the anatomical snuffbox suggests a scaphoid fracture, which is
typically caused by falling onto an outstretched hand. The scaphoid bone, which is one of the carpal
bones that forms the radiocarpal joint, receives its blood supply from an end artery that runs in a
distal to proximal direction. A fracture of scaphoid can damage this artery and cause avascular
necrosis of the proximal segment. Avascular necrosis of the scaphoid increases the risk of
osteoarthritis of the radiocarpal joint. The initial x-ray in the case of scaphoid fracture can be normal.
Therefore, patients with a normal x-ray should be followed up with a repeat x-ray after 2 weeks or an
MRI if a scaphoid fracture is strongly suspected.

C - Avascular necrosis of the lunate bone

Explanation Why

Avascular necrosis of the lunate is seen in Kienbock disease, a condition that is associated with
repetitive impact to the hand (e.g., playing volleyball). This patient with pain in the anatomic snuff
box after acute trauma is most likely to develop avascular necrosis of the scaphoid, not the lunate.

www.eduwaves360.com | Telegram : @eduwaves360


D - Hypesthesia of the hypothenar eminence

Image

Explanation Why

Hypesthesia of the hypothenar eminence is a feature of ulnar nerve palsy, which can occur following
a fall on the hand due to compression of the ulnar nerve by a fractured hook of hamate. Hook of
hamate fractures can cause decreased grip strength, and the initial wrist x-ray can be normal in
anteroposterior and lateral views. However, patients with a hook of hamate fracture would have
tenderness on the palmar surface of the hand at the site of the hamate. This patient with pain in the
anatomical snuffbox (dorsal surface of the hand) most likely has a different injury.

E - Contracture of the palmar aponeurosis

Explanation Why

Risk factors for contraction of the palmar aponeurosis (Dupuytren contracture) are repetitive trauma,

www.eduwaves360.com | Telegram : @eduwaves360


cigarette smoking, alcohol consumption, liver cirrhosis, diabetes mellitus, and autoimmune diseases.
This patient's injury is not a risk factor for Dupuytren contracture.

www.eduwaves360.com | Telegram : @eduwaves360


Question # 35

A 52-year-old woman with hypertension and type 2 diabetes mellitus comes to the physician because of
a 1-day history of severe pain and swelling of her left great toe. She has had similar episodes
sporadically over the past 3 years. She drinks 6 beers daily. She does not smoke or use illicit drugs. She
is allergic to hydrochlorothiazide and glipizide. Her current medications are amlodipine and metformin.
Examination shows erythema, warmth, and tenderness of the left first metatarsophalangeal joint and a
nodule over the right elbow. The most appropriate next step in treatment is the administration of a drug
that has which of the following mechanisms of action?

Answer Image

Reversible inhibition of
A
dihydrofolate reductase

Reversible inhibition of
B
prostaglandin synthesis

Irreversible inhibition of
C
cyclooxygenase

Selective inhibition of
D
cyclooxygenase-2

Increased renal excretion of


E
uric acid

www.eduwaves360.com | Telegram : @eduwaves360


Answer Image

Irreversible inhibition of
F
xanthine oxidase

www.eduwaves360.com | Telegram : @eduwaves360


Hint

Arthritis of the metatarsophalangeal joint in a patient with heavy alcohol use is diagnostic for acute gout.
Patients with gout can develop nodules over the olecranon process (tophi).

www.eduwaves360.com | Telegram : @eduwaves360


Correct Answer

A - Reversible inhibition of dihydrofolate reductase

Explanation Why

Methotrexate, which reversibly inhibits dihydrofolate reductase, is used as a DMARD to treat


rheumatoid arthritis (RA). Although RA can present in middle-aged women with joint pain and a
nodule over the olecranon process (rheumatoid nodule), it typically results in symmetrical arthritis
that predominantly affects the wrists, MCP and PIP joints and is associated with early morning
stiffness. Even if this patient had RA, methotrexate would not be the first-line drug during an acute
flare.

B - Reversible inhibition of prostaglandin synthesis

Image

www.eduwaves360.com | Telegram : @eduwaves360


Explanation Why

Acute gout is the most likely diagnosis in this patient presenting with podagra and tophi. NSAIDs,
such as indomethacin or ibuprofen, which reversibly inhibit prostaglandin synthesis by inhibiting
both COX-1 and COX-2, are the first line in the management of an acute gout flare. Other drugs that
can be used in a severe flare of gout include glucocorticoids and colchicine. Xanthine oxidase
inhibitors (e.g., allopurinol, febuxostat) can be used to prevent flares of gout but should not be used
within the first 2 weeks of an acute flare because they can worsen the acute gout attack.

C - Irreversible inhibition of cyclooxygenase

Explanation Why

Aspirin, an NSAID that irreversibly inhibits cyclooxygenase (COX), is avoided in gout as it inhibits
uric acid secretion at low doses (1–2 g/day).

D - Selective inhibition of cyclooxygenase-2

Explanation Why

Cyclooxygenase-2 (COX-2) inhibitors (e.g., celecoxib) are potent anti-inflammatory drugs that can
be used to treat an acute flare of gout. However, selective COX-2 inhibitors are sulfa drugs and
should, therefore, be avoided in this patient who is allergic to sulfa drugs (e.g., hydrochlorothiazide,
glipizide).

E - Increased renal excretion of uric acid

Explanation Why

Uricosuric drugs like probenecid reduce serum urate by increasing renal elimination of uric acid.
They are indicated in the management of chronic hyperuricemia and are not useful in the setting of
an acute gout flare.

www.eduwaves360.com | Telegram : @eduwaves360


F - Irreversible inhibition of xanthine oxidase

Explanation Why

Allopurinol and febuxostat are both drugs that act by irreversible inhibition of xanthine oxidase.
They reduce uric acid levels over time and are, therefore, used in the management of chronic gout.
However, they may precipitate flares of acute gout by increasing urate crystal mobilization and
should be avoided during the initial stage of acute inflammation.

www.eduwaves360.com | Telegram : @eduwaves360


Question # 36

A 56-year-old man comes to the physician because of a 1-day history of sudden severe pain in his right
great toe. Four months ago, he had an episode of severe joint pain involving his left knee that lasted
several days and resolved with over-the-counter analgesics. He has a history of nephrolithiasis and
hypertension treated with hydrochlorothiazide. Examination shows erythema, swelling, warmth, and
tenderness of the right metatarsophalangeal joint; range of movement is limited by pain. His serum uric
acid is 12 mg/dL. Arthrocentesis yields cloudy fluid with a leukocyte count of 18,500/mm3 (80%
segmented neutrophils). Polarized light microscopy of the synovial fluid is shown. Which of the
following is the mechanism of action of the most appropriate long-term pharmacotherapy for this
patient's condition?

Answer Image

A Increased conversion of uric acid to allantoin

www.eduwaves360.com | Telegram : @eduwaves360


Answer Image

B Inhibition of phospholipase A

C Inhibition of xanthine oxidase

D Increased renal excretion of urate

E Inhibition of cyclooxygenase

www.eduwaves360.com | Telegram : @eduwaves360


Hint

This patient presents with a typical history of acute gout and evidence of hyperuricemia (i.e., serum uric
acid levels > 7 mg/dL), most likely secondary to thiazide diuretic use and high protein intake (protein is
broken down to purines, and eventually into uric acid). The first-line agent for long-term therapy of gout
decreases uric acid production.

www.eduwaves360.com | Telegram : @eduwaves360


Correct Answer

A - Increased conversion of uric acid to allantoin

Explanation Why

Increased conversion of uric acid to allantoin is the mechanism of action of recombinant uricase
(e.g., rasburicase, pegloticase), which is a third-line medication for long-term pharmacotherapy of
gout. In a patient who is not currently undergoing any medical therapy for chronic gout, a
recombinant uricase should not be the first choice.

B - Inhibition of phospholipase A

Image

Explanation Why

Inhibition of phospholipase A is the mechanism of action of glucocorticoids, which are indicated for

www.eduwaves360.com | Telegram : @eduwaves360


acute gout attacks in patients who fail to respond or have contraindications to NSAIDs and
colchicine. Glucocorticoids are not an appropriate long-term pharmacotherapy for gout.

C - Inhibition of xanthine oxidase

Image

Explanation But

This patient takes hydrochlorothiazide, which is associated with increased serum uric acid levels.
Switching to a different diuretic medication should be considered.

Explanation Why

Inhibition of xanthine oxidase is the mechanism of action of allopurinol, which is indicated for the
treatment of chronic gout and prophylaxis of acute gout attacks. Indications for long-term
pharmacotherapy include recurrent gout flares (≥ 2 gout attacks per year) and evidence of tophi
development. Urate-lowering therapy should also be considered in patients with < 2 episodes of
acute gout who have hyperuricemia (> 9 mg/dL), advanced chronic kidney disease, or a history of
urolithiasis, which qualifies this patient for allopurinol treatment. However, allopurinol should not be
initiated within 2 weeks of resolution of the acute gout flare because of the risk of urate crystal
mobilization, which can worsen symptoms. When treatment with allopurinol is started,

www.eduwaves360.com | Telegram : @eduwaves360


antiinflammatory prophylaxis (colchicine, NSAIDs, or glucocorticoids) should be given as
prophylaxis against acute gout attacks.

D - Increased renal excretion of urate

Explanation Why

Increased renal excretion of urate is the mechanism of action of uricosuric medications


(benzbromarone, probenecid, lesinurad), which are sometimes used as an alternative treatment or in
combination with allopurinol. However, this patient has a history of nephrolithiasis, in which case
uricosuric agents are contraindicated because they increase urine uric acid concentration and can
precipitate uric acid stones.

E - Inhibition of cyclooxygenase

Image

www.eduwaves360.com | Telegram : @eduwaves360


Explanation Why

Inhibition of cyclooxygenase is the mechanism of action of NSAIDs and aspirin. NSAIDs are the
first-line treatment for acute gout attacks and should be discontinued 2–3 days after the symptoms
resolve; they are not indicated for long-term pharmacotherapy of gout. Aspirin is contraindicated in
acute gout attacks because it can elevate uric acid levels and thus delay symptom resolution.

www.eduwaves360.com | Telegram : @eduwaves360


Question # 37

A 54-year-old woman comes to the physician because of a 3-month history of upper midthoracic back
pain. The pain is severe, dull in quality, and worse during the night. Ten months ago, she underwent a
modified radical mastectomy for invasive ductal carcinoma of the right breast. Physical examination
shows normal muscle strength. Deep tendon reflexes are 2+ in all extremities. Examination of the back
shows tenderness over the thoracic spinous processes. An x-ray of the thoracic spine shows vertebral
osteolytic lesions at the levels of T4 and T5. The patient's thoracic lesions are most likely a result of
metastatic spread via which of the following structures?

Answer Image

Anterior mediastinal lymph


A
nodes

B Lateral axillary lymph nodes

C Azygos vein

www.eduwaves360.com | Telegram : @eduwaves360


Answer Image

D Thyrocervical trunk

E Thoracic duct

F Intercostal artery

www.eduwaves360.com | Telegram : @eduwaves360


Hint

This patient has metastatic lesions in the vertebrae due to spread of her breast cancer to a structure that
communicates with the paravertebral plexus of Batson.

www.eduwaves360.com | Telegram : @eduwaves360


Correct Answer

A - Anterior mediastinal lymph nodes

Explanation Why

The anterior mediastinal lymph nodes are involved in the metastatic spread of breast cancer to the
lungs; they are not a route for metastasis to the spine.

B - Lateral axillary lymph nodes

Image

Explanation Why

The lateral axillary lymph nodes receive lymphatic drainage from the upper limb. They can be
affected in patients with breast cancer if retrograde lymphatic flow and tumor spread occurs as a
result of obstruction of the central and apical axillary lymph nodes (upstream nodes) by tumor

www.eduwaves360.com | Telegram : @eduwaves360


metastases. They are not a route for metastasis to the spine.

C - Azygos vein

Image

Explanation But

A similar route via the internal iliac vein, which connects the prostatic venous plexus with the
vertebral venous plexus, is responsible for metastasis from prostate cancer to the lumbar vertebrae.

Explanation Why

The azygos vein receives blood from the right intercostal veins, which in turn receive blood from the
right breast. The azygos vein has numerous anastomoses with the vertebral venous plexus, which is a
longitudinal network of valveless veins in the spine that includes the paravertebral plexus of Batson.
The lack of valves in this network facilitates the spread of tumors and infections. Spinal metastases
from breast cancer most commonly develop in the thoracic vertebrae. Metastases to bone are most
commonly the result of hematogenous spread.

www.eduwaves360.com | Telegram : @eduwaves360


D - Thyrocervical trunk

Image

Explanation Why

The thyrocervical trunk receives blood from the subclavian artery. It is a potential route for
metastasis of lung cancer to the scapula via the suprascapular branch, or to the thyroid via the
inferior thyroid artery. It is not a potential route for metastasis of breast cancer to the spine.

www.eduwaves360.com | Telegram : @eduwaves360


E - Thoracic duct

Image

Explanation Why

The thoracic duct receives lymphatic drainage from the left axillary lymph nodes and drains into the
left subclavian vein. It is a potential route for pulmonary metastasis in the case of carcinoma of the
left breast, but it is not a potential route for metastasis of right-sided breast cancer to the spine.
Metastases to bone due to lymphatic spread are uncommon.

www.eduwaves360.com | Telegram : @eduwaves360


F - Intercostal artery

Image

Explanation Why

The anterior and posterior intercostal arteries are potential routes for metastasis of lung cancer to
structures of the thoracic wall (e.g., ribs); they are not routes for metastasis of breast cancer.

www.eduwaves360.com | Telegram : @eduwaves360


Question # 38

A 38-year-old man comes to the physician because of a 3-week history of right-sided knee pain. He
works as a bricklayer and reports that the pain worsens when he kneels. He has no history of trauma.
Examination of the right knee shows erythema, fluctuant swelling, and tenderness on palpation of the
kneecap. Passive flexion of the right knee elicits pain. Which of the following structures is most likely
affected in this patient?

Answer Image

A Tibial tubercle

B Anserine bursa

C Medial meniscus

D Suprapatellar bursa

E Synovial membrane

www.eduwaves360.com | Telegram : @eduwaves360


Answer Image

F Prepatellar bursa

www.eduwaves360.com | Telegram : @eduwaves360


Hint

This patient's occupational history puts him at risk for an overuse injury known as housemaid's knee.

www.eduwaves360.com | Telegram : @eduwaves360


Correct Answer

A - Tibial tubercle

Explanation Why

Traction apophysitis of the tibial tubercle (i.e., Osgood-Schlatter disease) is an overuse injury caused
by repeated strain of the patellar ligament on this structure (e.g., during sprinting and jumping
sports). This condition manifests with anterior knee pain that is worse with activity and extension
against resistance. However, physical examination would show enlargement and tenderness to
palpation over the tibial tubercle rather than the patella. Additionally, Osgood-Schlatter disease is not
associated with repetitive kneeling and is seen most commonly in teenage males.

B - Anserine bursa

Explanation Why

Inflammation of the anserine bursa, which is located below the pes anserinus tendon, can lead to
anteromedial tibial pain, especially when rising from a seated position or climbing stairs. Physical
examination would show swelling and tenderness to palpation of the medial side of the knee and
proximal tibia rather than over the patella. Moreover, this patient does not have any known risk
factors for pes anserine pain syndrome, e.g., knee osteoarthritis, obesity, or valgus knee deformity.

www.eduwaves360.com | Telegram : @eduwaves360


C - Medial meniscus

Image

Explanation Why

Overuse of the knee (e.g., excessive kneeling or squatting) can lead to chronic degenerative changes
of the medial meniscus in middle-aged to older adults. Patients often present with knee pain that is
worse with weight‑bearing or physical activity. However, a degenerative meniscus injury typically
manifests with restricted knee extension and knee instability, which patients describe as a feeling of a
knee locking or “giving out” while walking or climbing stairs, none of which is seen here.
Additionally, physical examination would show medial joint line tenderness with positive testing
(e.g., McMurray test, Apley grind test) rather than patellar erythema, tenderness, and swelling.

www.eduwaves360.com | Telegram : @eduwaves360


D - Suprapatellar bursa

Explanation Why

Inflammation of the suprapatellar bursa, which is located above the kneecap and facilitates
movement of the quadriceps tendons over the femur during knee flexion and extension, can be
caused by repeated flexion of the knee joint (e.g., repeated kneeling during bricklaying). However,
physical examination would show erythema, swelling, and tenderness to palpation above the kneecap
rather than over the anterior patella.

E - Synovial membrane

Explanation Why

Inflammation of the synovial membrane can indicate an inflammatory joint disease such as
rheumatoid arthritis (RA). RA usually manifests with symmetrical polyarthritis (most commonly in
MCP and PIP joints) with prolonged morning stiffness. Unilateral pain and swelling of the knee, as
seen in this patient, is a less likely first manifestation of RA. Moreover, local erythema is usually not
seen in RA. Finally, the fact that this patient must kneel as part of his job puts him at risk for damage
of a different structure.

www.eduwaves360.com | Telegram : @eduwaves360


F - Prepatellar bursa

Image

Explanation Why

Inflammation of the prepatellar bursa, which is located between the skin and kneecap, can be caused
by repeated flexion of the knee joint or recurrent trauma. Individuals with jobs that require frequent
kneeling (e.g., bricklayers, plumbers, mechanics) are especially prone to developing prepatellar
bursitis. The condition commonly manifests with localized erythema, fluctuant swelling (the
inflamed bursa), and tenderness to palpation and movement of the kneecap, all of which are observed
in this patient. Surgery may be indicated (e.g., drainage of pus, bursectomy) for recurrent bursitis if
conservative management fails.

www.eduwaves360.com | Telegram : @eduwaves360


Question # 39

A 34-year-old man is brought to the emergency department 3 hours after being bitten by a rattlesnake.
He was hiking in the Arizona desert when he accidentally stepped on the snake and it bit his right leg.
His pulse is 135/min and blood pressure is 104/81 mm Hg. Examination shows right lower leg swelling,
ecchymosis, and blistering. Right ankle dorsiflexion elicits severe pain. A manometer inserted in the
lateral compartment of the lower leg shows an intracompartmental pressure of 67 mm Hg. In addition to
administration of the antivenom, the patient undergoes fasciotomy. Two weeks later, he reports difficulty
in walking. Neurologic examination shows a loss of sensation over the lower part of the lateral side of
the right leg and the dorsum of the right foot. Right foot eversion is 1/5. There is no weakness in
dorsiflexion. Which of the following nerves is most likely injured in this patient?

Answer Image

A Tibial nerve

B Sural nerve

C Deep peroneal nerve

D Superficial peroneal nerve

E Saphenous nerve

www.eduwaves360.com | Telegram : @eduwaves360


Hint

This patient's symptoms are most likely due to intraoperative injury of the nerve that innervates the
fibularis longus and fibularis brevis muscles.

www.eduwaves360.com | Telegram : @eduwaves360


Correct Answer

A - Tibial nerve

Explanation Why

A tibial nerve injury causes impaired motor function of the flexor muscles of the foot, leading to an
inability to invert the foot or walk on the toes or balls of the foot, none of which is seen here. Patients
will also experience sensory loss on the sole of the foot, not the dorsum of the foot. Moreover, injury
during a lateral fasciotomy is unlikely because the tibial nerve is located in the deep posterior
compartment of the leg.

B - Sural nerve

Explanation Why

The sural nerve is a purely sensory branch of the tibial nerve, which runs superficially to the
superficial posterior compartment of the leg. Injury to this nerve causes impaired sensation on the
posterolateral side of the lower leg, ankle, heel, and foot, but not on the anterolateral side of the leg
and dorsum of the foot. Moreover, patients will not present with motor findings. Finally, injury to
this nerve during lateral fasciotomy is unlikely given its location on the posterior aspect of the leg.

C - Deep peroneal nerve

Explanation Why

A deep peroneal nerve injury causes impaired motor function of the foot and toe extensors as well as
decreased sensation in the webspace between the hallux and second digit of the foot. Patients will
present with difficulties with dorsiflexion and inversion that lead to the characteristic steppage gait.
This patient does not have any weakness in dorsiflexion. Moreover, injury during a lateral
fasciotomy is unlikely because the deep peroneal nerve is located in the anterior compartment of the
leg.

www.eduwaves360.com | Telegram : @eduwaves360


D - Superficial peroneal nerve

Image

Explanation Why

Weakness with foot eversion and sensory loss to the dorsum of the foot (except the first digital
webspace) and anterolateral leg indicate an injury of the superficial peroneal (fibular) nerve, which is
located in the lateral compartment of the leg. This patient originally presented with features of
compartment syndrome (severe leg swelling and pain, increased compartment pressure). Emergent
fasciotomy is the treatment of choice in order to prevent vascular compromise and tissue ischemia;
however, it is associated with a risk of nerve injury, as seen here. Risk factors for compartment
syndrome include animal venoms (e.g., rattlesnake bites), circumferential burns, long bone fractures,
and crush injuries.

www.eduwaves360.com | Telegram : @eduwaves360


E - Saphenous nerve

Explanation Why

The saphenous nerve is a purely sensory nerve that innervates the medial side of the leg, knee, and
foot. Fasciotomy of the lateral compartment of the leg would not affect the saphenous nerve because
it travels medially down the leg to the foot. Additionally, injury to the saphenous nerve would not
present with motor findings.

www.eduwaves360.com | Telegram : @eduwaves360


Question # 40

A 2860-g (6-lb 3-oz) male newborn is born at term to a primigravid woman via spontaneous vaginal
delivery. The mother has had no routine prenatal care. She reports that there is no family history of
serious illness. The initial examination of the newborn shows bowing of the legs and respiratory distress
upon palpation of the chest. The skin and joints are hyperextensible. X-rays of the chest and skull show
multiple rib fractures and small, irregular bones along the cranial sutures. The patient is at increased risk
of which of the following complications?

Answer Image

A Costochondral junction enlargement

B Intestinal rupture

C Intellectual disability

D Spinal canal stenosis

E Hearing loss

www.eduwaves360.com | Telegram : @eduwaves360


Hint

This patient's presentation with multiple perinatal fractures, bowing of legs, and hyperextensibility of the
skin and joints suggests a diagnosis of osteogenesis imperfecta.

www.eduwaves360.com | Telegram : @eduwaves360


Correct Answer

A - Costochondral junction enlargement

Explanation Why

Costochondral junction enlargement (rachitic rosary) is characteristic of rickets, which can cause
bone deformities (including tibial bowing) and impaired growth, as well as fractures in severe cases.
However, rickets does not cause skin or joint hypermobility. Costochondral prominence is not a
feature of osteogenesis imperfecta.

B - Intestinal rupture

Explanation Why

Intestinal rupture is a complication seen in patients with Ehlers-Danlos syndrome (EDS), especially
the vascular subtype, wherein defective synthesis of collagen type III leads to the formation of fragile
tissue that is susceptible to rupture. Affected individuals may present with hyperextensible skin and
joints, as seen here, but multiple bone fractures are not a feature of EDS. Intestinal rupture is not
associated with osteogenesis imperfecta.

C - Intellectual disability

Explanation Why

Several causes of short stature, including chromosomal abnormalities (e.g., Down syndrome,
Williams syndrome), inborn errors of metabolism (e.g., Lesch-Nyhan syndrome, homocystinuria),
and exposure to toxins in utero (e.g., fetal alcohol syndrome), can cause intellectual disability.
However, these conditions do not manifest with perinatal fractures and skin and joint
hyperextensibility. Patients with osteogenesis imperfecta typically have preserved cognitive ability.

www.eduwaves360.com | Telegram : @eduwaves360


D - Spinal canal stenosis

Explanation Why

Spinal stenosis can occur in patients with achondroplasia, resulting in low back and leg pain,
paresthesias, and incontinence. Patients with achondroplasia may also present with respiratory
distress due to a small chest cavity and varus deforming of the legs. However, achondroplasia does
not cause skin or joint hypermobility, nor does it increase the risk of developing fractures. Spinal
stenosis is not a complication of osteogenesis imperfecta.

E - Hearing loss

Image

Explanation But

In this patient, the described irregular bones around the cranial sutures are indicative of wormian
bones. Excessive amounts (> 10) of wormian bones are a feature of OI.

www.eduwaves360.com | Telegram : @eduwaves360


Explanation Why

Hearing loss occurs in patients with osteogenesis imperfecta (OI) as a result of bone formation
defects such as otosclerosis and fractures and atrophy of the ossicles. Autosomal dominant mutations
in the COL1A1 or COL1A2 genes, which encode the alpha chains of type I collagen, are responsible
for most cases of OI. The severe subtypes (types II, III) of OI are usually due to a new (sporadic)
mutation in one of these genes, while patients with the mild forms (types I, IV) typically have a
parent with the condition. Additional findings in patients with OI include blue sclerae (due to
choroidal veins appearing through the overlying thin sclerae), easy bruisability, and brittle,
opalescent teeth.

www.eduwaves360.com | Telegram : @eduwaves360


www.eduwaves360.com | Telegram : @eduwaves360
Join us on Telegram :

Click here : @eduwaves360

Unlocked the Medical premiums

Click here : www.eduwaves360.com

Medical Courses : https://t.me/usmle_study_materials_2

Discussion Group : @usmle_discussion_group

www.eduwaves360.com | Telegram : @eduwaves360


Question # 1

A 22-year-old man comes to the physician because of a progressive swelling and pain in his right ring
finger for the past 2 days. The pain began while playing football, when his finger got caught in the
jersey of another player who forcefully pulled away. Examination shows that the right ring finger is
extended. There is pain and swelling at the distal interphalangeal joint. When the patient is asked to
make a fist, his right ring finger does not flex at the distal interphalangeal joint. There is no joint laxity.
Which of the following is the most likely diagnosis?

Answer Image

Rupture of the flexor digitorum


A profundus tendon at its point of
insertion

B Closed fracture of the distal phalanx

Rupture of the extensor digitorum


C
tendon at its point of insertion

Inflammation of the flexor digitorum


D
profundus tendon sheath

Slipping of the central band of the


E
extensor digitorum tendon

www.eduwaves360.com | Telegram : @eduwaves360


Hint

This patient's injury is a common sports injury caused by forceful hyperextension of the DIP.

www.eduwaves360.com | Telegram : @eduwaves360


Correct Answer

A - Rupture of the flexor digitorum profundus tendon at its point of insertion

Image

Explanation But

The ring finger is the most commonly affected finger.

Explanation Why

Rupture of the flexor digitorum profundus (FDP) tendon, also known as “jersey finger,” is a common
sports injury. The FDP, along with the flexor digitorum superficialis (FDS), is responsible for finger
flexion. The tendons from the FDP muscle run through the carpal tunnel and insert on the palmar
side of the distal phalanges of each finger, whereas the FDS tendons insert on the middle phalanx.
This patient's inability to flex the DIP combined with the pain over the DIP joint, which is distal to
the attachment of the FDS, indicate that he has ruptured his FDP tendon.

www.eduwaves360.com | Telegram : @eduwaves360


B - Closed fracture of the distal phalanx

Explanation Why

The distal phalanges are the most commonly fractured bones in the hand. Distal phalanx fracture is a
common sports-related injury, but it would present with pain over the phalanx itself, rather than over
one of the interphalangeal joints. While deformity of the distal phalanx and nail is common with
distal phalanx fractures, a specific deficit in either flexion or extension over a particular joint would
be less common.

C - Rupture of the extensor digitorum tendon at its point of insertion

Explanation Why

The extensor digitorum (ED) tendons extend the fingers of the hand. Rupture of the ED tendons
leads to a pathology known as “mallet finger” or “baseball finger.” This injury often occurs when a
ball hits the tip of the finger and hyperflexes the distal interphalangeal joint (DIP). Patients with this
injury would be unable to extend the DIP of the affected finger and would likely have more pain on
the dorsal aspect of the finger over the DIP.

D - Inflammation of the flexor digitorum profundus tendon sheath

Explanation Why

Inflammation of the flexor tendon sheath is a condition known as “trigger finger.” The inflammation
causes a stenosing tenosynovitis that leads to a locking or catching sensation during active
flexion-extension activity, which can progress to the extent that the digit can only be extended
through passive manipulation. Patients may also have a palpable nodule along the axis of the flexor
tendon. The cause of trigger finger is unknown but may be related to repeated digital flexion.

www.eduwaves360.com | Telegram : @eduwaves360


E - Slipping of the central band of the extensor digitorum tendon

Explanation Why

Slipping or disruption of the central band of the extensor digitorum (ED) tendon is a condition
known as Boutonniere deformity. This can be caused by sharp or blunt trauma, such as jamming a
finger in a door. It can also be caused by rheumatoid arthritis, in which case the inflammation of the
joint space pushes the central band of the ED out of its normal anatomical position. Patients with
Boutonniere deformity present with hyperextension of the distal interphalangeal joint and flexion of
the proximal interphalangeal joint.

www.eduwaves360.com | Telegram : @eduwaves360


Question # 2

A 49-year-old man comes to the physician because of severe, shooting pain in his lower back for the
past 2 weeks. The pain radiates down the back of both legs and started after he lifted a concrete manhole
cover from the ground. Physical examination shows decreased sensation to light touch bilaterally over
the lateral thigh area and lateral calf bilaterally. Patellar reflex is decreased on both sides. The passive
raising of either the right or left leg beyond 30 degrees triggers a shooting pain down the leg past the
knee. Which of the following is the most likely underlying cause of this patient's current condition?

Answer Image

Involuntary contraction of the


A
paraspinal muscles

Inflammatory degeneration of
B
the spine

Compromised integrity of the


C
vertebral body

Herniation of nucleus pulposus


D
into vertebral canal

Inflammatory reaction in the


E
epidural space

www.eduwaves360.com | Telegram : @eduwaves360


Hint

Severe lower back pain radiating down the legs together with decreased sensation to touch, decreased
reflexes, and positive Lasègue sign (pain when raising legs passively) is suggestive of lumbar
radiculopathy. The association between heavy lifting and the onset of symptoms suggests an acute,
mechanical mechanism of injury.

www.eduwaves360.com | Telegram : @eduwaves360


Correct Answer

A - Involuntary contraction of the paraspinal muscles

Explanation Why

Involuntary contraction of the paraspinal muscles could be caused by a back spasm due to heavy
lifting, which would present with back pain. However, pain would usually be localized and not
radiate along a dermatome of a nerve root. Furthermore, decreased sensation to touch, decreased
reflexes, and positive Lasègue sign would not be expected.

B - Inflammatory degeneration of the spine

Explanation Why

Inflammatory degeneration of the spine can be caused by spinal arthritis due to ankylosing
spondylitis, degenerative osteoarthritis, or psoriatic arthritis, which could present with pain and
stiffness in the lower back. However, pain is usually dominant at rest, improves with activity, and has
a chronic duration rather than an acute onset after heavy lifting. Furthermore, decreased sensation to
touch, decreased reflexes, and positive Lasègue sign would not be expected.

C - Compromised integrity of the vertebral body

Explanation Why

A compromised integrity of the vertebral body could be caused by pathologic vertebral fracture (due
to osteoporosis or trauma), lytic metastases (cancer), or anterior wedge compression fracture due to
Pott's disease (tuberculosis). These could present with acute back pain and L4 radiculopathy.
However, this patient does not have any significant risk factors for osteoporosis (e.g., elderly, chronic
glucocorticoid use, post-menopausal female) or TB (e.g., immunosuppression, drug/alcohol abuse,
congregate settings), nor does he have a history of significant trauma (e.g., fall from height), making
vertebral fracture and Pott's disease unlikely.

www.eduwaves360.com | Telegram : @eduwaves360


D - Herniation of nucleus pulposus into vertebral canal

Image

Explanation But

Herniation most often occurs at L4/L5 or L5/S1 and normally affects the nerve below.

Explanation Why

The nucleus pulposus is the gelatinous core of the intervertebral disc and is surrounded by a dense
outer ring, the annulus fibrosus. Compression, tension, and shear and torque stresses on the spinal
disc (as with heavy lifting) together with degenerative changes (e.g., dehydration) can lead to a tear
in the annulus fibrosus, resulting in herniation of the nucleus pulposus into the vertebral canal with
subsequent compression of the spinal cord and/or nerve roots. This nerve impingement causes
symptoms of radiculopathy, which differ depending on the level of herniation.

www.eduwaves360.com | Telegram : @eduwaves360


E - Inflammatory reaction in the epidural space

Explanation Why

An inflammatory reaction in the epidural space might be caused by an epidural abscess or bone
metastases. Both could present with acute back pain and signs of spinal cord compression. However,
the patient has no evidence of cancer. Furthermore, an epidural abscess would demonstrate point
tenderness and constitutional symptoms (fever, chills) and is usually associated with risk factors such
as IV drug use, diabetes, or immunosuppression. Additionally, the acute onset of symptoms makes
both diagnoses unlikely.

www.eduwaves360.com | Telegram : @eduwaves360


Question # 3

A 15-year-old boy comes to the physician because of severe muscle cramps and pain for 3 months. He
first noticed these symptoms while attending tryouts for the high school football team. Since then, he
becomes easily fatigued and has severe muscle pain and swelling after 10 minutes of playing. However,
after a brief period of rest, the symptoms improve, and he is able to return to the game. Two days ago, he
had an episode of reddish-brown urine after playing football. There is no family history of serious
illness. He appears healthy. Vital signs are within normal limits. Physical and neurological examinations
show no abnormalities. Serum creatine kinase concentration is 333 U/L. Urinalysis shows:

Blood 2+

Protein negative

Glucose negative

RBC negative

WBC 1–2/hpf

Which of the following is the most likely cause of this patient's symptoms?

Answer Image

A Medium chain Acyl CoA dehydrogenase deficiency

B Thyroid hormone deficiency

C Dystrophin gene mutation

D Myophosphorylase deficiency

www.eduwaves360.com | Telegram : @eduwaves360


Answer Image

E Acid maltase deficiency

F CTG repeat in the DMPK gene

www.eduwaves360.com | Telegram : @eduwaves360


Hint

This individual has severe muscle cramps and pain after physical activity along with myoglobinuria
(red-brown urine with positive blood but no RBCs) and elevated serum creatine kinase. These features
along with the second wind phenomenon (ability to resume physical activity after a break) is indicative
of McArdle's disease.

www.eduwaves360.com | Telegram : @eduwaves360


Correct Answer

A - Medium chain Acyl CoA dehydrogenase deficiency

Explanation Why

Medium chain acyl-CoA dehydrogenase deficiency (MCAD deficiency) may present with exercise
intolerance and elevated serum creatinine kinase due to muscle catabolism. However, this weakness
would develop with more prolonged exercise, not within a few minutes. Additionally, patients with
MCAD deficiency would not experience the rapid symptom improvement seen in this patient.

B - Thyroid hormone deficiency

Explanation Why

Thyroid hormone deficiency causes hypothyroidism, which can present with myopathy as seen in
this patient. However, these symptoms would be present at rest rather than improve with rest as seen
in this patient. Additionally, hypothyroidism would likely present with other general symptoms such
as fatigue, cold intolerance, cold skin, weight gain, constipation, delayed reflexes, or myxedema.
Finally, hypothyroidism is uncommon in otherwise healthy teenage boys (5-8 times more common in
women).

C - Dystrophin gene mutation

Explanation Why

Dystrophin gene mutations cause Duchenne muscular dystrophy or Becker muscular dystrophy,
depending on the type of mutation. While these disorders can result in myalgias and elevated creatine
kinase, they are usually progressive conditions that would demonstrate symptoms both at rest and
with exercise. Additionally, symptoms in dystrophinopathies would not improve so significantly with
rest. Lastly, Duchenne muscular dystrophy typically presents before age 5 and usually leads to
ambulatory inability before age 13.

www.eduwaves360.com | Telegram : @eduwaves360


D - Myophosphorylase deficiency

Explanation Why

Myophosphorylase deficiency (McArdle's disease) is a glycogen storage disease that presents with
myalgias, rhabdomyolysis, myoglobinuria, and early fatigue from exercise due to inability to break
down glycogen for use as energy in muscle tissues. Many patients experience the second wind
phenomenon, in which symptoms improve after a brief period of rest. Treatment often involves
aerobic exercise programs and nutritional supplementation, although evidence of benefit is limited.

E - Acid maltase deficiency

Explanation But

Although there is a late-onset form of Pompe's disease that manifests in adolescents and young
adults, another diagnosis would be far more likely here.

Explanation Why

Lysosomal acid maltase deficiency (Pompe's disease) is a glycogen storage disease that can present
with myopathy and exercise intolerance, as seen in this patient. However, Pompe's disease is usually
diagnosed in infants and young children who present with failure to thrive along with myopathy and,
sometimes, hypertrophic cardiomyopathy and/or macroglossia. Moreover, the rapid improvement of
this patient's symptoms following rest is characteristic of another disease.

F - CTG repeat in the DMPK gene

Explanation Why

CTG repeats in the DMPK gene cause myotonic dystrophy. While myotonic dystrophy can be
associated with muscle pain and weakness, it causes symptoms at rest and is associated with delayed
muscle relaxation following contraction (myotonia). Additionally, myotonic dystrophy is associated
with arrhythmias, cataracts, testicular atrophy, and insulin resistance.

www.eduwaves360.com | Telegram : @eduwaves360


www.eduwaves360.com | Telegram : @eduwaves360
Question # 4

A 52-year-old man comes to the physician because of right shoulder pain that began after he repainted
his house 1 week ago. Physical examination shows right subacromial tenderness. The pain is reproduced
when the patient is asked to abduct the shoulder against resistance with the arm flexed forward by 30°
and the thumb pointing downwards. The tendon of which of the following muscles is most likely to be
injured in this patient?

Answer Image

A Supraspinatus

B Subscapularis

C Infraspinatus

D Teres minor

www.eduwaves360.com | Telegram : @eduwaves360


Answer Image

E Deltoid

www.eduwaves360.com | Telegram : @eduwaves360


Hint

Physical examination shows a positive empty can test, a maneuver that specifically assesses one of the
rotator cuff muscles.

www.eduwaves360.com | Telegram : @eduwaves360


Correct Answer

A - Supraspinatus

Image

Explanation Why

Pain during the empty can test suggests injury to the supraspinatus tendon and/or muscle. In this test,
forward flexion and internal rotation of the arm enables isolated examination of the supraspinatus
muscle, independent of the deltoid muscle.

B - Subscapularis

Explanation Why

Injury of the subscapularis muscle manifests with shoulder pain and/or weakness but it is not
associated with subacromial tenderness or a positive empty can test. Injury of the subscapularis

www.eduwaves360.com | Telegram : @eduwaves360


muscle can be clinically assessed using the lift-off test.

C - Infraspinatus

Image

Explanation Why

Injury of the infraspinatus muscle manifests with shoulder pain but it is not associated with
subacromial tenderness or a positive empty can test. It can be clinically assessed using the
infraspinatus test.

D - Teres minor

Explanation Why

Injury of the teres minor muscle manifests as shoulder pain and/or weakness but it is not associated
with subacromial tenderness or a positive empty can test. Instead, injury of the teres minor muscle

www.eduwaves360.com | Telegram : @eduwaves360


causes a positive hornblower's sign.

E - Deltoid

Explanation Why

Injury of the deltoid muscle manifests with shoulder pain and/or weakness but it is not associated
with subacromial tenderness or a positive empty can test. Injury of the deltoid muscle can be
clinically assessed using the akimbo test.

www.eduwaves360.com | Telegram : @eduwaves360


Question # 5

A 45-year-old man comes to the physician for a follow-up examination after being diagnosed with
hypertension 6 months ago. He has cut salt out of his diet and started exercising regularly, but home
blood pressure measurements continue to be elevated. His blood pressure is 160/85 mm Hg. An
antihypertensive medication is prescribed that decreases blood pressure by decreasing the
transmembrane calcium current across vascular smooth muscle cells. Side effects include peripheral
edema and flushing. Which of the following best describes why this drug does not affect skeletal muscle
contraction?

Answer Image

Skeletal muscle lacks voltage-gated L-type


A
calcium channels

Skeletal muscle contraction occurs


B independently of extracellular calcium
influx

Skeletal muscle ryanodine receptor


C activation occurs independently of
membrane depolarization

Skeletal muscle preferentially expresses N-


D
type and P-type calcium channels

www.eduwaves360.com | Telegram : @eduwaves360


Answer Image

Skeletal muscle calcium channels do not


E undergo conformational change when bound
to this drug

www.eduwaves360.com | Telegram : @eduwaves360


Hint

This patient has been prescribed a dihydropyridine calcium channel blocker.

www.eduwaves360.com | Telegram : @eduwaves360


Correct Answer

A - Skeletal muscle lacks voltage-gated L-type calcium channels

Explanation Why

Skeletal muscle does not lack voltage-gated L-type calcium channels. In fact, the DHPRs of skeletal
muscle are L-type calcium channels, which are blocked by calcium channel blockers like the one
prescribed to this patient.

B-

Skeletal muscle contraction occurs independently of extracellular calcium


influx

Image

www.eduwaves360.com | Telegram : @eduwaves360


Explanation Why

Skeletal muscle contraction occurs independently of extracellular calcium influx and is instead
dependent on the release of intracellular calcium stored in the sarcoplasmic reticulum. Skeletal
muscle contraction begins with depolarization of a muscle cell via nicotinic acetylcholine receptors.
Sensing the voltage change, dihydropyridine receptors (DHPRs) in the muscle cell membrane
undergo conformational changes, as do mechanically coupled ryanodine receptor channels, which
then open to allow an influx of calcium for the cross-bridge cycle into the sarcoplasm. Although
DHPRs are L-type calcium channels, which are blocked by calcium channel blockers like the one
prescribed here, their primary role in skeletal muscle contraction is voltage-sensing rather than
calcium permeability, and this role is not affected by calcium channel blockers.

C-

Skeletal muscle ryanodine receptor activation occurs independently of


membrane depolarization

Explanation Why

Skeletal muscle ryanodine receptor activation does not occur independently of membrane
depolarization. Rather, ryanodine receptors are activated by dihydropyridine receptors (DHPRs),
with which they are mechanically coupled, and which have been activated by depolarization of the
muscle cell membrane in response to activation of nicotinic acetylcholine receptors at the motor end
plate.

D-

Skeletal muscle preferentially expresses N-type and P-type calcium channels

Explanation Why

Skeletal muscle does not preferentially express N-type and P-type calcium channels. The DHPRs
instrumental in skeletal muscle contraction are L-type calcium channels, which are blocked by
calcium channel blockers like the one prescribed to this patient. While N-type calcium channels and
P-type calcium channels are found in neurons and involved in neurotransmitter release, they do not
play a role in skeletal muscle contraction.

www.eduwaves360.com | Telegram : @eduwaves360


E-

Skeletal muscle calcium channels do not undergo conformational change when


bound to this drug

Explanation Why

DHPRs in skeletal muscle are L-type calcium channels. When bound to calcium channel blockers
like the one prescribed to this patient, they undergo a conformational change that inhibits the influx
of extracellular calcium.

www.eduwaves360.com | Telegram : @eduwaves360


Question # 6

A 72-year-old man comes to the physician for a routine physical examination. He does not take any
medications. Physical examination shows no abnormalities. Laboratory studies show a calcium
concentration of 8.5 mg/dL, a phosphorus concentration of 3.1 mg/dL, an elevated bone-specific
alkaline phosphatase concentration, and a normal urine deoxypyridinoline concentration. Which of the
following is the most likely explanation for this patient's laboratory abnormalities?

Answer Image

A Increased parafollicular C-cell activity

B Decreased osteoclast activity

C Increased osteoblast activity

D Decreased parathyroid chief cell activity

E Increased chondroblast activity

www.eduwaves360.com | Telegram : @eduwaves360


Hint

Bone-specific alkaline phosphatase (ALP) is a marker of bone mineralization.

www.eduwaves360.com | Telegram : @eduwaves360


Correct Answer

A - Increased parafollicular C-cell activity

Explanation Why

Increased activity of thyroid parafollicular C cells would result in increased secretion of calcitonin.
Calcitonin binds to and inhibits osteoclasts and is released by C cells in response to increases in
serum calcium. However, this patient's serum calcium is normal, and C cell activity would not
explain the elevated bone-specific ALP seen here.

B - Decreased osteoclast activity

Explanation Why

Decreased osteoclast activity, which corresponds with decreased bone resorption, would appear in
conjunction with normal to depressed levels of urinary deoxypyridinoline and acid phosphatase, both
of which are markers of osteoclast activity. However, decreased osteoclast activity would not explain
this patient's elevated bone-specific ALP.

C - Increased osteoblast activity

Explanation But

ALP is also found elsewhere in the body, most importantly in the biliary system, where it can be used
as a marker for cholestasis. It is also found in the placenta, and placental ALP is a tumor marker for
seminoma.

Explanation Why

Increased osteoblast activity would explain the elevated bone-specific ALP concentration seen in this
patient. Bone-specific ALP is secreted by osteoblasts and catalyzes the hydrolysis of various
molecules, yielding inorganic phosphate, a substance that is essential for bone mineralization. Other

www.eduwaves360.com | Telegram : @eduwaves360


clinically useful markers of osteoblast activity include osteocalcin and propeptides of type I
procollagen. Bone-specific ALP is elevated in conditions that involve increased bone mineralization
such as Paget disease of the bone and bone metastases.

D - Decreased parathyroid chief cell activity

Explanation Why

Decreased parathyroid chief cell activity would result in decreased secretion of PTH. PTH has
several effects, one of which is to increase osteoclast resorption of bone and subsequent release of
calcium and phosphate into the blood. Decreased PTH secretion would therefore cause decreased
osteoclast bone resorption and lower levels of calcium and phosphate in the blood, none of which is
seen here. Furthermore, decreased PTH would not be expected to cause an elevated concentration of
bone-specific ALP.

E - Increased chondroblast activity

Explanation Why

Chondroblasts act as mesenchymal precursors during the endochondral ossification process and
secrete extracellular matrix. Increased chondroblast activity occurs in chondroblastoma, which would
not typically manifest with an isolated elevation of bone-specific alkaline phosphatase in an elderly
patient without bone pain. Instead, the peak incidence for chondroblastoma is 10–20 years of age,
and typical clinical findings include progressive pain.

www.eduwaves360.com | Telegram : @eduwaves360


Question # 7

A 48-year-old woman comes to the physician because of a 6-month history of muscle stiffness, myalgia,
and a 7-kg (15-lb) weight gain. Her last menstrual period was 4 months ago. Physical examination
shows cold, dry skin, and proximal muscle weakness. Deep tendon reflexes are 2+ bilaterally, with
delayed relaxation. The serum creatine kinase concentration is 2,940 U/L. Which of the following is the
most appropriate next step in diagnosis?

Answer Image

A Genetic testing

B Thyroid function tests

Serum assay for muscle specific


C
tyrosine kinase antibody

D Muscle biopsy

E Serum electrolytes

F Repetitive nerve stimulation

www.eduwaves360.com | Telegram : @eduwaves360


Hint

The patient has muscle stiffness, pain, weakness, delayed muscle relaxation on examination of reflexes,
and elevated creatine kinase, all of which indicates myopathy. Her other clinical features suggest that
these features may be due to an underlying condition.

www.eduwaves360.com | Telegram : @eduwaves360


Correct Answer

A - Genetic testing

Explanation Why

Genetic testing is indicated in patients with suspected hereditary muscle disorders, such as myotonic
dystrophies, which may also present with elevated creatine kinase levels, proximal muscle weakness,
and muscle pain. However, this patient lacks other typical findings of myotonic dystrophies, such as
cataracts, arrhythmias, and cognitive impairment. Moreover, this patient's weight gain and dry skin
make another diagnosis more likely.

Individuals with Duchenne muscular dystrophy also present with elevated creatine kinase levels,
proximal muscle weakness, and muscle pain. However, this disease occurs in young boys usually by
2–3 years of age.

www.eduwaves360.com | Telegram : @eduwaves360


B - Thyroid function tests

Image

Explanation Why

This patient presents with typical symptoms of hypothyroidism, including weight gain, dry skin,
amenorrhea, and myopathy. Other typical features of hypothyroidism include fatigue, cold
intolerance, hair loss, constipation, impaired cognition, and depression. In patients with
hypothyroidism, a shortage of thyroid hormone leads to decreased metabolic function (↓ protein
turnover and ↓ carbohydrate metabolism), which often manifests in the muscles. Muscle pain is a
typical feature of impaired carbohydrate metabolism. Hypothyroid myopathy typically presents with
slowed contraction and the delayed relaxation of muscles. To confirm the suspected diagnosis, the
best initial step here is to determine TSH levels and T4 levels.

www.eduwaves360.com | Telegram : @eduwaves360


C - Serum assay for muscle specific tyrosine kinase antibody

Explanation Why

Testing for antibodies against the muscle-specific tyrosine kinase autoantibody (MuSK-Ab), or the
acetylcholine receptor (AChR-Ab), is indicated in patients with suspected myasthenia gravis (MG),
which can present with proximal muscle weakness that worsens with exercise. However, this patient
lacks other typical findings of MG, such as eye muscle weakness that causes ptosis, diplopia, and
blurred vision, or bulbar muscle weakness, which causes slurred speech and difficulty chewing and/
or swallowing. Moreover, the weight gain, dry skin, and markedly elevated creatine kinase levels in
this patient make another diagnosis more likely.

D - Muscle biopsy

Explanation Why

Muscle biopsy is indicated in patients with suspected inflammatory myopathies (i.e., polymyositis,
dermatomyositis), which can also present with dry skin, elevated creatine kinase levels, and proximal
muscle weakness. However, this patient lacks other typical findings of inflammatory myopathies,
such as dysphagia (present in ∼ 30% of patients), and cutaneous manifestations (e.g., Gottron
papules) in the case of dermatomyositis. Moreover, the weight gain in this patient makes another
diagnosis more likely.

E - Serum electrolytes

Explanation Why

Hypokalemia is an electrolyte imbalance that can present with muscle weakness. However, this
patient lacks other typical findings of hypokalemia such as cardiac arrhythmias and constipation.
Moreover, hypokalemia would not cause weight gain or markedly elevated creatine kinase levels.

www.eduwaves360.com | Telegram : @eduwaves360


F - Repetitive nerve stimulation

Explanation Why

Repetitive nerve stimulation is indicated in patients with suspected Lambert-Eaton myasthenic


syndrome (LEMS), which can also present with proximal muscle weakness. However, LEMS
typically occurs in conjunction with an underlying malignancy (small cell lung cancer in two-thirds
of LEMS cases). Moreover, the weight gain, dry skin, and markedly elevated creatine kinase levels
in this patient make another diagnosis more likely.

www.eduwaves360.com | Telegram : @eduwaves360


Question # 8

A 47-year-old woman comes to the physician because of a 1-month history of progressive weakness.
She has had increased difficulty climbing stairs and standing from a seated position. She takes no
medications. Neurologic examination shows weakness of the proximal muscles. Skin examination
shows diffuse erythema of the upper back, posterior neck, and shoulders. A photograph of the patient's
eye is shown. Antibodies against which of the following are most likely to be present in this patient?

Answer Image

A Myeloperoxidase

B Centromeres

C Voltage-gated calcium channels

www.eduwaves360.com | Telegram : @eduwaves360


Answer Image

D La protein

E Scl-70 protein

F Mi-2 protein

G Histones

www.eduwaves360.com | Telegram : @eduwaves360


Hint

This patient has dermatomyositis, which typically manifests with proximal muscle weakness, a
heliotrope rash, and the shawl sign.

www.eduwaves360.com | Telegram : @eduwaves360


Correct Answer

A - Myeloperoxidase

Explanation Why

Antimyeloperoxidase antibodies (p-ANCA) are associated with microscopic polyangiitis and


eosinophilic granulomatosis with polyangiitis. Though these vasculitides can involve the skin,
typical findings include palpable purpura and skin nodules. They are not associated with proximal
muscle weakness, a heliotrope rash, or the shawl sign.

B - Centromeres

Explanation Why

Anticentromere antibodies are typically associated with limited systemic sclerosis (CREST
syndrome), which presents with a tight mask-like face, microstomia, calcinosis cutis, sclerodactyly,
Raynaud phenomenon, telangiectasias, and gastrointestinal symptoms (e.g., dysphagia, reflux).
Limited systemic sclerosis is not associated with proximal muscle weakness, a heliotrope rash, or the
shawl sign.

C - Voltage-gated calcium channels

Explanation Why

Antibodies against presynaptic voltage-gated calcium channels are associated with Lambert-Eaton
myasthenic syndrome (LEMS). This condition typically manifests with proximal muscle weakness
that improves with repetitive use, and patients often have autonomic symptoms such as dry mouth,
constipation, and orthostatic dysregulation. However, LEMS is not associated with a heliotrope rash
or the shawl sign.

www.eduwaves360.com | Telegram : @eduwaves360


D - La protein

Explanation Why

Anti-La and anti-Ro antibodies are associated with Sjögren syndrome. This condition typically
manifests with dry mouth and dry eyes, which results in blurred vision, dysphagia, and dental caries.
Patients can also develop fatigue, arthralgia, and Raynaud phenomenon. Sjögren syndrome is not
associated with proximal muscle weakness, a heliotrope rash, or the shawl sign.

E - Scl-70 protein

Explanation Why

Anti-Scl-70 antibodies are typically associated with diffuse systemic sclerosis, which can present
with fatigue and proximal muscle weakness. However, a heliotrope rash or the shawl sign would not
be expected. Instead, patients would present with features such as a tight mask-like face with
microstomia, sclerodactyly, Raynaud phenomenon, gastrointestinal symptoms (e.g., dysphagia,
reflux), renal disease, and interstitial lung disease.

www.eduwaves360.com | Telegram : @eduwaves360


F - Mi-2 protein

Image

Explanation Why

Anti-Mi-2 antibodies are associated with dermatomyositis and polymyositis. These antibodies, along
with anti-Jo-1 antibodies and anti-SRP antibodies, are relatively specific for these conditions.
Dermatomyositis is associated with several malignancies (e.g., ovarian adenocarcinoma), and all
patients diagnosed with this condition should be evaluated with age-appropriate screening.

G - Histones

Explanation Why

Antihistone antibodies are associated with drug-induced lupus erythematosus (DILE). Medications
known to cause DILE include procainamide, hydralazine, isoniazid, and sulfadiazine. This condition
typically manifests with fever, polyarthritis, serositis, myalgia, and generalized rash. This patient

www.eduwaves360.com | Telegram : @eduwaves360


does not take medications, however, and her rash is not typical for this diagnosis.

www.eduwaves360.com | Telegram : @eduwaves360


Question # 9

A 63-year-old man comes to the physician because of a 4-hour history of severe pain in the right knee.
His temperature is 38.0°C (100.1°F). Examination shows erythema, edema, and warmth of the right
knee; range of motion is limited by pain. Arthrocentesis of the knee joint yields cloudy fluid with a
leukocyte count of 24,000/mm3 (74% segmented neutrophils). Polarized light microscopy shows
needle-shaped, negatively birefringent crystals. Five hours after treatment with a drug is begun, he
develops abdominal cramp, diarrhea, and vomiting. The patient was most likely treated with a drug that
predominantly acts on which of the following parts of the cell?

Answer Image

A Cell wall

B Cytosolic enzyme

C Nucleus

D Cytoskeleton

E Membrane transporter

www.eduwaves360.com | Telegram : @eduwaves360


Hint

This man has acute arthritis of the knee. Arthrocentesis that shows needle-shaped, negatively
birefringent crystals confirms the diagnosis of an acute gout attack. Given his gastrointestinal adverse
effects, he was most likely treated with colchicine.

www.eduwaves360.com | Telegram : @eduwaves360


Correct Answer

A - Cell wall

Explanation Why

Vancomycin interferes with bacterial cell wall synthesis by inhibiting peptidoglycan synthesis. It is
used as empiric treatment for septic arthritis caused by gram-positive cocci. Septic arthritis typically
presents with fever and monoarthritis, as seen in this patient. However, synovial fluid analysis would
show a purulent fluid with a leukocyte count of ≥50,000 cells/mm3 and a positive Gram stain.
Moreover, vancomycin is not associated with abdominal cramping, diarrhea, and vomiting shortly
after administration. Unlike most prokaryotic cells, eukaryotic cells (including human cells) do not
have a cell wall, only a cell membrane.

B - Cytosolic enzyme

Explanation Why

NSAIDs exert their anti-inflammatory effect by reversibly inhibiting cyclooxygenase, which


decreases synthesis of prostaglandins. Naproxen or indomethacin are commonly used for the
treatment of acute gout attack. Though prolonged use can cause peptic ulcer disease, NSAIDs are not
associated with abdominal cramping, diarrhea, and vomiting shortly after administration. Aspirin is
contraindicated in gout as it increases the risk of recurrent flares by decreasing excretion of uric acid.

C - Nucleus

Explanation Why

Glucocorticoids exert their effect by binding to nuclear receptors (i.e., glucocorticoid receptors) in
the cytosol. The glucocorticoid-receptor complex translocates into the nucleus where it
downregulates pro-inflammatory gene transcription. Glucocorticoids are used to treat acute gout
attack. However, they are not associated with abdominal cramping, diarrhea, and vomiting shortly
after administration.

www.eduwaves360.com | Telegram : @eduwaves360


D - Cytoskeleton

Image

Explanation Why

Colchicine inhibits microtubule polymerization by binding to the cytoskeleton. This leads to


disruption of several inflammatory mechanisms, such as phagocytosis of urate crystals, cell
chemotaxis, leukocyte activation, and migration. Colchicine is used to treat an acute gout attack, but
should be avoided in patients with severe renal or hepatic impairment. Side effects include GI upset
as observed in this patient (severe diarrhea, vomiting, abdominal pain), myelosuppression, and hair
loss.

www.eduwaves360.com | Telegram : @eduwaves360


E - Membrane transporter

Explanation Why

Probenecid inhibits reabsorption of uric acid in the kidney by blocking tubular urate transporters.
Although probenecid is a second-line treatment for hyperuricemia associated with chronic gouty
arthritis, it is not used for the treatment of acute gout attack. Moreover, it is not associated with
abdominal cramping and diarrhea shortly after administration.

www.eduwaves360.com | Telegram : @eduwaves360


Question # 10

A 47-year-old woman comes to the physician because of body aches for the past 9 months. She also has
stiffness of the shoulders and knees that is worse in the morning and tingling in the upper extremities.
Examination shows marked tenderness over the posterior neck, bilateral mid trapezius, and medial
aspect of the left knee. A complete blood count and erythrocyte sedimentation rate are within the
reference ranges. Which of the following is the most likely diagnosis?

Answer Image

A Polymyositis

B Major depressive disorder

C Axial spondyloarthritis

D Fibromyalgia

www.eduwaves360.com | Telegram : @eduwaves360


Answer Image

E Polymyalgia rheumatica

F Systemic lupus erythematosus

G Rheumatoid arthritis

www.eduwaves360.com | Telegram : @eduwaves360


Hint

Dysregulation of the neuroendocrine and autonomic nervous systems is thought to play a role in the
pathogenesis of this condition.

www.eduwaves360.com | Telegram : @eduwaves360


Correct Answer

A - Polymyositis

Explanation Why

Polymyositis predominantly affects the proximal muscles, can cause muscle tenderness, and is more
common among women. However, it would also cause muscle weakness, which is not reported by
this patient, and the ESR is typically increased in patients with polymyositis.

B - Major depressive disorder

Explanation Why

Depression is a risk factor for this patient's current condition but she does not meet the diagnostic
criteria for major depressive disorder, which includes > 5 SIGECAPS symptoms, one of which must
be depressed mood or anhedonia.

www.eduwaves360.com | Telegram : @eduwaves360


C - Axial spondyloarthritis

Image

Explanation Why

Axial spondyloarthritis (ankylosing spondylitis) can manifest with morning stiffness but more
commonly causes tenderness over the sacroiliac joints. In addition, patients with this condition
typically have an elevated ESR.

www.eduwaves360.com | Telegram : @eduwaves360


D - Fibromyalgia

Image

Explanation Why

Fibromyalgia, a condition that most commonly develops among women 20–50 years of age, is
characterized by stiffness and tenderness at the entheses of certain sites such as the neck, back,
shoulders, and medial aspect of the knees (“tender points” of fibromyalgia). Other features that may
be present in patients with fibromyalgia include paresthesias, headache, fatigue, unrefreshing sleep,
cognitive dysfunction (“fibro fog”), and autonomic dysfunction. CBC and ESR are typically normal
in patients with this condition. Fibromyalgia is associated with comorbidities such as tension
headaches, sleep disturbances, anxiety disorder, chronic fatigue syndrome, depression, and irritable
bowel syndrome. First-line therapies for fibromyalgia include nonpharmacologic interventions such
as patient education, dietary changes, sleep hygiene, and regular physical activity.

www.eduwaves360.com | Telegram : @eduwaves360


E - Polymyalgia rheumatica

Image

Explanation Why

Polymyalgia rheumatica most commonly develops among middle-aged and elderly women and
would result in symmetric pain and stiffness of the shoulder, pelvic girdle, and neck. However, fever,
weight loss, and an increased ESR are also usually present in patients with polymyalgia rheumatica.

F - Systemic lupus erythematosus

Explanation Why

Systemic lupus erythematosus manifests in middle-aged women and can cause myalgia, joint
stiffness, as well as tingling due to peripheral neuropathy. However, patients with SLE would often
have other symptoms such as fever, fatigue, photosensitivity, and arthralgia, and typically have an
elevated ESR and leukopenia, thrombocytopenia, or anemia on a CBC.

www.eduwaves360.com | Telegram : @eduwaves360


G - Rheumatoid arthritis

Explanation Why

Rheumatoid arthritis (RA) can manifest with pain and morning stiffness, but the distribution of this
patient's musculoskeletal symptoms (involvement of the neck, back, shoulders, and knees with
sparing of the distal joints) is not typical of RA, which usually affects the metacarpophalangeal joints
and proximal interphalangeal joints of the fingers first. In addition, patients with RA typically have
an increased ESR.

www.eduwaves360.com | Telegram : @eduwaves360


Question # 11

A 58-year-old man comes to the physician because of a 6-month history of headaches and back pain.
Examination shows mild sensorineural hearing loss. Serum concentration of alkaline phosphatase is
increased. An x-ray of the skull is shown. The most appropriate pharmacotherapy for this patient is a
drug that has which of the following mechanisms of action?

Answer Image

A Inhibition of tubulin polymerization

B Inhibition of nuclear factor-κB

C Formation of DNA strand breaks

www.eduwaves360.com | Telegram : @eduwaves360


Answer Image

D Stimulation of osteoblast activity

E Inhibition of proteasomes

F Apoptosis of osteoclasts

www.eduwaves360.com | Telegram : @eduwaves360


Hint

The combination of hearing loss, headaches, and an elevated alkaline phosphatase concentration is
concerning for a diagnosis of Paget disease of bone. The x-ray showing characteristic deformities of
bone including thickening of the calvaria, sclerotic and osteolytic lesions, and osteoblastic lesions
confirms the diagnosis.

www.eduwaves360.com | Telegram : @eduwaves360


Correct Answer

A - Inhibition of tubulin polymerization

Explanation Why

Inhibition of tubulin polymerization occurs after administration of vinca alkaloids or colchicine.


Vinca drugs are potent antineoplastic agents and colchicine is used in gout. Neither are indicated in
this patient.

B - Inhibition of nuclear factor-κB

Explanation Why

Inhibition of nuclear factor-κB is the mechanism of denosumab and glucocorticoids. Denosumab is


most often used in the treatment of osteoporosis, whereas glucocorticoids can be used as an
intraarticular injection to treat osteoarthritis. Osteoarthritis and vertebral compression fractures due
to osteoporosis can cause back pain but this patient's imaging findings, increased ALP, and new
onset hearing loss are consistent with neither osteoporosis nor osteoarthritis.

C - Formation of DNA strand breaks

Explanation Why

Formation of DNA strand breaks is the mechanism by which various antineoplastic drugs, such as
bleomycin, cladribine, and doxorubicin, function. Chemotherapy does not have a role in treating
Paget disease of bone.

www.eduwaves360.com | Telegram : @eduwaves360


D - Stimulation of osteoblast activity

Explanation Why

Stimulation of osteoblasts is the goal of therapy with teriparatide in patients with osteoporosis. This
drug is a parathyroid hormone (PTH) analog but has an apparently paradoxical effect when given in
a pulsatile fashion compared to the continuous release of PTH under physiologic conditions.
Advanced osteoporosis can cause back pain due to compression fractures but osteoporosis would not
cause headaches, sensorineural hearing loss, increased ALP, or the imaging findings seen in this
patient.

E - Inhibition of proteasomes

Explanation Why

Inhibition of proteasomes is the mechanism of bortezomib, which is most commonly used in treating
multiple myeloma. The patient described here presents with chronic back pain, headache, and
osteolytic lesions, all of which could be consistent with multiple myeloma. However, this patient's
skull imaging with diffuse sclerosis and calavarial thickening is not consistent with this diagnosis.

F - Apoptosis of osteoclasts

Explanation Why

The mainstay of treatment for Paget disease of bone is bisphosphonate therapy, which functions by
inducing apoptosis of osteoclasts. These drugs bind directly to the bone surface and are eventually
taken up by osteoclasts during bone resorption. Once within a cell, they interfere with osteoclast
survival, leading to apoptosis and reduced bone resorption.

www.eduwaves360.com | Telegram : @eduwaves360


Question # 12

A 62-year-old man with prostate cancer comes to the physician because of low back pain for 2 weeks
and a 4.5-kg (10-lb) weight loss. Physical examination shows localized tenderness over the lumbar
spine. An x-ray of the lumbar spine shows several osteoblastic lesions at the level of L2 and L4
vertebrae. Microscopic examination of a bone biopsy specimen from the L4 vertebra shows irregular
bone trabeculae and star-shaped cells with long, cytoplasmic processes located deep within the lacunae.
Exchange of nutrients and waste products between these cells most likely occurs through which of the
following structures?

Answer Image

A Macula adherens

B Macula communicans

C Zonula occludens

D Hemidesmosomes

E Zonula adherens

www.eduwaves360.com | Telegram : @eduwaves360


Hint

Star-shaped cells within the bone canaliculi are osteocytes. They are the mature forms of osteoblasts that
have become trapped within the lacunae and bone canaliculi.

www.eduwaves360.com | Telegram : @eduwaves360


Correct Answer

A - Macula adherens

Explanation Why

Macula adherens, also known as desmosomes, primarily function to provide structural support for
cell-to-cell adhesion. This is especially critical for the maintenance of tissues that experience strong
mechanical stressors, such as epithelia, bladder tissue, and cardiac muscle. Desmosomes do not
facilitate intercellular exchange or communication.

B - Macula communicans

Explanation Why

Macula communicans, also known as gap junctions, are cell-to-cell channels that allow a rapid
exchange of ions, nutrients, and other molecules. Gap junctions are composed of two connexons that
connect adjacent cells and create an intercellular network that spans osteocytes, osteoclasts,
osteoblasts, and other progenitor cells that reside in bone.

C - Zonula occludens

Explanation Why

Zonula occludens (also known as tight junctions) are extracellular proteins that seal the paracellular
space and prevent passive paracellular movement of molecules. Comprised of claudin and occludin
proteins, these structures are prominent in the epithelial layer of the gut. These cells do not provide a
path for intercellular exchange of nutrients.

www.eduwaves360.com | Telegram : @eduwaves360


D - Hemidesmosomes

Explanation Why

Hemidesmosomes are multiprotein structures that connect epithelial basal cells to the basement
membrane (basal lamina). Although hemidesmosomes can be involved in signaling pathways (e.g.,
epithelial migration), they are not permeable and therefore do not facilitate intercellular exchange of
molecules.

E - Zonula adherens

Explanation Why

Zonula adherens, also known as adherens junctions, are strong structures that exist in the paracellular
space between epithelial and endothelial cells. These structures lie just more basal than tight
junctions (i.e., closer to the basement membrane). They primarily function to anchor intracellular
actin microfilaments to the cellular membrane. Zonula adherens are not permeable and do not
facilitate the exchange of nutrients or molecules.

www.eduwaves360.com | Telegram : @eduwaves360


Question # 13

A 37-year-old woman undergoes diagnostic laparoscopy under general anesthesia for evaluation of
chronic pelvic pain. Postoperatively, the patient requires prolonged intubation. Neostigmine is
administered. Results of acceleromyography during train-of-four ulnar nerve stimulation are shown.
Which of the following drugs is most likely to have been used preoperatively in this patient?

Answer Image

A Clonazepam

B Ropivacaine

www.eduwaves360.com | Telegram : @eduwaves360


Answer Image

C Rocuronium

D Tizanidine

E Succinylcholine

www.eduwaves360.com | Telegram : @eduwaves360


Hint

Decreased muscle contraction after administration of an acetylcholinesterase inhibitor (e.g.,


neostigmine) suggests that the patient was in phase I neuromuscular blockade when neostigmine was
administered.

www.eduwaves360.com | Telegram : @eduwaves360


Correct Answer

A - Clonazepam

Explanation Why

Clonazepam is a long-acting benzodiazepine that is not used in general anesthesia. However, short-
acting benzodiazepines (e.g., midazolam) could be administered for pre-operative sedation.

B - Ropivacaine

Explanation Why

Ropivacaine is an amide and local anesthetic commonly used for local anesthesia or a regional block
(e.g., brachial plexus block). It is not used as a muscle relaxant for general anesthesia.

www.eduwaves360.com | Telegram : @eduwaves360


C - Rocuronium

Image

Explanation Why

Rocuronium is a non-depolarizing neuromuscular agent that causes a neuromuscular blockade.


Electromyography would show a fade-off with train-of-four stimulation during recovery. However,
during recovery, the electromyography in this patient shows the same amplitude of muscle
contraction following all 4 muscle stimuli. Furthermore, after administration of neostigmine, there
would be an increase in the muscle contraction amplitude rather than the decrease seen here.

D - Tizanidine

Explanation Why

Tizanidine is a centrally-acting muscle relaxant. It is used in the treatment of diseases associated with
spasticity (e.g., multiple sclerosis), but it is not used for muscle relaxation prior to intubation.

www.eduwaves360.com | Telegram : @eduwaves360


E - Succinylcholine

Image

Explanation Why

Use of the depolarizing muscle relaxant succinylcholine is characterized by 2 phases of


neuromuscular blockade. Phase I block is caused by persistent depolarization of the acetylcholine
receptors of the neuromuscular junction. During this phase, administration of an acetylcholinesterase
inhibitor (e.g., neostigmine) potentiates the depolarizing effect of succinylcholine at the
neuromuscular junction, as evidenced by the diminishing amplitude in this patient's repeated train-of-
four monitoring. Administration of a large dose of succinylcholine can cause phase II blockade.
However, a phase II block is characterized by a fade-off in amplitude with train-of-four stimulation
and the use of an acetylcholinesterase inhibitor in phase II would either have no effect or reverse the
effect of succinylcholine. Succinylcholine has a rapid onset of action and is typically degraded
completely in 5–10 minutes by pseudocholinesterase. Some individuals are homozygous for an
atypical pseudocholinesterase, which breaks down succinylcholine more slowly. In these patients,
paralysis can last for hours and they must be maintained on mechanical ventilation until spontaneous
respirations resumes.

www.eduwaves360.com | Telegram : @eduwaves360


Question # 14

A 15-year-old boy comes to the physician for the evaluation of progressive difficulty climbing stairs for
the last 2 years. During this period, he has also had problems running and standing up from a seated
position. He is at the 50th percentile for height and weight. Examination shows enlarged calf muscles
bilaterally and a waddling gait. Which of the following is the most likely cause of this patient's
condition?

Answer Image

A Nonsense mutation

B In-frame deletion

C Trinucleotide repeat expansions

D Splice site mutation

E Frameshift mutation

www.eduwaves360.com | Telegram : @eduwaves360


Hint

Proximal muscle weakness in the lower extremities (difficulty climbing stairs, standing from a seated
position, waddling gait), bilateral calf pseudohypertrophy, and onset of symptoms during adolescence is
highly suggestive of Becker muscular dystrophy.

www.eduwaves360.com | Telegram : @eduwaves360


Correct Answer

A - Nonsense mutation

Explanation But

As certain nonsense mutations can induce exon skipping and thereby produce various phenotypes,
nonsense mutations within exon 1 or 27 have indeed been associated with a BMD phenotype.
However, these mutations are exceptions, and another mechanism is much more likely in this patient.

Explanation Why

Nonsense mutations are point mutations that cause insertion of a premature stop codon, which results
in a nonfunctional protein or gene product. About 10%–15% of cases of Duchenne muscular
dystrophy (DMD) are due to a nonsense mutation in the dystrophin gene. The disease typically
manifests in boys with symptoms identical to BMD such as progressive muscle weakness and calf
pseudohypertrophy, which are seen in this patient. However, symptoms manifest during infancy or
early childhood and are more severe than in BMD.

B - In-frame deletion

Explanation But

Genetic analysis for mutations in the dystrophin gene is the test of choice to confirm the diagnosis of
dystrophinopathies.

Explanation Why

Becker muscular dystrophy (BMD) is an X-linked recessive disorder that is most commonly caused
by an in-frame deletion in the dystrophin gene, which leads to a partially functional protein. In in-
frame mutations, multiples of three nucleotides (i.e., complete codons) are added or deleted, leaving
the reading frame intact. The second most common cause of BMD is a missense mutation, where a
single nucleotide is substituted. BMD usually manifests in boys during adolescence or early
adulthood with slow, progressive muscle weakness and calf pseudohypertrophy, as seen in this
patient. Symptoms are typically less severe and appear later than in patients with Duchenne muscular
dystrophy.

www.eduwaves360.com | Telegram : @eduwaves360


C - Trinucleotide repeat expansions

Explanation Why

Trinucleotide repeat expansions cause a variety of diseases including myotonic dystrophy (MD, CTG
repeats) and Friedreich ataxia (FA, GAA repeats). MD and FA both manifest with progressive
weakness. However, MD is associated with muscle pain and atrophy (not pseudohypertrophy), and
patients often develop cardiac arrhythmias, testicular atrophy, frontal baldness, and/or cataracts,
which are not seen this patient. While FA also manifests with gait difficulties, this is due to a spastic
paresis that affects the entire lower leg, leading to a wide-based, staggering gait, which is not present
in this patient. Moreover, one would expect an onset of symptoms during childhood and different
findings, including nystagmus, diabetes mellitus, and skeletal deformities. FA does not cause calf
pseudohypertrophy.

D - Splice site mutation

Explanation Why

Splice site mutations are associated with a variety of diseases including cancers, dementia, epilepsy,
and some types of β-thalassemia. However, they are not an underlying cause of muscular dystrophy.

E - Frameshift mutation

Explanation Why

Frameshift mutations cause insertion or deletion of nucleotides, which changes the originally-
encoded sequence of triplets and thus shifts the way it is read during translation. X-linked recessive
frameshift mutations of the dystrophin gene are a common cause of Duchenne muscular dystrophy
(DMD). The disease typically manifests in boys with symptoms identical to BMD such as
progressive muscle weakness and calf pseudohypertrophy, which are seen in this patient. However,
symptoms in DMD manifest during infancy or early childhood and are more severe than symptoms
in BMD.

www.eduwaves360.com | Telegram : @eduwaves360


Question # 15

A 19-year-old man comes to the physician because of episodic tingling and numbness in his right hand
for the past 3 weeks. He says that he cannot hold a pen very well and that this is affecting his
handwriting. The symptoms started after he played in a football game. Physical examination shows
decreased pinch strength in the right hand. Sensation to light touch is decreased over the right little
finger and the medial palmar surface of the right hand. Radiologic evaluation in this patient is most
likely to show a fracture in which of the following bones?

Answer Image

A A

B B

C C

www.eduwaves360.com | Telegram : @eduwaves360


Answer Image

D D

E E

F F

G G

www.eduwaves360.com | Telegram : @eduwaves360


Hint

Decreased pinch strength and sensory deficits over the little finger and palmar surfaces of the medial
aspect of the right hand are suggestive of ulnar nerve entrapment at the level of the wrist.

www.eduwaves360.com | Telegram : @eduwaves360


Correct Answer

A-A

Explanation Why

Injury to the capitate bone may occur after falling onto an outstretched hand. However, isolated
fractures are very rare (3% of all carpal fractures) and patients typically present with pain on the
dorsum of the wrist or proximal hand. Neurovascular compromise is possible with dislocation and
typically affects the median nerve; the ulnar nerve is very unlikely to be affected.

B-B

Explanation Why

Trapezoid bone fractures usually occur as a result of high energy trauma (e.g., when a player dives
for an opponent with an outstretched arm). However, they are overall very rare (< 2% of all carpal
fractures) and typically present with radial-dorsal wrist pain. There may be involvement of the radial
and median nerve, but the ulnar nerve is very unlikely to be affected.

C-C

Explanation Why

Injury to the trapezium bone is rare (only ∼4% of all carpal fractures) but can occur from axial
loading or hyperextension of an adducted thumb. Trapezium bone fracture could cause injury to the
median nerve, but the ulnar nerve would be unlikely to be affected.

www.eduwaves360.com | Telegram : @eduwaves360


D-D

Explanation Why

Scaphoid bone fractures are common (60-70% of all carpal fractures) and can occur following a fall
onto the outstretched hand (hyperextension). Scaphoid fractures can involve the radial artery
(positive Allen test), but nerve injury is uncommon.

E-E

Explanation Why

As the lunate bone lies well protected in the proximal row of carpal bones, fractures are very rare
(only ∼4% of all carpal bone injuries). Injury due to a fall onto the extended wrist (hyperextension
injury) could cause median nerve injury, but ulnar nerve injury would be very unlikely.

F-F

Explanation Why

Fracture of the pisiform bone can occurs from a fall onto the outstretched hand when the wrist is
extended, or when the heel of the hand is used to strike or swing an object (e.g., baseball bat, golf
club, or racquet). The pisiform bone marks the ulnar border of Guyon's canal, and a fracture could
thus result in injury to the ulnar nerve. However, fractures to the pisiform bone are extremely rare (<
1% of carpal bone fractures) and so fracture to a different bone is much more likely to be the cause
of his symptoms.

www.eduwaves360.com | Telegram : @eduwaves360


G-G

Image

Explanation Why

Blunt trauma to the hook of hamate (e.g., fall on an outstretched hand) can lead to entrapment of the
ulnar nerve, which is referred to as a distal ulnar lesion. In addition to paresthesias, numbness,
weakness, and pain, this lesion can also cause claw hand deformity.

www.eduwaves360.com | Telegram : @eduwaves360


Question # 16

A 36-year-old woman comes to the emergency department because of left wrist pain and swelling that
started immediately after a fall. She was walking her dog when the dog abruptly pulled away, causing
her to fall forward onto both hands. Physical examination shows left wrist pain with pinching and
grasping, moderate swelling, and mild tenderness. Her left thumb is placed in a thumb immobilization
splint. An x-ray of the wrist 2 days later is shown. The most likely cause of these findings is a fracture of
which of the following bones?

Answer Image

A Scaphoid

B Ulna

C Radius

www.eduwaves360.com | Telegram : @eduwaves360


Answer Image

D Lunate

E Hamate

www.eduwaves360.com | Telegram : @eduwaves360


Hint

This type of fracture is typically associated with tenderness to palpation of the anatomical snuffbox.

www.eduwaves360.com | Telegram : @eduwaves360


Correct Answer

A - Scaphoid

Explanation Why

The scaphoid is the most commonly fractured carpal bone. Fracture results from an axial load injury
onto a hyperextended and radially deviated wrist and manifests with mild wrist pain, swelling, and
painful pinching and grasping, as seen in this patient. Although many scaphoid fractures can be
treated conservatively with immobilization, proximal pole fractures carry a high risk of avascular
necrosis and often require surgical fixation.

B - Ulna

Explanation Why

A fracture of the ulna is often caused by a direct blow to or a fall onto a pronated forearm. These
fractures result in localized pain, swelling, and often gross deformation and/or loss of range of
motion at the forearm or wrist. Although this patient has wrist pain and swelling, the mechanism of
injury makes an ulnar fracture less likely. Furthermore, the x-ray does not show an ulnar fracture.

C - Radius

Explanation Why

A fracture of the distal radius (e.g., Colles fracture) can be caused by an axial load injury onto a
hyperextended wrist, particularly in patients with osteoporosis or those who have sustained a high-
energy trauma. Distal radius fractures manifest with wrist pain, swelling, and grasp deficits, which
are seen in this patient. However, this patient's young age and low-energy mechanism of injury make
a Colles fracture less likely. Furthermore, the x-ray shows no fracture at the distal radius.

www.eduwaves360.com | Telegram : @eduwaves360


D - Lunate

Explanation Why

Lunate fractures and peri-lunate dislocations are rare injuries resulting from high-energy trauma to a
hyperextended and ulnar deviated wrist. Patients present with wrist pain, swelling, and occasionally
median nerve injuries such as acute carpal tunnel syndrome. Although this patient has wrist pain and
swelling after an axial load injury, the low-energy mechanism of injury makes a lunate fracture or
dislocation less likely. Moreover, the x-ray is inconsistent with a lunate fracture.

E - Hamate

Explanation Why

A fracture of the hamate body is a rare carpal bone injury that is usually sustained from direct axial
load onto a clenched wrist. It manifests with wrist swelling, ulnar wrist pain, and tenderness over the
hamate. Although this patient has wrist pain and swelling after an axial load injury, the fact that her
injury involved hyperextension make a hamate fracture less likely. Moreover, the x-ray shows no
fracture at the hamate.

www.eduwaves360.com | Telegram : @eduwaves360


Question # 17

A 25-year-old woman comes to the physician because of pain and weakness in her right forearm and
hand for several months. Two years ago, she sustained a fracture of her ulnar shaft with dislocation of
the radial head that was treated surgically. Physical examination shows mild tenderness a few
centimeters distal to the lateral epicondyle. She has marked weakness when attempting to extend her
right middle finger. There is radial deviation on extension of the wrist. Sensation is not impaired. Which
of the following nerves is most likely affected in this patient?

Answer Image

A Ulnar nerve

B Anterior interosseous nerve

C Superficial radial nerve

D Posterior interosseous nerve

www.eduwaves360.com | Telegram : @eduwaves360


Answer Image

E Musculocutaneous nerve

www.eduwaves360.com | Telegram : @eduwaves360


Hint

This patient's weakness in her wrist extensor muscles with intact cutaneous sensation suggests a pure
motor neuropathy that is sometimes associated with dislocation of the radial head.

www.eduwaves360.com | Telegram : @eduwaves360


Correct Answer

A - Ulnar nerve

Image

Explanation Why

Injuries to the ulnar nerve can manifest in a number of ways depending on the location of the lesion.
The vast majority of injuries, however, include sensory defects in the small and ring fingers, as well
as weakness in the intrinsic hand muscles, neither of which are seen in this patient. Additionally,
patients with ulnar nerve dysfunction are unable to cross their fingers and may have an ulnar claw
sign on examination.

www.eduwaves360.com | Telegram : @eduwaves360


B - Anterior interosseous nerve

Explanation Why

The anterior interosseous nerve (AIN) is a motor branch of the median nerve. Damage to it would
not result in changes in cutaneous sensation, which is consistent with this patient's symptoms.
However, weakness with finger extension would not be expected in an injury to this nerve, either.
The classic physical examination finding in patients with AIN injuries is an inability to make the “A-
OK” sign with the thumb and index finger, although injury to this branch alone is rare.

C - Superficial radial nerve

Explanation Why

The superficial radial nerve is a distal branch of the radial nerve. It is a purely sensory nerve and
provides cutaneous innervation to the radial half of the dorsal hand, the thumb, the index finger, and
the radial half of the dorsal long finger (except for the tips of the fingers, which are innervated by the
median nerve). However, this patient has no cutaneous sensory symptoms. Furthermore, injury to
this nerve would not account for her motor changes.

D - Posterior interosseous nerve

Explanation But

The posterior interosseous nerve can also be compressed within the radial tunnel (radial tunnel
syndrome).

Explanation Why

The posterior interosseous nerve is a motor nerve that branches off from the radial nerve in close
proximity to the radial head, which makes it susceptible to injury due to radial head dislocation or
subluxation. Occasionally, posterior interosseous neuropathy can develop years after a fracture of the
forearm or elbow. Compression of this nerve frequently impairs extension of the fingers (more than

www.eduwaves360.com | Telegram : @eduwaves360


the wrist) because function of the extensor carpi radialis longus, which is innervated directly by the
radial nerve, is preserved. The unopposed action of this muscle further explains the radial deviation
seen on examination. Finally, this patient's normal sensory examination is one of the hallmarks of
posterior interosseous neuropathy because this nerve does not provide cutaneous innervation.

E - Musculocutaneous nerve

Image

Explanation Why

The musculocutaneous nerve is responsible for elbow flexion, forearm supination, and sensation over
the lateral, volar forearm. None of these functions are affected in this patient.

www.eduwaves360.com | Telegram : @eduwaves360


Question # 18

A 68-year-old woman with osteoarthritis comes to the physician because of a swollen and painful right
knee for the past 2 days. Temperature is 37°C (98.6°F). Examination shows erythema and swelling of
the right knee with a normal range of motion. An x-ray of the right knee shows punctate radiodensities
in both menisci and in the joint capsule. Arthrocentesis of the right knee joint yields 5 mL of cloudy
fluid with a leukocyte count of 27,000/mm3. Which of the following is the most likely underlying
mechanism of this patient's knee pain?

Answer Image

Calcium pyrophosphate dihydrate


A
crystal deposition

B Immune complex deposition

C Noncaseating granuloma formation

D Gram-negative diplococci infection

Monosodium urate crystal


E
precipitation

www.eduwaves360.com | Telegram : @eduwaves360


Hint

The radiodensities in the menisci and joint capsule on x-ray are caused by chondrocalcinosis.

www.eduwaves360.com | Telegram : @eduwaves360


Correct Answer

A - Calcium pyrophosphate dihydrate crystal deposition

Image

Explanation Why

This patient has pseudogout, which is caused by calcium pyrophosphate dihydrate crystal deposition
within joints leading to episodes of acute inflammatory arthritis. This condition is most common
among older adults and often manifests with monoarthritis of the knee that causes swelling, warmth,
and tenderness. An x-ray would often show calcium deposits in the cartilage (chondrocalcinosis) as
seen here. Arthrocentesis is the gold standard for establishing the diagnosis; examination of synovial
fluid typically shows an elevated leukocyte count and rhomboid crystals that are weakly birefringent
under polarized light. Certain conditions such as joint trauma, familial chondrocalcinosis,
hyperparathyroidism, and hemochromatosis are associated with pseudogout but in most cases, the
cause of pseudogout is unknown (primary CPPD disease). Acute treatment for pseudogout includes
NSAIDs and intra-articular corticosteroids. Colchicine can be used for prophylaxis.

www.eduwaves360.com | Telegram : @eduwaves360


B - Immune complex deposition

Explanation Why

Activation of T cells in response to immune complex deposition occurs in patients with rheumatoid
arthritis (RA). Though arthrocentesis of affected joints also can also yield cloudy fluid with an
elevated leukocyte count (5,000–50,0000/mm3), RA clinically manifests with polyarthritis, morning
stiffness and eventual joint deformities (not acute, monoarticular arthritis). Moreover, an x-ray of the
affected joint in rheumatoid arthritis would typically show joint space narrowing, juxta-articular
osteopenia, bony erosions, and subchondral cysts, not the abnormal punctate radiodensities seen in
this case.

C - Noncaseating granuloma formation

Explanation Why

Noncaseating granuloma formation is the pathological hallmark of sarcoidosis. Though an acute


presentation of sarcoidosis (Löfgren syndrome) can also cause arthritis, this condition typically
manifests with symmetrical and migratory polyarthritis (not monoarthritis) and the abnormal
punctate radiodensities that are seen here would not be expected. The typical features of sarcoidosis
include fever, uveitis, lupus pernio, erythema nodosum, interstitial lung disease, and bilateral hilar
lymphadenopathy.

D - Gram-negative diplococci infection

Explanation Why

Neisseria gonorrhoeae is a gram-negative diplococcus that can cause acute, monoarticular arthritis
that manifests with a painful, swollen knee as seen in this patient. However, arthrocentesis of septic
arthritis typically results in fever and a leukocyte count > 50,000/mm3. In addition, N. gonorrhoeae
arthritis does not cause the abnormal punctate radiodensities seen in this case.

www.eduwaves360.com | Telegram : @eduwaves360


E - Monosodium urate crystal precipitation

Explanation Why

Monosodium urate crystals deposit within the joints in patients with gout. This condition typically
manifests with podagra, but it can also cause tenderness, swelling, and erythema in other joints such
as the knee as seen in this patient. However, gout does not cause the abnormal punctate
radiodensities seen in this case.

www.eduwaves360.com | Telegram : @eduwaves360


Question # 19

A 42-year-old man comes to the physician for 1 month of worsening right knee pain. He has not had any
trauma other than stubbing his toe 3 days ago at the garage where he works as a mechanic. Examination
of the right knee shows swelling and erythema with fluctuance over the inferior patella. There is
tenderness on palpation of the patella but no joint line tenderness or warmth. The range of flexion is
limited because of the pain. Which of the following is the most likely underlying cause of this patient's
symptoms?

Answer Image

A Inflammation of the patellar tendon

B Noninflammatory degeneration of the joint

C Infection of the joint

D Deposition of crystals in the joint

E Inflammation of periarticular fluid-filled sac

www.eduwaves360.com | Telegram : @eduwaves360


Hint

This condition commonly occurs in individuals with occupations that require excessive kneeling.

www.eduwaves360.com | Telegram : @eduwaves360


Correct Answer

A - Inflammation of the patellar tendon

Image

Explanation But

Inflammation of the insertion of the patellar tendon onto the tibial tuberosity and its subsequent
disruption (apophysitis) is the cause of the pediatric condition Osgood-Schlatter disease.

Explanation Why

Inflammation of the patellar tendon causes patellar tendonitis. This condition is most often the result
of overuse (especially with running and jumping activities) and manifests with progressive anterior
knee pain and swelling, similar to that described by this patient. However, the presence of a fluctuant
collection overlying the patella is inconsistent and argues strongly against this diagnosis.

www.eduwaves360.com | Telegram : @eduwaves360


B - Noninflammatory degeneration of the joint

Explanation Why

Noninflammatory degeneration of the joint is seen in osteoarthritis. This is most often attributed to
old age or chronic overuse of the joint. Although patients with osteoarthritis can also present with
unilateral knee pain, they generally do not have appreciable fluctuance over the prepatellar bursa.
Additionally, osteoarthritis would be unlikely in a 42-year-old patient unless there was a history of
significant trauma to the knee.

C - Infection of the joint

Explanation Why

Infection of the joint is the underlying cause of septic arthritis, which usually manifests as
monoarticular pain and swelling, as seen in this patient. However, patients with septic arthritis also
have systemic signs of infection such as fever. Additionally, septic arthritis has an acute course and
rarely evolves over the course of one month. Finally, because joint infection in septic arthritis occurs
via hematogenous spread or direct contamination, most patients will have either a concomitant
infection (e.g. endocarditis or abscess) or significant trauma to the affected joint, neither of which are
present.

D - Deposition of crystals in the joint

Explanation Why

Deposition of uric acid crystals in the joint can cause acute gout attacks, which manifest as intense
pain and inflammation in the big toe (podagra), knee, fingers, or wrists. Acute gout attacks usually
become maximally painful over the span of a few hours, not one month. In addition, gout usually
causes warmth over the affected joint, which is not seen in this patient.

www.eduwaves360.com | Telegram : @eduwaves360


E - Inflammation of periarticular fluid-filled sac

Explanation But

Surgery may be indicated (e.g., drainage of pus, bursectomy) for recurrent bursitis if conservative
management fails.

Explanation Why

Prepatellar bursitis is an inflammation of the synovial bursa located between the skin and patella. It
is often caused by overuse injuries or repeated trauma to the knee. Professions which require
frequent kneeling (e.g., carpet installers, masons, plumbers, mechanics) are especially prone to
developing prepatellar bursitis. The condition most often manifests with localized swelling,
tenderness on palpation and with range of motion, and a palpable fluid collection (the inflamed
bursa), all of which are observed in this patient; fever, warmth, or spreading erythema are
characteristically absent.

www.eduwaves360.com | Telegram : @eduwaves360


Question # 20

A 35-year-old man comes to the physician because of a 3-month history of intermittent right lateral hip
pain that radiates to the thigh. Climbing stairs and lying on his right side aggravates the pain.
Examination shows tenderness to palpation over the upper lateral part of the right thigh. There is no
swelling. When the patient is asked to resist passive adduction of the right leg, tenderness is noted. An
x-ray of the pelvis shows no abnormalities. Which of the following structures is the most likely source
of this patient's pain?

Answer Image

A Iliotibial band

B Acetabulum

C Lateral femoral cutaneous nerve

D Lumbar nerve roots

E Femoral head

F Greater trochanter

www.eduwaves360.com | Telegram : @eduwaves360


Hint

Lateral hip tenderness that is exacerbated by lying on the affected side and triggered by resisted
adduction is consistent with tendinopathy involving the area where the gluteus muscles attach to the hip.

www.eduwaves360.com | Telegram : @eduwaves360


Correct Answer

A - Iliotibial band

Explanation Why

Iliotibial band syndrome typically manifests with lateral knee pain rather than hip pain. Moreover,
iliotibial band syndrome is an overuse injury that occurs mostly in runners and cyclists.

B - Acetabulum

Explanation Why

Femoroacetabular impingement occurs when the femoral head rubs abnormally against the
acetabulum, which can manifest with insidious groin pain and lateral hip pain. However, hip
tenderness on palpation is usually absent. Classical findings of the condition, such as pain with
flexion, adduction, and internal rotation of the hip (positive impingement test), or limitation of
internal rotation and/or flexion of the hip, are also not seen in this patient. Moreover, the x-ray would
show a deep acetabular socket, acetabular retroversion, or a non-spherical femoral head, with or
without evidence of labral tears and osteoarthritis.

C - Lateral femoral cutaneous nerve

Explanation Why

Compression of the lateral femoral cutaneous nerve can cause meralgia paresthetica, which manifests
with outer thigh pain that is aggravated by climbing stairs and a normal pelvic x-ray. However, it is
also typically associated with tingling, burning, or numbness, which are not seen in this patient.
Common risk factors for meralgia paresthetica include diabetes, obesity, pregnancy, trauma, or hip
surgery, none of which are present here.

www.eduwaves360.com | Telegram : @eduwaves360


D - Lumbar nerve roots

Explanation Why

A radiculopathy of the L3 nerve root as a result of an L2/L3 disc herniation can manifest with hip
and thigh pain as well normal findings on x-ray. However, patients would also have weakness of the
hip flexors and/or a history of lower back pain, neither of which is seen here. Moreover, the pain
associated with radiculopathy is typically aggravated by raising the extended leg (a positive straight
leg raise test), not by resisted hip adduction, as is the case here.

E - Femoral head

Explanation Why

Osteonecrosis of the femoral head may also cause hip pain. Osteonecrosis often occurs in patients
who sustained a femoral bone fracture/dislocation or took corticosteroids for a prolonged period,
neither of which is reported in this patient. Moreover, an x-ray would likely show abnormalities such
as osteopenia and microfractures of the subchondral bone, or a crescent sign (a subchondral line of
radiolucency seen in late stages of the disease).

F - Greater trochanter

Explanation But

Treatment measures of greater trochanteric pain syndrome include relative rest, heat pads, NSAIDs,
and stretching and strengthening exercises.

Explanation Why

This patient likely presents with greater trochanteric pain syndrome, which results from tendinopathy
at the attachment of the gluteus medius and/or minimus muscle. It manifests with lateral thigh or hip
pain near the great trochanter. Typical findings include tenderness to palpation over the greater
trochanter, exacerbation of pain when lying on the affected side, and triggering of pain by resisted

www.eduwaves360.com | Telegram : @eduwaves360


adduction, all of which are present in this patient. X-ray is usually normal and essentially rules out
bone involvement.

www.eduwaves360.com | Telegram : @eduwaves360


Question # 21

A 3-year-old boy is brought to the physician by his parents because of clumsiness and multiple falls over
the past 4 months. He started walking at the age of 18 months and could walk up steps by the time he
was 27 months old but now struggles to walk at all without assistance. When standing up from a lying
position, he crawls onto his knees and slowly walks himself up with his hands. There is bilateral calf
enlargement. Analysis of a left calf biopsy specimen from this patient is most likely to show which of
the following?

Answer Image

Perivascular inflammation with muscle


A
fiber ischemia and atrophy

Proliferation of mitochondria within


B
muscle fibers

Degeneration of muscle fibers with


C
fibrofatty replacement

Atrophic muscle fibers interspersed


D
among hypertrophic muscle fibers

Necrotic muscle fibers with lymphocytic


E
infiltrate

www.eduwaves360.com | Telegram : @eduwaves360


Hint

The presence of calf pseudohypertrophy and the Gower sign in this patient are highly suggestive of
Duchenne muscular dystrophy.

www.eduwaves360.com | Telegram : @eduwaves360


Correct Answer

A - Perivascular inflammation with muscle fiber ischemia and atrophy

Explanation Why

Perivascular inflammation with muscle fiber ischemia and atrophy would be consistent with
dermatomyositis., which is typically characterized by proximal and symmetrical muscle weakness,
especially of the pelvic and shoulder girdle. However, in a patient with dermatomyositis, skin
involvement would be expected (e.g., Gottron papules or heliotrope rash), while calf
pseudohypertrophy would not.

B - Proliferation of mitochondria within muscle fibers

Explanation Why

Mitochondrial myopathies are characterized by the appearance of red ragged fibers due to the
proliferation of mitochondria within muscle fibers. As the respiratory chain of enzymes is impaired,
the proliferation of mitochondria acts as a compensatory mechanism. There are different forms of
mitochondrial myopathies, but patients usually present with muscle weakness that is exacerbated by
exertion. Neither the Gower sign nor calf pseudohypertrophy are features of mitochondrial
myopathies.

www.eduwaves360.com | Telegram : @eduwaves360


C - Degeneration of muscle fibers with fibrofatty replacement

Image

Explanation Why

Degeneration of muscle fibers with fibrofatty replacement is the underlying pathology of muscular
dystrophies such as Duchenne muscular dystrophy. Muscle fiber degeneration occurs due to
dystrophin gene mutations, which lead to alterations of the dystrophin protein. Complete impairment
of dystrophin results in disturbance of cellular signaling pathways, which promotes necrosis in these
cells. Subsequently, these muscle cells are replaced with connective and fat tissue and the muscle
appears larger (calf pseudohypertrophy).

D - Atrophic muscle fibers interspersed among hypertrophic muscle fibers

Explanation Why

Atrophic muscle fibers interspersed among hypertrophic muscle fibers would be found in spinal

www.eduwaves360.com | Telegram : @eduwaves360


muscular atrophy. While spinal muscular atrophy can manifest with progressive muscle weakness,
difficulty walking, Gower sign, and calf pseudohypertrophy, a patient with this disease would most
likely also present with fasciculations and varying degrees of bulbar palsy, neither of which are
present.

E - Necrotic muscle fibers with lymphocytic infiltrate

Explanation Why

Necrotic muscle fibers with a lymphocytic infiltrate would be consistent with polymyositis, which is
characterized by proximal and symmetrical muscle weakness, especially of the pelvic and shoulder
girdle. However, the age of onset in polymyositis is usually older than 20 years and calf
pseudohypertrophy would not be expected.

www.eduwaves360.com | Telegram : @eduwaves360


Question # 22

A 62-year-old man comes to the physician because of a 1-day history of dull pain and stiffness of the
right knee. He takes chlorthalidone for hypertension. Physical examination of the right knee shows a
large effusion and mild erythema; range of motion is limited by pain. Arthrocentesis of right knee yields
a cloudy aspirate. Gram stain is negative. Analysis of the synovial fluid shows a leukocyte count of
15,000/mm3 and 55% neutrophils. Microscopic examination of the synovial fluid under polarized light
shows positively birefringent rods and rhomboid crystals. Further evaluation of this patient is most
likely to show which of the following findings?

Answer Image

Thickening of the synovia at the


A
metacarpophalangeal joints

B Calcification of the meniscal cartilage

Narrowing of the knee joint space with


C
subchondral sclerosis

D Elevation of serum uric acid concentration

E Expression of human leukocyte antigen-B27

www.eduwaves360.com | Telegram : @eduwaves360


Answer Image

F Chalky nodules on the external ear

www.eduwaves360.com | Telegram : @eduwaves360


Hint

Examination of the synovial fluid that shows neutrophil-predominant, mild leukocytosis and
rhomboid-shaped, positively birefringent crystals indicates an inflammatory arthritis caused by calcium
pyrophosphate dihydrate deposition, which is caused by pseudogout (calcium pyrophosphate disease).

www.eduwaves360.com | Telegram : @eduwaves360


Correct Answer

A - Thickening of the synovia at the metacarpophalangeal joints

Explanation Why

Synovial thickening at the metacarpophalangeal joints is seen in patients with rheumatoid arthritis
(RA), which can manifest in older patients with pain, stiffness, swelling, and erythema of the knee.
However, patients with RA typically develop polyarthritis rather than monoarthritis. In addition,
though synovial fluid analysis of patients with RA shows sterile fluid with leukocytosis as seen in
this case, it does not show rhomboid-shaped, positively birefringent crystals.

B - Calcification of the meniscal cartilage

Explanation But

Therapies for pseudogout are similar to those for gout and include NSAIDs, colchicine, and intra-
articular corticosteroid injections

Explanation Why

Calcification along the joint lines of cartilage is visible on x-rays of patients with pseudogout, which
is caused by deposition of calcium pyrophosphate dihydrate crystals, particularly in the meniscus
(knee affected in > 50% of cases) and annulus fibrosus of intervertebral discs. Pseudogout typically
occurs idiopathically in patients > 50 years of age. Calcifications are usually found incidentally when
patients develop acute monoarthritis and x-rays are obtained as part of the workup.

C - Narrowing of the knee joint space with subchondral sclerosis

Explanation Why

Knee joint space narrowing with subchondral sclerosis is caused by osteoarthritis, which can present
with knee pain and stiffness. However, synovial fluid analysis of patients with osteoarthritis shows

www.eduwaves360.com | Telegram : @eduwaves360


calcium phosphate apatite crystals, not rhomboid-shaped, positively birefringent crystals. In addition,
osteoarthritis more commonly manifests with chronic symptoms, in contrast to this patient's acute
onset of pain, stiffness, swelling, and erythema of the knee.

D - Elevation of serum uric acid concentration

Image

Explanation Why

Elevated serum uric acid concentration is associated with gout, which can manifest with acute-onset,
monoarticular joint pain and stiffness (particularly among patients who take thiazides such as
chlorthalidone, which can reduce the excretion of uric acid). However, synovial fluid analysis of
patients with gout shows needle-shaped, negatively birefringent monosodium urate crystals, not the
rhomboid-shaped, positively birefringent crystals seen here.

www.eduwaves360.com | Telegram : @eduwaves360


E - Expression of human leukocyte antigen-B27

Explanation Why

Human leukocyte antigen-B27 (HLA-B27) positivity is seen in ∼ 90% of patients with seronegative
spondyloarthropathies, which can manifest with pain, stiffness, swelling, and erythema of the knee.
However, synovial fluid analysis of patients with seronegative spondyloarthropathies is typically
unremarkable.

F - Chalky nodules on the external ear

Image

Explanation Why

Chalky nodules (e.g., bone tophi and soft tissue tophi) on the external ear and soft tissues (especially
articular structures, tendons, and bursas) develop in patients with chronic tophaceous gout. However,
tophi typically only develop after years of poorly controlled chronic gout. In addition, synovial fluid

www.eduwaves360.com | Telegram : @eduwaves360


analysis of patients with gout shows needle-shaped, negatively birefringent, monosodium urate
crystals, not the rhomboid-shaped, positively birefringent crystals seen in this case.

www.eduwaves360.com | Telegram : @eduwaves360


Question # 23

A 36-year-old woman comes to the physician because of a 2-month history of progressively worsening
lower back pain and weakness in the lower extremities. The pain is worse with movement and improves
with lying down on a flat surface. She was diagnosed with pulmonary tuberculosis 6 months ago and is
currently taking isoniazid and rifampin. Physical examination shows sensory loss over the anterolateral
aspect of the lower-thigh, patella, and medial aspect of the right lower leg. Strength is 2/5 with knee
extension and the patellar reflex is absent. An x-ray of the spine shows a paravertebral opacity with
anterior wedging of the vertebral body. Which of the following nerve roots is most likely to be affected
in this patient?

Answer Image

A L5

B S2

C S1

D L3

E L4

www.eduwaves360.com | Telegram : @eduwaves360


Hint

A history of radiculopathy symptoms in a patient with primary tuberculosis and x-ray findings of
anterior wedging of the vertebral bodies and paravertebral opacity is suggestive of spinal tuberculosis
(Pott disease).

www.eduwaves360.com | Telegram : @eduwaves360


Correct Answer

A - L5

Explanation Why

L5 radiculopathy typically manifests with diminished sensation extending diagonally from the
dorsolateral thigh over the lateral knee and the anterior leg to the dorsomedial side of the foot,
including the medial toes. This pattern is inconsistent with this patient's sensory deficits. In addition,
an impinged L5 nerve root would not affect knee extension or the patellar reflex but would instead
lead to impaired dorsiflexion of the foot due to impaired innervation of the tibialis anterior and
extensor hallucis longus muscles.

B - S2

Explanation Why

S2 radiculopathy is relatively rare compared to other radiculopathies and may present with
diminished sensation affecting the posterior aspect of the thigh, partly extending down to the
posterior leg and the perineum. Moreover, affected individuals often have urinary or fecal
incontinence and sexual dysfunction, none of which this patient has.

C - S1

Explanation Why

S1 radiculopathy typically presents with sensory deficits involving the posterolateral aspect of the
thigh, anterolateral as well as the posterolateral lower leg, and the lateral foot, which is inconsistent
with this patient's presentation. Moreover, an impinged S1 nerve root would not affect the patellar
reflex but would instead lead to diminished ankle jerk reflex in combination with difficulty walking
on toes due to impaired innervation of the gastrocnemius, peroneus longus, and peroneus brevis
muscles.

www.eduwaves360.com | Telegram : @eduwaves360


D - L3

Explanation Why

L3 radiculopathy typically manifests with impaired sensation extending from the mid-anterolateral
thigh to the medial thigh just above the patella, unlike the deficits seen in this patient. While affected
individuals may present with a reduced adductor reflex and/or a mildly reduced patellar reflex along
with mild impairment of knee extension, impaired hip flexion and adduction is more characteristic of
L3 radiculopathy.

E - L4

Image

Explanation Why

L4 radiculopathy characteristically manifests with sensory loss or paresthesia extending from the
distal anterolateral thigh over the patella to the medial lower leg, as seen in this patient. Diminished

www.eduwaves360.com | Telegram : @eduwaves360


knee extension would also be expected since innervation of the quadriceps is impaired. This patient's
reduced patellar reflex further supports the diagnosis of impingement of the L4 nerve root as the
underlying cause.

www.eduwaves360.com | Telegram : @eduwaves360


Question # 24

An investigator is studying the microstructure of skeletal muscle fibers to identify new targets for
skeletal muscle relaxation. He obtains a biopsy from the brachioradialis of a test subject and observes it
under a transmission electron microscope. A photomicrograph of the structures seen during microscopy
is shown. Which of the following parts contains the greatest proportion of actin filaments?

Answer Image

A A

www.eduwaves360.com | Telegram : @eduwaves360


Answer Image

B B

C C

D D

E E

www.eduwaves360.com | Telegram : @eduwaves360


Hint

The structure in question decreases in length during muscle contraction.

www.eduwaves360.com | Telegram : @eduwaves360


Correct Answer

A-A

Image

Explanation Why

This structure is the I band, a region of the sarcomere that contains the greatest proportion of actin
filaments. During contraction, the power stroke mechanism of myosin heads form a crossbridge with
actin that pulls the thin filament along the myosin (the thick filament) toward the M line, decreasing
the width of the I band.

B-B

Explanation Why

This structure is the A band, which extends the full length of the myosin filament. A bands may also

www.eduwaves360.com | Telegram : @eduwaves360


contain actin filaments, which overlap with myosin filaments during contraction. The A band
remains the same length throughout contraction.

C-C

Explanation Why

This structure is the H zone, a central, pale segment on either side of the M line, within the A band,
that contains only myosin. Actin filaments do not overlap with myosin filaments in this zone. The H
zone decreases in size during contraction, when actin slides over myosin toward the M line

D-D

Explanation Why

This structure is the M line, where myomesin proteins crosslink and anchor myosin filaments. There
is no actin in this region of the sarcomere.

E-E

Explanation Why

This structure is the Z line, which separates one sarcomere from the other. It consists of proteins that
serve as anchoring points for actin filaments and titin.

www.eduwaves360.com | Telegram : @eduwaves360


Question # 25

A 26-year-old woman comes to the physician because of severe pain in her right wrist one day after
falling onto her hands and knees while rollerskating. Physical examination shows abrasions over the
knees and bruising over the volar aspect of the right wrist. There is swelling and tenderness on palpation
of the volar wrist joint, as well as restricted range of motion due to pain. An x-ray of the hand shows
volar dislocation of the lunate bone. Further evaluation is most likely to show which of the following?

Answer Image

A Paresthesia over the volar aspect of the first 3 fingers on wrist flexion

B Anesthesia over the dorsal aspect of the first 3 fingers

C Tenderness to palpation of the anatomic snuffbox

D Involuntary flexion of the 4th and 5th interphalangeal joints when extending all
fingers

E Pale skin color on the volar surface when pressure is applied to the radial artery

www.eduwaves360.com | Telegram : @eduwaves360


Hint

Volar dislocation of the lunate bone can cause compression of structures within the carpal tunnel.

www.eduwaves360.com | Telegram : @eduwaves360


Correct Answer

A - Paresthesia over the volar aspect of the first 3 fingers on wrist flexion

Explanation Why

Paresthesia over the volar aspect of D1, D2, and D3 on wrist flexion (e.g., positive Phalen's test) is
consistent with median nerve injury, which can occur due to carpal tunnel compression from lunate
dislocation. Lunate dislocations are most commonly the result of falling onto a hyperextended and
ulnar-deviated wrist and typically present with acute wrist pain and swelling and tenderness to
palpation of the volar wrist, as seen in this patient. Other complications related to lunate dislocation
include avascular necrosis.

B - Anesthesia over the dorsal aspect of the first 3 fingers

Explanation Why

Anesthesia limited to the dorsal aspects of D1, D2, and D3 is consistent with a radial nerve
neuropathy at the level of the wrist, which can be caused by external compression (e.g., due to a tight
watch, handcuffs) and manifests with wrist tenderness. Because the radial nerve lies over the
extensor retinaculum at the level of the wrist, it would not be affected by a lunate dislocation. Given
this patient's mechanism of injury and x-ray demonstrating a lunate dislocation, anesthesia of the
dorsal aspect of D1, D2, and D3 would be unlikely.

C - Tenderness to palpation of the anatomic snuffbox

Explanation Why

Tenderness to palpation of the anatomic snuffbox is seen in scaphoid fractures, which most
commonly occur due to falling onto a hyperextended and radially-deviated wrist. Although scaphoid
fractures manifest with acute wrist pain and swelling as well as tenderness with volar palpation of the
wrist, an x-ray indicating lunate dislocation in the absence of a scaphoid fracture makes anatomic
snuffbox tenderness unlikely.

www.eduwaves360.com | Telegram : @eduwaves360


D-

Involuntary flexion of the 4th and 5th interphalangeal joints when extending all
fingers

Explanation Why

Involuntary flexion of D4 and D5 interphalangeal joints with finger extension is consistent with ulnar
nerve injury due to compression of Guyon's canal. Acute ulnar tunnel syndrome may occur from
lunate dislocation but has only rarely been reported. A different complication of lunate dislocation is
much more likely.

E-

Pale skin color on the volar surface when pressure is applied to the radial
artery

Explanation Why

On a modified Allen test, performed by applying pressure to the radial artery, pale skin on the hand's
volar surface would suggest an ulnar artery injury such as from a laceration or an occlusive
pathology such as hypothenar hammer syndrome (HHS). Acute HHS is the result of blunt trauma to
the hypothenar eminence. Although a patient with HHS may present with acute wrist pain, swelling,
and tenderness to palpation of the volar wrist, this patient's x-ray shows a lunate dislocation, making
a different finding more likely.

www.eduwaves360.com | Telegram : @eduwaves360


Question # 26

A 23-year-old woman is brought to the emergency department because of sudden onset of severe knee
pain and swelling that began 30 minutes earlier while she was at soccer practice. She was running down
the field, suddenly “cut inwards”, and heard a loud pop when she first felt the pain. She is unable to bear
weight on the right knee. Examination shows marked edema and tenderness to palpation of the anterior
right knee. When the knee is flexed and anterior traction is provided to the foreleg, anterior translation
of the foreleg is observed. An illustration of a normal right knee in anterior view is shown. The most
likely cause of this patient's condition is injury to which of the following labeled structures?

Answer Image

A A

www.eduwaves360.com | Telegram : @eduwaves360


Answer Image

B B

C C

D D

E E

F F

www.eduwaves360.com | Telegram : @eduwaves360


Hint

The test performed here is the Lachman test. In this case, it is positive, demonstrating laxity.

www.eduwaves360.com | Telegram : @eduwaves360


Correct Answer

A-A

Image

Explanation But

The ACL can be injured in isolation or together with other structures in the knee joint (e.g., along
with the medial meniscus, as part of the “unhappy triad”).

Explanation Why

“A” refers to the anterior cruciate ligament (ACL), the most commonly injured knee ligament. The
mechanism of this patient's injury is typical for damage to the ACL: Sudden deceleration and
twisting to change direction as occurs during pivoting movements (e.g., “cutting inwards”). This
results in an audible pop followed by knee swelling (i.e., hemarthrosis) and joint instability. Clinical
tests for injury to the ACL (e.g., the Lachman test, anterior drawer test, and pivot shift test) assess for
laxity of the injured ligament compared to the contralateral side.

www.eduwaves360.com | Telegram : @eduwaves360


B-B

Explanation Why

“B” refers to the lateral meniscus, which can be injured when sudden directional change is applied to
a flexed knee with the foot firmly planted, similar to the pivoting movement described by this
patient. Examination of a patient with a tear of the lateral meniscus would show a positive
McMurray test and a positive Apley grinding test on internal rotation of the leg. However, the
positive Lachman test described here indicates injury of another structure.

C-C

Explanation Why

“C” refers to the lateral collateral ligament (LCL), which is not commonly injured in isolation. The
typical mechanism for injury to the LCL is force applied to a fully extended knee from the medial or
anteromedial aspect of the joint (varus stress), which is not the scenario described here. Clinical
signs of injury to the LCL are a positive varus stress test and lateral joint laxity. The positive
Lachman test described here, however, indicates injury of another structure.

D-D

Explanation Why

“D” refers to the posterior cruciate ligament (PCL), which is most commonly injured when a flexed
knee receives a sudden posterior-directed force (e.g., the dashboard injury). It can also be injured in
sports during a fall onto a flexed knee with a plantarflexed foot, which is not the scenario described
in this case. The clinical sign of injury to the PCL is posterior movement with traction (i.e., a positive
posterior drawer test). However, the Lachman test described here indicates injury of another
structure.

www.eduwaves360.com | Telegram : @eduwaves360


E-E

Explanation Why

“E” refers to the medial meniscus, which can be injured when sudden directional change is applied to
a flexed knee with the foot firmly planted, similar to the pivoting movement described by this
patient. Examination of a patient with a tear of the medial meniscus would show a positive
McMurray test and a positive Apley grinding test on external rotation of the leg. However, the
positive Lachman test described here indicates injury of another structure.

F-F

Explanation Why

“F” refers to the medial collateral ligament (MCL), which can be injured when sudden directional
change is applied to a flexed knee with the foot firmly planted, similar to the pivoting movement
described by this patient. It can also be damaged by a direct blow to the lateral knee (valgus stress).
Clinical signs of injury to the MCL are a positive valgus stress test and medial joint laxity, but the
positive Lachman test described here indicates injury of another structure.

www.eduwaves360.com | Telegram : @eduwaves360


Question # 27

A 27-year-old man comes to the physician because of pain and swelling in his right knee that began
3 days ago when he fell during football practice. He fell on his flexed right knee as he dove to complete
a pass. He felt some mild knee pain but continued to practice. Over the next 2 days, the pain worsened
and the knee began to swell. Today, the patient has an antalgic gait. Examination shows a swollen and
tender right knee; flexion is limited by pain. The right knee is flexed and pressure is applied to proximal
tibia; 8 mm of backward translation of the foreleg is observed. Which of the following is most likely
injured?

Answer Image

A Medial meniscus

B Posterior cruciate ligament

C Anterior cruciate ligament

D Medial collateral ligament

E Lateral collateral ligament

F Lateral meniscus

www.eduwaves360.com | Telegram : @eduwaves360


Hint

Flexion of the quadriceps of the right thigh would translate the tibia forward; this is called a positive
quadriceps active test and it is another sign of the patient's condition.

www.eduwaves360.com | Telegram : @eduwaves360


Correct Answer

A - Medial meniscus

Explanation Why

Medial meniscus injury is a common cause of knee pain but would not explain this patient's positive
posterior drawer test. Instead, McMurray test would be positive.

B - Posterior cruciate ligament

Image

Explanation Why

Posterior cruciate ligament (PCL) injury can occur when the flexed knee is subjected to excessive
posterior force, as in the case of this patient. He has a positive posterior drawer sign on the affected
side, which is highly suggestive of PCL injury. PCL injury is confirmed by MRI and can be treated

www.eduwaves360.com | Telegram : @eduwaves360


conservatively (in the case of isolated injury) or surgically (in the case of multi-ligament injury or
chronic symptoms).

C - Anterior cruciate ligament

Explanation But

Patients with PCL injury can sometimes appear to have a positive anterior drawer sign if the
examiner is not careful. If a patient has a posterior sag sign from their PCL injury, then the affected
tibia is posteriorly displaced when the knee is flexed (as when performing an anterior drawer test). If
the examiner is not aware of the posterior sag and performs an anterior drawer test, the affected knee
will feel as though it has significantly more anterior movement compared to the unaffected knee
(because the affected knee started from a posteriorly displaced position) and may be mistaken for a
positive anterior drawer test. The result of this mistake is a misdiagnosis of ACL injury when, in fact,
it is the PCL that is injured!

Explanation Why

Anterior cruciate ligament injury causes knee pain but would not explain this patient's posterior
drawer sign. Instead, ACL injury would present with a positive anterior drawer sign and a positive
Lachman sign.

D - Medial collateral ligament

Explanation Why

Medial collateral ligament injury causes knee pain but would not explain this patient's posterior
drawer sign. MCL injury frequently occurs after valgus stress and presents with medial joint line
tenderness and valgus stress laxity.

www.eduwaves360.com | Telegram : @eduwaves360


E - Lateral collateral ligament

Explanation Why

Lateral collateral ligament injury causes knee pain but would not explain the patient's positive
posterior drawer test. Isolated LCL injury is rare; it would present with lateral joint line tenderness
with varus stress laxity.

F - Lateral meniscus

Explanation Why

Lateral meniscus injury causes knee pain but would not explain this patient's positive posterior
drawer test. Instead, McMurray test would be positive.

www.eduwaves360.com | Telegram : @eduwaves360


Question # 28

A 6-year-old girl is brought to the emergency department because of right elbow swelling and pain
30 minutes after falling onto her outstretched right arm. She has been unable to move her right elbow
since the fall. Physical examination shows bruising, swelling, and tenderness of the right elbow; range of
motion is limited by pain. An x-ray of the right arm shows a supracondylar fracture of the humerus with
anterior displacement of the proximal fragment. Further evaluation is most likely to show which of the
following findings?

Answer Image

A Absent distal radial pulse

B Radial deviation of the wrist

C Atrophy of the thenar eminence

D Inability to abduct shoulder

E Absent pulse of the proximal brachial artery

F Inability to flex the elbow

www.eduwaves360.com | Telegram : @eduwaves360


Hint

In supracondylar fractures, the proximal fragment of the distally fractured humerus can displace
anteriorly into soft tissue structures, impinging important blood vessels and nerves.

www.eduwaves360.com | Telegram : @eduwaves360


Correct Answer

A - Absent distal radial pulse

Explanation Why

An absent radial pulse in the setting of a distal or supracondylar humerus fracture indicates
disruption of the supplying brachial artery, which passes anterior to the distal humerus and is,
therefore, commonly injured. Displaced supracondylar fractures may also result in neuropraxic
injuries. Anterolateral displacement of the proximal fragment can cause radial nerve injury, while
medial displacement of the proximal fragment can cause median nerve injury.

B - Radial deviation of the wrist

Explanation Why

Damage to the ulnar nerve causes radial deviation of the wrist (proximal lesions), finger spreading,
and flexion of the last two digits. Ulnar nerve damage due to supracondylar fractures is possible but
very rare and only occurs with flexion type supracondylar fractures. A different examination finding
is more likely with this patient's fracture pattern.

www.eduwaves360.com | Telegram : @eduwaves360


C - Atrophy of the thenar eminence

Image

Explanation Why

Atrophy of the thenar eminence is a sign of injury to the median nerve, which may occur due to
supracondylar fractures. However, atrophy of muscles is a more chronic finding of nerve damage and
would unlikely be seen in an acute setting.

D - Inability to abduct shoulder

Explanation Why

The inability to abduct the shoulder suggests damage to the axillary nerve, which innervates the
deltoid muscle. However, the axillary nerve branches off from the posterior cord of the brachial
plexus and is too proximal to be damaged by a supracondylar fracture.

www.eduwaves360.com | Telegram : @eduwaves360


E - Absent pulse of the proximal brachial artery

Explanation Why

The brachial artery is a continuation of the axillary artery, passing below the level of teres major.
Impairments of blood flow from the axillary artery, therefore, manifest with pulselessness at the
proximal brachial artery. However, the proximal brachial artery is too proximal to be caused by a
supracondylar fracture.

F - Inability to flex the elbow

Explanation Why

Impaired elbow flexion is consistent with a musculocutaneous nerve injury, due to this nerve's
innervation of the biceps and brachialis muscles. Injury to the sensory branch of the
musculocutaneous nerve (e.g., the lateral antebrachial cutaneous nerve) is possible with a distal
humerus fracture and would present with anterior lateral elbow and forearm paresthesias. However, a
supracondylar fracture is too distal to affect the motor function of the musculocutaneous nerve.

www.eduwaves360.com | Telegram : @eduwaves360


Question # 29

An 80-year-old woman is brought to the emergency department for left hip pain 30 minutes after she fell
while walking around in her room. Examination shows left groin tenderness. The range of motion of the
left hip is limited because of pain. An x-ray of the hip shows a linear fracture of the left femoral neck
with slight posterior displacement of the femur. Which of the following arteries was most likely
damaged in the patient's fall?

Answer Image

A Lateral femoral circumflex

B Deep femoral artery

C Medial circumflex femoral

D Deep circumflex iliac

E Obturator

F Superior gluteal artery

www.eduwaves360.com | Telegram : @eduwaves360


Hint

The artery that courses posterior to the femoral neck is at greatest risk of injury in this case. Damage to
this artery can cause avascular necrosis of the femoral head.

www.eduwaves360.com | Telegram : @eduwaves360


Correct Answer

A - Lateral femoral circumflex

Explanation Why

The lateral circumflex femoral artery arises from the profunda femoris distal to the greater trochanter.
This structure supplies blood to muscles of the anterior and lateral compartments of the thigh and is
at risk in femoral shaft or intertrochanteric fractures. Although they are small, variable branches off
of the lateral femoral circumflex can supply a minority of the femoral head's blood supply. However,
this vessel is unlikely to have been damaged in this patient with posterior displacement because of its
location anterior to the femoral head.

B - Deep femoral artery

Explanation Why

The deep femoral artery, also known as the profunda femoris, is one of two major divisions of the
femoral artery (the other being the superficial femoral artery). It typically branches off the femoral
artery near the greater trochanter, distal to the site of a femoral neck fracture. Rarely, open reduction
and internal fixation of a femoral neck fracture may lead to injury of this vessel, but this patient's
fracture in itself would be unlikely to pose a risk.

www.eduwaves360.com | Telegram : @eduwaves360


C - Medial circumflex femoral

Image

Explanation Why

The medial circumflex femoral artery is a major branch of the profunda femoris. It courses
medioposteriorly to the neck of the femur, where it supplies blood to the head and neck of the femur.
A minority of femoral neck fractures will go on to develop osteonecrosis of the femoral head
stemming from injuries to this artery and its small branches to the bone.

D - Deep circumflex iliac

Explanation Why

The deep circumflex iliac artery resides on the medial aspect of the iliac crest. Because of its
intrapelvic course, it is not at risk in cases of femoral neck fractures.

www.eduwaves360.com | Telegram : @eduwaves360


E - Obturator

Explanation Why

The obturator artery supplies muscles in the medial compartment of the thigh, including gracilis, the
leg adductors, and external obturator. Other smaller divisions provide blood to the inferior ramus of
the ischium and the head of the femur (via the ligamentum teres femoris). This blood supply can be
disrupted in the femoral head and acetabular fractures but would very likely be spared in a femoral
neck fracture, as in this patient.

F - Superior gluteal artery

Explanation Why

The superior gluteal artery exits the pelvis posteriorly and supplies all three gluteal muscles as well
as the tensor fasciae latae. It also has branches that supply the ischium. This structure is not
vulnerable to injury in femoral neck fractures such as this patient experienced.

www.eduwaves360.com | Telegram : @eduwaves360


Question # 30

A 28-year-old man is brought to the emergency department 20 minutes after being involved in a
bicycling accident. He complains of severe pain over the front of his right shoulder. He refuses to move
his right arm. Physical examination shows supraclavicular swelling and bruising. The shoulder's range
of motion is limited by pain. An x-ray of the shoulder shows a fracture of the middle third of the clavicle
with complete superior displacement of the medial clavicular segment. Which of the following muscles
is responsible for the displacement of this segment?

Answer Image

A Trapezius

B Deltoid

C Subclavius

D Pectoralis major

E Sternocleidomastoid

www.eduwaves360.com | Telegram : @eduwaves360


Hint

The muscle in question is innervated by the spinal accessory nerve and functions predominantly in the
rotation of the head.

www.eduwaves360.com | Telegram : @eduwaves360


Correct Answer

A - Trapezius

Explanation Why

The superior fibers of the trapezius originate at the external occipital protuberance and nuchal
ligament, and insert into the posterior border of the lateral third of the clavicle. In the setting of a
clavicular fracture, the trapezius would pull the lateral clavicular fragment both superiorly and
posteriorly. Although the trapezius is innervated by the spinal accessory nerve, a different muscle
would be responsible for superior displacement of the medial clavicle.

B - Deltoid

Explanation Why

The deltoid muscle attaches to the humerus and the anterior border of the lateral third of the clavicle.
In the setting of a clavicular fracture, the deltoid muscle would therefore pull the lateral clavicular
segment downwards.

C - Subclavius

Explanation Why

The subclavius muscle originates from the first rib and inserts onto the inferior surface of the
clavicle. In the setting of a clavicular fracture, it would therefore pull the clavicle downwards.

www.eduwaves360.com | Telegram : @eduwaves360


D - Pectoralis major

Explanation Why

The pectoralis major muscle originates at the sternum and medial half of the clavicle and inserts at
the humerus. In the setting of a clavicular fracture with an intact acromioclavicular joint and
glenohumeral joint, this muscle would cause medial displacement of the lateral clavicular segment.

E - Sternocleidomastoid

Image

Explanation Why

The sternocleidomastoid muscle is superiorly attached to the mastoid process and inferiorly attached
to the medial third of the clavicle. Therefore, in the setting of a fracture, as seen in this patient, the
sternocleidomastoid pulls the medial segment of the fractured clavicle upwards.

www.eduwaves360.com | Telegram : @eduwaves360


Question # 31

A 37-year-old man is brought to the emergency department after being attacked with a knife. Physical
examination shows a 4-cm laceration in the midline of the right forearm. An MRI of the right arm shows
damage to a nerve that runs between the superficial and deep flexor digitorum muscles. Loss of
sensation over which of the following areas is most likely in this patient?

Answer Image

A Dorsal aspect of the forearm

B Palmar surface of the little finger

C Lateral aspect of the forearm

D Fingertip of the index finger

E Medial aspect of the forearm

F Dorsum of the thumb

www.eduwaves360.com | Telegram : @eduwaves360


Hint

The median nerve, which originates from the lateral and medial cords of the brachial plexus, passes
down the arm into the cubital fossa and runs in the midline of the ventral compartment of the forearm
between the flexor digitorum superficialis and flexor digitorum profundus.

www.eduwaves360.com | Telegram : @eduwaves360


Correct Answer

A - Dorsal aspect of the forearm

Explanation Why

The dorsal aspect of the forearm is innervated by a branch of the radial nerve called the posterior
antebrachial cutaneous nerve, which lies in the extensor compartment of the forearm. A different
nerve lies between the flexor digitorum superficialis and flexor digitorum profundus.

B - Palmar surface of the little finger

Explanation Why

The palmar surface of the little finger is innervated by palmar branches of the ulnar nerve, which
enters the anterior compartment of the forearm between the two heads of flexor carpi ulnaris and lies
between the flexor digitorum superficialis and flexor digitorum profundus. However, the ulnar nerve
is located medially; it is less likely to be affected by a laceration in the midline of the forearm.

C - Lateral aspect of the forearm

Explanation Why

The lateral aspect of the forearm is innervated by a branch of the musculocutaneous nerve called the
lateral antebrachial cutaneous nerve. Neither the musculocutaneous nerve nor the lateral antebrachial
cutaneous nerve lies between the flexor digitorum superficialis and flexor digitorum profundus.

www.eduwaves360.com | Telegram : @eduwaves360


D - Fingertip of the index finger

Image

Explanation Why

The median nerve passes through the carpal tunnel into the hand. Within the hand, the median nerve
motor branch innervates the thenar muscles and the lateral two lumbricals. The median nerve also
provides sensory innervation to the thenar eminence as well as the dorsal and palmar aspect of the
lateral 3½ fingers including the fingertips. Note that the lateral border of the thenar eminence is
innervated by the radial nerve.

E - Medial aspect of the forearm

Explanation Why

The medial aspect of the forearm is innervated by the medial antebrachial cutaneous nerve. A
different nerve lies between the flexor digitorum superficialis and flexor digitorum profundus.

www.eduwaves360.com | Telegram : @eduwaves360


F - Dorsum of the thumb

Explanation Why

The dorsum of the thumb is innervated by the superficial branch of the radial nerve, which travels
through the forearm under the brachioradialis. A different nerve lies between the flexor digitorum
superficialis and flexor digitorum profundus.

www.eduwaves360.com | Telegram : @eduwaves360


Question # 32

An investigator is studying muscle contraction in tissue obtained from the thigh muscle of an
experimental animal. After injection of radiolabeled ATP, the tissue is stimulated with electrical
impulses. Radioassay of these muscle cells is most likely to show greatest activity in which of the
following structures?

Answer Image

A H zone

B I band

C A band

D Z line

E M line

www.eduwaves360.com | Telegram : @eduwaves360


Hint

Activity will be greatest at the site of cross-bridge formation.

www.eduwaves360.com | Telegram : @eduwaves360


Correct Answer

A - H zone

Explanation Why

The H zone contains the central portion of myosin filaments, where there are no overlapping actin
filaments. Myosin heads, which bind and hydrolyze ATP, are not present in the H zone.

B - I band

Explanation Why

The I band contains only actin filaments, which do not bind ATP.

www.eduwaves360.com | Telegram : @eduwaves360


C - A band

Image

Explanation Why

The outer segments of the A band are the sites where actin and myosin filaments overlap and form
cross-bridges via the myosin heads, which bind and hydrolyze ATP. Following injection of
radiolabeled ATP, radioassay is most likely to show the greatest activity in the A band of the
sarcomere.

D - Z line

Explanation Why

The Z line is where actin filaments are anchored to the cytoskeleton. Myosin heads, which bind and
hydrolyze ATP, are not present at the Z line.

www.eduwaves360.com | Telegram : @eduwaves360


E - M line

Explanation Why

The M line is where myosin filaments are anchored to the cytoskeleton. Myosin heads, which bind
and hydrolyze ATP, are not present at the M line.

www.eduwaves360.com | Telegram : @eduwaves360


Question # 33

A 2850-g (6-lb 5-oz) newborn is delivered at term to a 19-year-old primigravid woman via normal
spontaneous vaginal delivery. The mother has had no prenatal care. Examination of the newborn in the
delivery room shows a small, retracted jaw and hypoplasia of the zygomatic arch. This patient's
condition is most likely caused by abnormal development of the structure that also gives rise to which of
the following?

Answer Image

A Facial nerve

B Cricothyroid muscle

C Incus

D Greater horn of hyoid

E Platysma

www.eduwaves360.com | Telegram : @eduwaves360


Hint

This patient's hypoplastic mandible and zygomatic bone suggest anomalous development of the first
branchial arch.

www.eduwaves360.com | Telegram : @eduwaves360


Correct Answer

A - Facial nerve

Explanation Why

The facial nerve is the nerve associated with the second branchial arch. Anomalies of the second
branchial arch involve the muscles of facial expression, posterior belly of the digastric, stylohyoid
muscle, stapedius, derivates of the Reichert cartilage, and/or the facial nerve (e.g., underdeveloped
facial muscles, facial nerve dysfunction, aplasia of the stapes with conductive hearing loss). Second
branchial cleft cyst and Treacher Collins syndrome are further second branchial arch anomalies.
However, neither the mandible nor the zygomatic bone is derived from the second branchial arch.
The facial nerve is, therefore, unlikely to be involved here.

B - Cricothyroid muscle

Explanation Why

The cricothyroid muscle is a derivative of the fourth branchial arch. Anomalies of the fourth
branchial arch involve the structures of the larynx and the vagal nerve (e.g., congenital laryngeal
stenosis, vagal nerve dysfunction). Fourth branchial cleft cyst and double aorta are also fourth
branchial arch anomalies. However, neither the mandible nor the zygomatic bone is derived from the
fourth branchial arch, and this newborn has no other evidence of fourth branchial arch anomalies.

www.eduwaves360.com | Telegram : @eduwaves360


C - Incus

Image

Explanation But

The first branchial arch is associated with cranial nerve V3 and the maxillary artery. It also gives rise
to the mylohyoid, anterior belly of the digastric, tensor tympani, tensor veli palatini, and all of the
muscles of mastication.

Explanation Why

The incus is one of the bones derived from the first branchial arch, along with the malleus, mandible,
squamous temporal bone, palatine bone, zygomatic bone, vomer, and maxilla. Given this patient's
hypoplastic mandible and zygomatic bone, malformations of other first branchial arch derivatives
may be present, e.g., hypoplasia of the incus (manifesting as conductive hearing loss). The first
branchial arch is also involved in the Pierre Robin sequence and in Treacher-Collins syndrome.

www.eduwaves360.com | Telegram : @eduwaves360


D - Greater horn of hyoid

Explanation Why

The greater horn of the hyoid bone is a derivative of the third branchial arch. Anomalies of the third
branchial arch involve the structures of the pharynx and glossopharyngeal nerve (e.g.,
glossopharyngeal nerve dysfunction). Third branchial cleft cyst and internal carotid artery hypoplasia
are also third branchial arch anomalies. However, neither the mandible nor the zygomatic bone is
derived from the third branchial arch, and this newborn has no other evidence of third branchial arch
anomalies.

E - Platysma

Explanation Why

The platysma is a derivative of the second branchial arch. Anomalies of the second branchial arch
involve the muscles of facial expression, posterior belly of the digastric, stylohyoid muscle,
stapedius, derivates of the Reichert cartilage, and/or the facial nerve (e.g., underdeveloped facial
muscles, facial nerve dysfunction, aplasia of the stapes with conductive hearing loss). Second
branchial cleft cyst and Treacher-Collins syndrome are further second branchial arch anomalies.
However, neither the mandible nor the zygomatic bone is derived from the second branchial arch.
The platysma is, therefore, unlikely to be involved here.

www.eduwaves360.com | Telegram : @eduwaves360


Question # 34

A 36-year-old woman comes to the physician because of blurred vision and difficulty keeping her eyes
open. She also has occasional difficulty chewing, especially when eating meat or other foods that require
prolonged chewing. The symptoms are worse at the end of the day. Physical examination shows bilateral
drooping of the eyelids, which becomes more pronounced when she is asked to look upwards for
30 seconds. Which of the following is the most likely cause of this patient's symptoms?

Answer Image

Inhibition of calcium release from


A
the sarcoplasmic reticulum

Interrupted transmission of T-tubule


B
depolarization

Sustained blockade of actin myosin-


C
binding sites

Decreased generation of end plate


D
potential

Impaired flow of calcium ions


E
between gap junctions

www.eduwaves360.com | Telegram : @eduwaves360


Hint

This patient's ptosis, muscle fatiguability, and positive Simpson test suggest a diagnosis of myasthenia
gravis. Cholinesterase inhibitors can be used both to diagnose and treat myasthenia gravis.

www.eduwaves360.com | Telegram : @eduwaves360


Correct Answer

A - Inhibition of calcium release from the sarcoplasmic reticulum

Explanation Why

The inhibition of calcium release from the sarcoplasmic reticulum facilitates muscle relaxation and is
the mechanism of action of hydralazine. This patient, however, presents with myasthenia gravis
(MG), which affects a different step in neuromuscular conduction and muscular contraction.

B - Interrupted transmission of T-tubule depolarization

Explanation Why

Interrupted transmission of T-tubule depolarization impairs the fast transmission of action potentials
to muscle cells, which may reduce muscle strength. This pathology can be seen following myocardial
infarction, for example. This patient, however, presents with symptoms of myasthenia gravis (MG),
which affects a different step of neuromuscular conduction.

C - Sustained blockade of actin myosin-binding sites

Explanation Why

The blockade of myosin-binding sites on actin filaments is facilitated by tropomyosin. However, in a


patient with myasthenia gravis (MG), another pathomechanism that sustains blockade of these
binding sites is responsible for the symptoms of ptosis and muscle weakness.

www.eduwaves360.com | Telegram : @eduwaves360


D - Decreased generation of end plate potential

Image

Explanation Why

In patients with myasthenia gravis (MG), antibodies against post-synaptic ACh receptors compete
with ACh at the post-synaptic membrane, which leads to decreased end plate potential despite
normal ACh release. This causes muscle weakness and reduced endurance with repetitive muscle
use, as manifested in this patient with difficulty chewing for a prolonged period of time and
worsening of symptoms in the evening. Ptosis in myasthenia gravis can be provoked with the
Simpson test, as seen here.

E - Impaired flow of calcium ions between gap junctions

Explanation Why

Impaired flow of calcium ions between gap junctions would impair the coupled contraction of

www.eduwaves360.com | Telegram : @eduwaves360


cardiac myocytes. Gap junctions are not present in the striated muscle that is affected by myasthenia
gravis.

www.eduwaves360.com | Telegram : @eduwaves360


Question # 35

A 3629-g (8-lb) newborn is examined shortly after spontaneous vaginal delivery. She was delivered at
40 weeks' gestation and pregnancy was uncomplicated. Her mother is concerned because she is not
moving her left arm as much as her right arm. Physical examination shows her left arm to be adducted
and internally rotated, with the forearm extended and pronated, and the wrist flexed. The Moro reflex is
present on the right side but absent on the left side. Which of the following brachial plexus structures is
most likely injured in this infant?

Answer Image

A Upper trunk

B Long thoracic nerve

C Lower trunk

D Axillary nerve

E Posterior cord

www.eduwaves360.com | Telegram : @eduwaves360


Hint

This infant's left arm is in a “waiter's tip” position, which suggests impaired shoulder abduction and
lateral rotation, as well as impaired elbow flexion and forearm supination. This injury is common in
neonates who require lateral neck traction during vaginal delivery and is associated with shoulder
dystocia.

www.eduwaves360.com | Telegram : @eduwaves360


Correct Answer

A - Upper trunk

Image

Explanation Why

Erb palsy is due to injury to the upper trunk of the brachial plexus, which contributes to the motor
and sensory function of the suprascapular nerve, musculocutaneous nerve, and axillary nerve. Erb
palsy most commonly occurs secondary to shoulder dystocia or other birth trauma but can arise in
adults as a result of neck and/or upper arm trauma.

B - Long thoracic nerve

Explanation Why

Injury to the long thoracic nerve, which receives nerve fibers from the C5–C7 nerve roots, classically

www.eduwaves360.com | Telegram : @eduwaves360


manifests with impaired abduction of the arm above 90° and winging of the scapula (due to
innervation of the serratus anterior muscle). An isolated long thoracic nerve injury due to birth
trauma would be very unusual. This patient's examination findings are consistent with injury to a
different structure.

C - Lower trunk

Explanation Why

Injury to the lower trunk of the brachial plexus results in Klumpke palsy. Klumpke palsy commonly
occurs secondary to birth trauma. It typically manifests as paralysis of the intrinsic hand muscles and
forearm flexors, resulting in the characteristic “claw hand” and absent grasp reflex. Horner syndrome
may also be present. This patient's “waiter's tip” position is not consistent with a lower trunk injury.

D - Axillary nerve

Explanation Why

Injury to the axillary nerve would result in the inability to externally rotate and abduct the arm above
15°. An isolated axillary nerve injury would not explain this patient's other findings.

E - Posterior cord

Explanation Why

Injury to the posterior cord classically results in wrist drop (loss of radial nerve innervation of
forearm extensors) and inability to externally rotate or abduct the arm (loss of axillary nerve
innervation of deltoid muscle). Isolated damage to the posterior cord during birth is extremely rare
and would not be consistent with the examination findings in this patient.

www.eduwaves360.com | Telegram : @eduwaves360


Question # 36

A 19-year-old man is brought to the emergency department because of severe right shoulder pain and
inability to move the shoulder after a fall while climbing outdoors. Examination shows the right upper
extremity is externally rotated and slightly abducted. There is loss of the the normal rounded appearance
of the shoulder. The right humeral head is palpated below the coracoid process. Sensation to pinprick
over which of the following skin areas is most likely to be reduced?

Answer Image

A Medial aspect of the upper arm

B Lateral aspect of the shoulder

C Hypothenar eminence and ulnar-sided fingers

D Thenar eminence and radial-sided fingers

E Skin above the clavicle

F Lateral aspect of forearm

G Dorsal aspect of forearm and hand

www.eduwaves360.com | Telegram : @eduwaves360


Hint

A combination of external rotation and abduction of the arm, flattening of the shoulder contour, and
palpation of the humeral head below the coracoid process is indicative of anterior shoulder dislocation.
The axillary nerve is stretched over the displaced head of the humerus (neurapraxia), making it
especially prone to injury.

www.eduwaves360.com | Telegram : @eduwaves360


Correct Answer

A - Medial aspect of the upper arm

Explanation Why

The medial aspect of the upper arm is supplied by the medial brachial cutaneous nerve, a direct
branch of the brachial plexus. Injuries are rare but can occur as a result of surgical error or with
trauma to the brachial plexus itself.

B - Lateral aspect of the shoulder

Explanation Why

The nerve most commonly damaged as a result of anterior shoulder dislocation is the axillary nerve,
which has a dermatome over the lateral shoulder. This injury thus often causes loss of sensation in
this area.

C - Hypothenar eminence and ulnar-sided fingers

Explanation Why

The hypothenar eminence and ulnar-sided fingers are innervated by the ulnar nerve. Injuries to this
nerve can be seen with fractures of the medial condyle of the humerus or the hamate. The ulnar nerve
is unaffected by anterior shoulder dislocations.

www.eduwaves360.com | Telegram : @eduwaves360


D - Thenar eminence and radial-sided fingers

Explanation Why

The thenar eminence and radial-sided fingers are innervated by the median nerve. Median nerve
injuries can arise as a result of supracondylar humeral fractures, carpal tunnel syndrome, or
lacerations at the wrist. The median nerve is unaffected by anterior shoulder dislocations.

E - Skin above the clavicle

Explanation Why

The skin above the clavicle is innervated by the supraclavicular nerve, along with the skin of the
posterior shoulder and the anterolateral chest wall. The supraclavicular nerve runs superior to the
humeral head and is unaffected by an anterior dislocation.

F - Lateral aspect of forearm

Explanation Why

Musculocutaneous nerve injury would affect sensation over the lateral aspect of the forearm. These
injuries are uncommon and would not be affected by an anterior shoulder dislocation.

G - Dorsal aspect of forearm and hand

Explanation Why

The dorsal aspect of the forearm and hand are innervated by the radial nerve. While radial nerve
injuries may occur in anterior shoulder dislocations, they are less common than axillary nerve

www.eduwaves360.com | Telegram : @eduwaves360


injuries. Radial nerve palsy is usually due to its compression in the axilla or upper arm ('Saturday
night palsy') and fractures of the humeral midshaft.

www.eduwaves360.com | Telegram : @eduwaves360


Question # 37

A 21-year-old man comes to the physician for a follow-up examination. Four days ago, he injured his
right knee while playing soccer. Increased laxity of the right knee joint is noted when the knee is flexed
to 30° and an abducting force is applied to the lower leg. The examination finding in this patient is most
likely caused by damage to which of the following structures?

Answer Image

A Posterior cruciate ligament

B Medial meniscus

C Lateral collateral ligament

D Lateral meniscus

E Medial collateral ligament

F Anterior cruciate ligament

www.eduwaves360.com | Telegram : @eduwaves360


Hint

The patient's injury is likely to be one of the injuries that comprise the unhappy triad.

www.eduwaves360.com | Telegram : @eduwaves360


Correct Answer

A - Posterior cruciate ligament

Explanation Why

A posterior cruciate ligament tear would cause increased joint laxity when the knee is flexed and a
posterior translating force is applied to the lower leg (posterior drawer sign), unlike the findings in
this patient.

B - Medial meniscus

Explanation But

The medial meniscus is often injured along with the structure that is injured in this patient.

Explanation Why

A tear in the medial meniscus would cause knee pain when the knee is flexed and the lower leg is
externally rotated, unlike the findings in this patient.

C - Lateral collateral ligament

Explanation Why

A tear in the lateral collateral ligament would cause increased joint laxity when the knee is flexed
and an adducting force is applied to the lower leg (varus stress test), unlike the findings in this
patient.

www.eduwaves360.com | Telegram : @eduwaves360


D - Lateral meniscus

Explanation Why

A tear in the lateral meniscus would cause knee pain when the knee is flexed and the lower leg is
internally rotated, unlike the findings in this patient.

E - Medial collateral ligament

Image

Explanation Why

Injury to the medial collateral ligament would cause increased laxity of the knee joint when the knee
is flexed and an abducting force is applied to the lower leg (valgus stress test). Medial collateral
ligament injury is often associated with injury to the anterior cruciate ligament and medial meniscus
(unhappy triad).

www.eduwaves360.com | Telegram : @eduwaves360


F - Anterior cruciate ligament

Explanation But

The anterior cruciate ligament is often injured along with the structure that is injured in this patient.

Explanation Why

An anterior cruciate ligament tear would cause increased joint laxity when the knee is flexed and an
anterior translating force is applied to the lower leg (anterior drawer sign and Lachman sign), unlike
the findings in this patient.

www.eduwaves360.com | Telegram : @eduwaves360


Question # 38

A 59-year-old woman comes to the physician because of a 1-year history of pain and stiffness in her
fingers and knees. The stiffness lasts for about 10 minutes after she wakes up in the morning. She also
reports that her knee pain is worse in the evening. She drinks one glass of wine daily. Her only
medication is acetaminophen. She is 175 cm (5 ft 9 in) tall and weighs 102 kg (225 lb); BMI is
33 kg/m2. Physical examination shows firm nodules on the distal interphalangeal joints of the index,
ring, and little fingers of both hands. Which of the following is the most likely diagnosis?

Answer Image

A Pseudogout

B Rheumatoid arthritis

C Gout

D Septic arthritis

www.eduwaves360.com | Telegram : @eduwaves360


Answer Image

E Osteoarthritis

www.eduwaves360.com | Telegram : @eduwaves360


Hint

X-ray of the involved joints would most likely show decreased joint space, osteophytes, and subchondral
sclerosis.

www.eduwaves360.com | Telegram : @eduwaves360


Correct Answer

A - Pseudogout

Explanation Why

Pseudogout is joint inflammation due to calcium pyrophosphate crystal deposition. Chronic CPPD
can cause progressively worsening pain affecting large and small joints. However, the nodules seen
in this patient would not be present in CPPD.

B - Rheumatoid arthritis

Image

Explanation Why

Rheumatoid arthritis (RA) predominantly occurs among women and can affect both large and small
joints. It presents as a symmetrical arthropathy with joint pain and early morning stiffness and can

www.eduwaves360.com | Telegram : @eduwaves360


cause rheumatoid nodules in the affected small joints, as seen here. However, in patients with RA,
morning joint stiffness characteristically lasts for > 30 minutes, and distal interphalangeal joints
(DIP) are not as frequently affected.

C - Gout

Image

Explanation Why

Gout can cause joint pain and firm nodular swellings at the affected joints (gouty tophi). This patient
also has a few risk factors for gout (e.g., advanced age, obesity, alcohol consumption). However,
gout arthropathy is typically acute, asymmetric, and monoarticular, unlike this patient's chronic,
symmetric polyarthritis with early morning stiffness.

www.eduwaves360.com | Telegram : @eduwaves360


D - Septic arthritis

Explanation Why

Septic arthritis causes joint pain. However, it has an acute onset, typically affects a single joint, and is
characterized by signs of inflammation (erythema, edema, warmth), unlike in this patient.

E - Osteoarthritis

Image

Explanation Why

This patient presents with symptoms characteristic of osteoarthritis (OA). She has pain in weight-
bearing joints (knees) that is worse in the evening (due to ambulation during the day). After
prolonged immobility, she experiences joint stiffness lasting < 30 minutes. She also has distal
interphalangeal joint nodules (Heberden's nodes). Obesity and advanced age are known risk factors
for OA.

www.eduwaves360.com | Telegram : @eduwaves360


www.eduwaves360.com | Telegram : @eduwaves360
Question # 39

A 51-year-old woman comes to the emergency department because of a 1-day history of severe pain in
her left knee. To lose weight, she recently started jogging for 30 minutes a few times per week. She has
type 2 diabetes mellitus and hypertension treated with metformin and chlorothiazide. Her sister has
rheumatoid arthritis. She is sexually active with two partners and uses condoms inconsistently. On
examination, her temperature is 38.5°C (101.3°F), pulse is 88/min, and blood pressure is 138/87 mm Hg.
The left knee is swollen and tender to palpation with a significantly impaired range of motion. A 1.5-cm,
painless ulcer is seen on the plantar surface of the left foot. Which of the following is most likely to help
establish the diagnosis?

Answer Image

A Perform MRI of the knee

B Perform arthrocentesis

www.eduwaves360.com | Telegram : @eduwaves360


Answer Image

C Measure HLA-B27

D Measure rheumatoid factor

E Perform ultrasonography of the knee

F Measure serum uric acid levels

www.eduwaves360.com | Telegram : @eduwaves360


Hint

Permanent joint destruction is a dreaded complication of this patient's condition.

www.eduwaves360.com | Telegram : @eduwaves360


Correct Answer

A - Perform MRI of the knee

Image

Explanation Why

MRI of the knee is the imaging modality of choice for the diagnosis of occult meniscal tears, which
can arise from knee injuries or degenerative processes. Although there is no history of recent knee
trauma, this patient recently started exercising and she could have suffered a previously undiagnosed
meniscal tear. However, meniscal tears cannot explain this patient's fever, and in the presence of a
distant wound site, another diagnosis would be more likely.

www.eduwaves360.com | Telegram : @eduwaves360


B - Perform arthrocentesis

Image

Explanation But

Following arthrocentesis and culture, empiric treatment for septic arthritis (parenteral antibiotics
based on the Gram stain) should be initiated immediately to avoid severe joint damage, which is
primarily caused by bacterial invasion of cartilage.

Explanation Why

Arthrocentesis is the best next step in this patient with typical symptoms of septic arthritis, including
fever, joint pain, and restricted range of motion. The knee is a commonly affected joint in septic
arthritis, which is most commonly caused by hematogenous spread of bacteria (most commonly
Staphylococcus aureus) from a distant wound site, such as an infected diabetic ulcer. Typical
findings in arthrocentesis include a synovial fluid WBC count of over 50,000/μL with dominance of
polymorphonuclear cells (PMN).

www.eduwaves360.com | Telegram : @eduwaves360


C - Measure HLA-B27

Image

Explanation Why

HLA-B27 can aid in the diagnosis of reactive arthritis, which usually presents with fever, malaise,
and lower extremity joint pain. This patient's inconsistent use of condoms puts her at risk for
infection with Chlamydia trachomatis, which can cause reactive arthritis. However, it typically
presents as asymmetrical polyarthritis that is often combined with sacroiliitis and extra‑articular
symptoms (e.g., conjunctivitis or anterior uveitis), which this patient does not have. Furthermore, this
patient lacks symptoms of a preceding infection such as diarrhea or dysuria.

D - Measure rheumatoid factor

Explanation Why

Rheumatoid factor test can aid in the diagnosis of rheumatoid arthritis (RA). This patient's family

www.eduwaves360.com | Telegram : @eduwaves360


history puts her at an increased risk for RA, which may also affect the knee joints. However, it would
typically present with symmetrical polyarthritis of the finger and wrist joints, morning stiffness, and
constitutional symptoms, which this patient does not have.

E - Perform ultrasonography of the knee

Explanation Why

Ultrasonography of the knee can aid in the diagnosis of Osgood-Schlatter disease, which presents
with anterior knee pain worsened by activities such as running and jumping. However, this diagnosis
is not associated with fever, or limited range of motion, and occurs mostly in boys 9–14 years of age.

F - Measure serum uric acid levels

Image

www.eduwaves360.com | Telegram : @eduwaves360


Explanation Why

Serum uric acid levels can be elevated in patients with gout. This patient's use of thiazide diuretics,
weight, and diabetes mellitus put her at increased risk for gout. She also has some typical findings of
an acute gout attack such as joint pain, swelling, and restricted range of motion. However, the
presence of a distant wound site makes another diagnosis more likely. Furthermore, serum uric acid
levels can be normal, raised, or even low during an acute gouty attack. Therefore, measuring serum
uric acid levels may not be useful in diagnosis.

www.eduwaves360.com | Telegram : @eduwaves360


Question # 40

A 31-year-old woman comes to the physician because of a 2-day history of low-grade intermittent fever,
dyspnea, and chest pain that worsens on deep inspiration. Over the past 4 weeks, she has had pain in her
wrists and the fingers of both hands. During this period, she has also had difficulties working on her
computer due to limited range of motion in her fingers, which tends to be more severe in the morning.
Her temperature is 37.7°C (99.8°F). Physical examination shows a high-pitched scratching sound over
the left sternal border. Further evaluation of this patient is most likely to reveal which of the following
findings?

Answer Image

A Mutation of the HFE gene

Blood urea nitrogen level > 60


B
mg/dL

Decreased C3 complement
C
levels

Increased titer of anti-


D
citrullinated peptide antibodies

E Coxsackie virus RNA

www.eduwaves360.com | Telegram : @eduwaves360


Hint

This patient is most likely presenting with acute pericarditis (chest pain, dyspnea, high-pitched
scratching heart sound) following a month-long history of symptoms of rheumatoid arthritis
(symmetrical small joint pain of the wrists and fingers, morning stiffness).

www.eduwaves360.com | Telegram : @eduwaves360


Correct Answer

A - Mutation of the HFE gene

Explanation Why

Mutation of the HFE gene causes hemochromatosis, which can manifest with symmetrical
polyarthralgias. Hemochromatosis-related cardiomyopathies, such as dilated cardiomyopathy and
restrictive cardiomyopathy, may occur, but acute pericarditis is not a feature of hemochromatosis.
Moreover, this patient lacks other features of this condition, such as diabetes, hyperpigmented skin
(“bronze diabetes”), and cirrhosis.

B - Blood urea nitrogen level > 60 mg/dL

Explanation Why

Elevated BUN levels indicate uremia, typically secondary to chronic kidney disease (CKD). Uremia
can cause pericarditis, and bone pain may be present as a consequence of CKD-mineral and bone
disorder (CKD-MBD). However, the bone pain in CKD is generalized in nature. Moreover, this
patient lacks other signs of late-stage renal failure, such as edema and oliguria.

C - Decreased C3 complement levels

Explanation Why

Decreased C3 complement levels are typical in patients with systemic lupus erythematosus (SLE).
SLE is a female-predominant disease and can manifest with symmetric small joint arthralgia,
morning stiffness, and pericarditis. However, lupus-associated arthralgias are migratory in nature and
joint stiffness would not be prolonged, usually lasting only minutes. Moreover, this patient is lacking
other features of SLE, such as a malar or discoid rash and oral ulcers.

www.eduwaves360.com | Telegram : @eduwaves360


D - Increased titer of anti-citrullinated peptide antibodies

Image

Explanation Why

Anti-CCP titers are used to diagnose rheumatoid arthritis (RA). The sensitivity of anti-CCPs is
similar to that of rheumatoid factor (∼ 60%) and their specificity is higher (> 90%).

E - Coxsackie virus RNA

Explanation Why

Coxsackie virus RNA on PCR is diagnostic for a coxsackie infection. The picornavirus coxsackie B
is the most common cause of viral pericarditis. Coxsackie infection may be accompanied by
generalized arthralgia and myalgia. It does not commonly manifest with small joint arthritis and
morning stiffness. Moreover, this patient's arthralgia began weeks before her symptoms of
pericarditis.

www.eduwaves360.com | Telegram : @eduwaves360


www.eduwaves360.com | Telegram : @eduwaves360
www.eduwaves360.com | Telegram : @eduwaves360
Join us on Telegram :

Click here : @eduwaves360

Unlocked the Medical premiums

Click here : www.eduwaves360.com

Medical Courses : https://t.me/usmle_study_materials_2

Discussion Group : @usmle_discussion_group

www.eduwaves360.com | Telegram : @eduwaves360


Question # 1

A 21-year-old man comes to the military base physician for evaluation of progressive discomfort in his
right shoulder for the past 4 months. He joined the military 6 months ago and is part of a drill team. In
anticipation of an upcoming competition, he has been practicing rifle drills and firing exercises 8 hours a
day. Physical examination shows tenderness to palpation and a firm mass in the superior part of the right
deltopectoral groove. Range of motion is limited by pain and stiffness. Which of the following is the
most likely diagnosis?

Answer Image

A Osteoid osteoma

B Lipoma

C Acromioclavicular joint separation

D Clavicle stress fracture

www.eduwaves360.com | Telegram : @eduwaves360


Answer Image

E Myositis ossificans

F Arteriovenous fistula

www.eduwaves360.com | Telegram : @eduwaves360


Hint

This patient's presentation indicates a condition involving metaplasia of the connective tissue.

www.eduwaves360.com | Telegram : @eduwaves360


Correct Answer

A - Osteoid osteoma

Explanation Why

Osteoid osteomas typically affect boys and young men (10–30 years of age) and can cause chronic
pain. However, osteoid osteomas are not associated with muscle overuse or repetitive trauma.
Moreover, osteoid osteomas occur in the cortex of the diaphysis of long bones and would not explain
this patient's palpable mass in the deltopectoral groove.

B - Lipoma

Image

Explanation Why

Lipomas may present as a palpable mass of the deltopectoral groove, which is seen in this patient.

www.eduwaves360.com | Telegram : @eduwaves360


However, these benign tumors are typically soft and nontender, not firm and tender to palpation like
the one seen here. Moreover, lipomas are not related to muscle overuse and repetitive trauma, such as
the rifle drills and firing exercises reported by this patient, and they would not explain this patient's
limited range of motion.

C - Acromioclavicular joint separation

Explanation Why

An acromioclavicular (AC) joint separation is typically caused by direct force to the point of the
shoulder (e.g., a direct fall). Clinical features include shoulder pain, tenderness over the AC joint,
and limited range of motion, all of which are seen here. However, AC joint separation typically
occurs acutely (i.e., immediately after trauma) rather than over the course of several months.
Moreover, physical examination can show a palpable elevation of the clavicle, but this elevation
would not explain the palpable mass in the deltopectoral groove.

www.eduwaves360.com | Telegram : @eduwaves360


D - Clavicle stress fracture

Image

Explanation Why

Clavicle stress fractures may occur because of overuse and repetitive trauma of the shoulder, such as
the rifle drills and firing exercises reported by this patient. Clinical features include shoulder pain,
tenderness to palpation, and limited range of motion. Clavicle fracture would not, however, explain
this patient's mass in the deltopectoral groove. Instead, sagging and shortening of the shoulder as
well as tenting of the skin over the fracture site would be seen.

www.eduwaves360.com | Telegram : @eduwaves360


E - Myositis ossificans

Image

Explanation Why

Myositis ossificans is caused by heterotopic ossification of the muscle and soft tissue. This condition
can be genetic but typically occurs as a result of muscle overuse, trauma, neurologic conditions, or
chronic degenerative diseases. This patient's history of performing rifle drills and firing exercises
likely resulted in repetitive trauma to the deltopectoral groove (due to recoil from the rifle),
predisposing him to myositis ossificans. Clinical features include chronic pain, stiffness, restricted
range of motion, and possibly palpable masses on physical examination, all of which are seen here.

F - Arteriovenous fistula

Explanation Why

Arteriovenous (AV) fistulas can occur after repetitive trauma, such as the rifle drills and firing

www.eduwaves360.com | Telegram : @eduwaves360


exercises reported by this patient. However, AV fistulas are typically soft and compressible, unlike
the firm and tender mass seen here. Moreover, AV fistulas are not typically located in the
deltopectoral groove and would not explain this patient's limited range of motion. The most common
AV fistulas in the arm are iatrogenic fistulas serving as access points for hemodialysis.

www.eduwaves360.com | Telegram : @eduwaves360


Question # 2

A 26-year-old man comes to the physician because of fever and right leg pain and swelling for the past
3 days. Two weeks ago, he had a short episode of abdominal pain and diarrhea that resolved without
treatment. He is an avid hunter and went hunting for wild boar with his father 3 weeks ago. He took
some of the meat home for consumption. His temperature is 38.6°C (101.5°F). Examination shows
periorbital edema and conjunctival hemorrhage. Palpation of the right calf elicits tenderness. Range of
motion of the right ankle is restricted by pain. A sample of which of the following is most likely to show
the causal pathogen?

Answer Image

A Muscle

B Blood

C Stool

D Lymph node

E Skin

www.eduwaves360.com | Telegram : @eduwaves360


Hint

An episode of abdominal pain and diarrhea followed by fever, leg pain and swelling, periorbital edema
and conjunctival hemorrhage in a patient who consumed wild boar meat suggests trichinellosis.

www.eduwaves360.com | Telegram : @eduwaves360


Correct Answer

A - Muscle

Image

Explanation Why

Trichinellosis is a helminth infection most commonly caused by ingesting the encysted larvae of
Trichinella spiralis from undercooked meat (especially pork). The larvae invade the small bowel
mucosa, where they develop into adult roundworms and may cause intestinal symptoms. New larvae
released by adult worms then migrate to striated muscles, where they encyst and cause myositis,
often leading to severe muscle pain, as seen here. T. spiralis larvae are most likely to be seen in a
sample of muscle tissue from the involved portion of this patient's right calf.

www.eduwaves360.com | Telegram : @eduwaves360


B - Blood

Explanation Why

A blood sample can be useful in the diagnosis of helminth infections, including trichinellosis. A
typical (albeit unspecific) laboratory abnormality associated with helminth infections is eosinophilia.
Trichinellosis also manifests with leukocytosis, elevated creatinine kinase, and specific antibodies,
which are usually used to confirm the diagnosis. However, Trichinella spiralis larvae cannot be
visualized in a blood sample.

C - Stool

Explanation Why

A stool sample can be useful for direct identification of helminths such as Strongyloides stercoralis.
Strongyloidiasis can manifest with fever, abdominal pain, and diarrhea. However, it is also
associated with skin lesions (e.g., larva currens) and respiratory symptoms, neither of which are
observed in this patient. Also, fever, severe leg pain and swelling, conjunctival hemorrhage, and
periorbital edema are inconsistent with this diagnosis. Trichinella spiralis larvae cannot be visualized
in a stool sample.

D - Lymph node

Explanation Why

A lymph node tissue sample can be useful for direct identification of helminths such as Wuchereria
bancrofti, a common cause of lymphatic filariasis. Lymphatic filariasis can manifest with fever
(acute infection) and lower extremity swelling due to lymphedema (chronic infection). However, this
condition manifests with painful lymphadenopathy rather than gastrointestinal symptoms, leg pain,
periorbital edema, and conjunctival hemorrhage. Trichinella spiralis larvae cannot be visualized in a
lymph node tissue sample.

www.eduwaves360.com | Telegram : @eduwaves360


E - Skin

Explanation Why

A skin sample can be useful for direct identification of helminths like Enterobius vermicularis (tape
test) and Onchocerca volvulus (skin snip test). Neither infection is associated with fever, severe leg
pain and swelling, conjunctival hemorrhage, or periorbital edema. Trichinella spiralis larvae cannot
be visualized in a skin sample.

www.eduwaves360.com | Telegram : @eduwaves360


Question # 3

A 65-year-old woman comes to the physician because of increased difficulty hearing. She has also had
dull and progressive pain in her hip and lower back for the past 2 months that is worse with exertion.
Examination of the ears shows impaired hearing on the left with whispered voice test and lateralization
to the right with Weber testing. There is localized tenderness over the right hip and groin area with
decreased range of motion of the hip. The remainder of the examination shows no abnormalities. Serum
studies show:

Total protein 6.5 g/dL

Alkaline phosphatase 950 U/L

Calcium 9 mg/dL

Phosphorus 4 mg/dL

Which of the following is the most likely underlying mechanism of this patient's symptoms?

Answer Image

A Proliferation of plasma cells in the bone marrow

B Decreased bone mass with microarchitectural disruption

C Defective bone matrix mineralization

D Increased rate of bone remodeling

E Metastatic destruction of the bone

www.eduwaves360.com | Telegram : @eduwaves360


Hint

This patient has bone pain, signs of sensorineural hearing loss (lateralization of the Weber test), and an
elevated alkaline phosphatase. This combination of findings is suggestive of Paget disease of the bone.

www.eduwaves360.com | Telegram : @eduwaves360


Correct Answer

A - Proliferation of plasma cells in the bone marrow

Explanation Why

Abnormal proliferation of plasma cells in the bone marrow is seen in patients with multiple myeloma
(MM), which commonly presents with bone pain and radiographic findings of osteolytic lesions,
osteopenia, and/or pathologic fractures. Classic laboratory findings include elevated total protein,
anemia, elevated creatinine, and hypercalcemia. While this patient does have bone pain, she does not
have any of the other features of MM.

B - Decreased bone mass with microarchitectural disruption

Explanation Why

Decreased bone mass and microarchitectural disruption are characteristic of osteoporosis, which is
classically diagnosed either on routine screening or after a pathologic fracture has occurred.
Commonly affected sites include the lumbar spine, femoral head, distal radius, and humerus.
Although age and gender are risk factors for this patient, laboratory findings are usually within the
normal range, unlike those of this patient, who has elevated alkaline phosphatase.

C - Defective bone matrix mineralization

Explanation Why

Defective bone matrix mineralization is seen in patients with osteomalacia, a bone disorder that is
most commonly due to vitamin D deficiency but may also arise secondary to proximal renal tubular
acidosis, hypophosphatemia, or low dietary calcium intake. The presentation and associated
laboratory findings of patients with osteomalacia depend largely on the underlying etiology, but
generally, osteomalacia manifests as diffuse bone pain, muscle weakness, and/or pathologic
fractures. Imaging is non-specific and often reveals low bone mineral density and cortical thinning.
This patient's isolated elevation of alkaline phosphatase is not consistent with this diagnosis.

www.eduwaves360.com | Telegram : @eduwaves360


D - Increased rate of bone remodeling

Explanation Why

This patient has Paget disease of bone (PDB), a relatively common, benign condition that is likely
hereditary and characterized by accelerated bone turnover, which results in a disorganized bone
matrix with abnormal woven bone interspersed with lamellar bone. While most patients are
asymptomatic, it may manifest with hearing impairment (due to auditory canal narrowing) and bone
pain, as seen here. This patient's isolated elevation of alkaline phosphatase is consistent with the
diagnosis; calcium, phosphorus, and PTH levels are usually normal in patients with PDB. While
osteolytic bone lesions may be seen earlier in the course of the disease, later it is dominated by
sclerotic bone with cortical thickening (due to osteoblast activity), and microfissures and fractures
are also common. Patients with PDB are at increased risk of high-output heart failure (due to
increased blood flow from arteriovenous shunts formed during bone remodeling) and osteogenic
sarcoma.

E - Metastatic destruction of the bone

Explanation Why

Cancers that commonly metastasize to bone include prostate cancer, breast cancer, RCC, thyroid
cancer, and lung cancer. This patient's age is a risk factor for cancer but the location of her symptoms
(e.g., cranial nerve involvement) and isolated elevation in alkaline phosphatase make a different
diagnosis more likely.

www.eduwaves360.com | Telegram : @eduwaves360


Question # 4

An 18-year-old man is brought to the emergency department 30 minutes after being stabbed in the chest
during a fight. He has no other injuries. His pulse is 120/min, blood pressure is 90/60 mm Hg, and
respirations are 22/min. Examination shows a 4-cm deep, straight stab wound in the 4th intercostal space
2 cm medial to the right midclavicular line. The knife most likely passed through which of the following
structures?

Answer Image

Pectoral fascia, transversus


A
thoracis muscle, right lung

Intercostal muscles, internal


B
thoracic artery, right heart

Serratus anterior muscle, pleura,


C
inferior vena cava

www.eduwaves360.com | Telegram : @eduwaves360


Answer Image

External oblique muscle, superior


D
epigastric artery, azygos vein

Pectoralis minor muscle, dome of


E the diaphragm, right lobe of the
liver

www.eduwaves360.com | Telegram : @eduwaves360


Hint

www.eduwaves360.com | Telegram : @eduwaves360


Correct Answer

A - Pectoral fascia, transversus thoracis muscle, right lung

Explanation Why

The pectoralis major, sheathed in its fascia, spans the anterior chest wall, originating from the medial
clavicle, anterior sternum, and six upper costal cartilages, and inserting into the proximal humerus.
The transversus thoracis muscles lie on the internal surface of the thoracic cage, originating from the
sternum and spanning out laterally to the 2nd to 6th ribs. The pectoralis fascia, transversus thoracis,
and right lung would be intersected by a line originating at a point 2 cm medial to the midclavicular
line.

B - Intercostal muscles, internal thoracic artery, right heart

Image

www.eduwaves360.com | Telegram : @eduwaves360


Explanation Why

The internal thoracic arteries (internal mammary arteries) runs parallel to the sternum on either side,
just deep to the costal cartilage. To pass through an intercostal muscle, an internal mammary artery,
and the right heart, a knife would have to enter just adjacent to the right border of the sternum. The
stab wound in this case, however, lies further laterally, near the right midclavicular line.

C - Serratus anterior muscle, pleura, inferior vena cava

Image

Explanation Why

The serratus anterior muscle spans the lateral wall of the thoracic cage. The inferior vena cava enters
the thoracic cavity through the diaphragm at the level of the T8 vertebra and lies adjacent to the
spine. A knife through any intercostal space will always pierce the parietal pleura but a 4-cm stab
wound in an adult in the midclavicular line will pierce neither the serratus anterior nor the inferior
vena cava.

www.eduwaves360.com | Telegram : @eduwaves360


D - External oblique muscle, superior epigastric artery, azygos vein

Image

Explanation Why

The superior epigastric artery begins inferior to the sternum as a continuation of the internal thoracic
artery (internal mammary artery) and lies near the midline in the epigastric and umbilical region
within the rectus sheath, which is partly formed by the aponeurosis of the external oblique muscle.
The azygos vein lies on the right posterior chest wall in a groove just lateral to the spine. These
structures are too far apart in an adult to be penetrated by a 4-cm knife.

E - Pectoralis minor muscle, dome of the diaphragm, right lobe of the liver

Explanation Why

At the end of expiration, the right diaphragmatic dome would be present at the level of the right 5th
intercostal space in the midclavicular line and the liver lies just inferior to the right diaphragmatic

www.eduwaves360.com | Telegram : @eduwaves360


dome. A stab wound in the 4th intercostal space in the midclavicular line could, therefore, potentially
pierce the dome of the diaphragm and the right lobe of the liver if the stab wound if angulated
inferiorly. However, the pectoralis minor muscle originates lateral to the midclavicular line from the
3rd, 4th, and 5th ribs. A stab wound in the 4th intercostal space medial to the midclavicular line would
pierce the pectoralis major but it is unlikely to pierce the pectoralis minor.

www.eduwaves360.com | Telegram : @eduwaves360


Question # 5

A 70-year-old man is brought to the emergency department because of severe back pain that began when
he was lifting a box 1 hour ago. He also has a 2-year history of increasingly severe right hip pain.
Physical examination shows tenderness to palpation of the lower spine as well as erythema of the skin
over the right hip. Neurologic examination shows decreased hearing in the left ear; the Weber test
localizes to the left side. Serum studies show an alkaline phosphatase concentration of 410 U/L, calcium
concentration of 9.5 mg/dL, and parathyroid hormone level of 32 pg/mL. An x-ray of the spine shows a
fracture of the L4 vertebra. Which of the following is the most likely diagnosis?

Answer Image

A Osteitis fibrosa cystica

B Osteoporosis

C Osteomalacia

D Osteitis deformans

E Osteonecrosis

F Osteopetrosis

www.eduwaves360.com | Telegram : @eduwaves360


Hint

This patient has a pathologic fracture of the spine, chronic hip pain, impaired hearing, and an elevated
serum alkaline phosphatase concentration (with normal serum calcium and parathormone).

www.eduwaves360.com | Telegram : @eduwaves360


Correct Answer

A - Osteitis fibrosa cystica

Explanation Why

Osteitis fibrosa cystica is a manifestation of unopposed osteoclast activation in patients with severe
hyperparathyroidism. Affected patients could potentially present with bone pain and elevated serum
alkaline phosphatase concentrations, which are seen here. However, serum calcium and parathyroid
hormone (PTH), and phosphate concentrations would usually be abnormal. Furthermore, osteitis
fibrosa cystica would not explain the hearing loss or the erythema over the area of bone pain seen in
this patient.

B - Osteoporosis

Explanation Why

Osteoporosis is characterized by a decrease in bone mineral density and can manifest with
pathological fractures. However, this condition is more common in women and does not cause
hearing loss, long-standing bone pain, erythema, or elevated serum alkaline phosphatase
concentrations.

C - Osteomalacia

Explanation Why

Osteomalacia is characterized by impaired mineralization of the bone (secondary to vitamin D


deficiency) and can manifest with bone pain, pathological fractures, and elevated serum alkaline
phosphate concentrations. However, this condition does not typically cause hearing loss or erythema
over areas of bone pain. In addition, patients with osteomalacia would have decreased serum calcium
and phosphate concentrations.

www.eduwaves360.com | Telegram : @eduwaves360


D - Osteitis deformans

Image

Explanation Why

Osteitis deformans is another name for Paget disease of bone, an idiopathic disease associated with
increased bone remodeling. The bone remodeling is caused by increased activity of osteoblasts and
osteoclasts, which leads to the formation of disorganized (woven) bone that is weaker than normal
bone and abnormal in structure. Paget disease of bone therefore manifests with pathological
fractures, bone pain with overlying erythema (due to increased vascularity of the underlying bone),
and hearing loss (due to ankylosis of the ossicles or compression of the vestibulocochlear nerve).
Elevated serum alkaline phosphatase concentrations with normal serum calcium and phosphate
concentrations, as seen in this patient, are a hallmark laboratory finding.

www.eduwaves360.com | Telegram : @eduwaves360


E - Osteonecrosis

Explanation Why

Osteonecrosis results from the interruption of blood supply to a bone, most commonly associated
with excessive alcohol consumption or corticosteroid therapy. The long-standing hip pain in this
patient could be explained by femoral head osteonecrosis, but not the overlying erythema. Moreover,
this condition would not explain this patient's hearing loss or the acute onset of back pain while
attempting to lift a box. Furthermore, osteonecrosis does not typically cause elevated serum alkaline
phosphatase concentrations, which are seen in this patient.

F - Osteopetrosis

Explanation But

Although an elevated alkaline phosphatase value is not typically associated with osteopetrosis, it
could be observed in the specific situation where a patient with osteopetrosis has just had a
pathological fracture.

Explanation Why

Osteopetrosis is characterized by increased bone mineralization and can manifest with bone pain,
pathological fractures, and hearing loss (due to sclerosis in the skull). However, this condition is a
genetically-inherited bone disease and typically manifests at a much earlier age. Furthermore, long-
standing osteopetrosis would be characterized by pancytopenia (due to bone marrow replacement)
and hepatosplenomegaly (due to extramedullary hematopoiesis), neither of which are seen in this
case. Moreover, this condition does not typically cause erythema over areas of bone pain.

www.eduwaves360.com | Telegram : @eduwaves360


Question # 6

An investigator is working with a mutant strain of mice that lack a consistent density of sarcolemmal
transverse tubules in the skeletal muscle cells. Which of the following is the most likely associated
finding as a result of this abnormality?

Answer Image

Increased
activation of
A myosin-light-
chain
phosphatase

Decreased entry
of calcium at
B
the presynaptic
membrane

Increased
accumulation of
C mitochondria in
subsarcolemmal
region

Decreased
expression of
D sarcolemmal
Na+/K+ ATPase

www.eduwaves360.com | Telegram : @eduwaves360


Answer Image

Impaired
synchronization
E
of cross-bridge
formation

Impaired
binding of
acetylcholine to
F
nicotinic
acetylcholine
receptors

www.eduwaves360.com | Telegram : @eduwaves360


Hint

The sarcolemma is analogous to the membrane of skeletal and cardiac muscle cells.

www.eduwaves360.com | Telegram : @eduwaves360


Correct Answer

A - Increased activation of myosin-light-chain phosphatase

Explanation Why

Myosin light-chain phosphatase (MLCP) is only present in smooth muscle and is activated by the
presence of nitric oxide or cyclic GMP. MLCP is not present in skeletal muscle.

B - Decreased entry of calcium at the presynaptic membrane

Image

Explanation Why

Decreased presynaptic calcium entry is seen in Lambert-Eaton myasthenic syndrome because of a


decreased number of functional presynaptic voltage-gated calcium channels. The binding of
autoantibodies specific to these calcium channels impairs calcium influx presynaptically. In contrast,

www.eduwaves360.com | Telegram : @eduwaves360


a differential density of sarcolemmal T tubules would affect calcium release postsynaptically within
the myocyte.

C - Increased accumulation of mitochondria in subsarcolemmal region

Explanation Why

Increased accumulation of subsarcolemmal mitochondria is characteristic of mitochondrial


myopathies, such as myoclonic epilepsy. This accumulation contributes to the classic, so-called
“ragged red” appearance of affected myocytes on histological analysis. A lack of a consistent density
of sarcolemmal T tubules, however, is not a feature of mitochondrial myopathies.

D - Decreased expression of sarcolemmal Na+/K+ ATPase

Explanation Why

Decreased expression of sarcolemmal Na+-K+ ATPase is characteristic of muscle atrophy. A lack of


a consistent density of sarcolemmal T tubules is not associated with muscle atrophy.

www.eduwaves360.com | Telegram : @eduwaves360


E - Impaired synchronization of cross-bridge formation

Image

Explanation But

The system of T tubules is only present in skeletal and cardiac muscle.

Explanation Why

Transverse tubules (T tubules) are invaginations of the sarcolemma. The sarcoplasmic reticulum,
which stores the calcium needed for muscle contraction, has enlarged terminal cisterns that are in
contact with T tubules at the junction of the A band and I band. As a result of this coupling between
the sarcoplasmic reticulum and T tubules, depolarization of the T tubule leads to calcium release
from the sarcoplasmic reticulum. The binding of calcium to troponin C shifts tropomyosin away
from the myosin-binding sites on the actin filament. This shift allows the myosin head to bind to
actin, leading to the formation of a cross-bridge with this final step facilitating sarcomere
contraction. The simultaneous release of calcium is required for the synchronized contraction of all
sarcomeres within a myocyte and is mediated by the specific spatial relationship of the T tubules to
the sarcoplasmic reticulum. A differential density of T tubules within a skeletal myocyte would,
therefore, lead to desynchronized cross-bridge formation.

www.eduwaves360.com | Telegram : @eduwaves360


F - Impaired binding of acetylcholine to nicotinic acetylcholine receptors

Image

Explanation Why

Impaired binding of acetylcholine to nicotinic acetylcholine receptors on the motor end plate is seen
in myasthenia gravis (MG). The binding of autoantibodies specific to this receptor causes a reduction
in the number of functional postsynaptic nicotinic acetylcholine receptors. Consequently, there is
inadequate stimulation of the postsynaptic nicotinic acetylcholine receptors, accounting for the
characteristic symptoms of MG (i.e., ptosis, diplopia, and weakness). However, a differential density
of sarcolemmal T tubules would not affect the binding of acetylcholine to nicotinic acetylcholine
receptors.

www.eduwaves360.com | Telegram : @eduwaves360


Question # 7

An investigator is studying the crossbridge cycle of muscle contraction. Tissue from the biceps brachii
muscle is obtained at the autopsy of an 87-year-old man. Investigation of the muscle tissue shows
myosin heads attached to actin filaments. Binding of myosin heads to which of the following elements
would most likely cause detachment of myosin from actin filaments?

Answer Image

A cGMP

B Calcium

C Troponin C

D ATP

E Calmodulin

F Tropomyosin

G ADP

www.eduwaves360.com | Telegram : @eduwaves360


Hint

Absence of which of the following is the underlying cause of rigor mortis?

www.eduwaves360.com | Telegram : @eduwaves360


Correct Answer

A - cGMP

Explanation Why

cGMP is implicated in the dephosphorylation (inactivation) of myosin light-chain kinase, which


promotes smooth-muscle relaxation and inhibits contraction. However, cGMP does not bind to
myosin heads and it is not involved in the detachment of the actin-myosin crossbridges in skeletal
muscle (e.g., the biceps brachii).

B - Calcium

Explanation Why

Calcium is released during muscle activation and binds to troponin C, which allows troponin C to
displace tropomyosin from the myosin binding site of actin. This process allows actin-myosin
crossbridges to form in order to generate muscle contraction. However, calcium is not involved in the
detachment of actin-myosin crossbridges.

C - Troponin C

Explanation Why

Troponin C displaces tropomyosin from the myosin binding site of actin. This process allows
actin-myosin crossbridges to form in order to generate muscle contraction. However, troponin C is
not involved in the detachment of actin-myosin crossbridges.

www.eduwaves360.com | Telegram : @eduwaves360


D - ATP

Image

Explanation But

When a person dies and breathing and circulation stop, muscle cells lack oxygen and cannot use
aerobic respiration to efficiently produce ATP. Without ATP, the crossbridges between myosin and
actin filaments cannot be resolved and muscles become tense, which is referred to as rigor mortis.

Explanation Why

Binding of ATP to myosin heads causes myosin to detach from actin filaments, which disrupts the
crossbridge cycle and ends muscle contraction. Hydrolysis of myosin-bound ATP to ADP and Pi, on
the other hand, alters the conformation of myosin heads to a cocked state, which primes the myosin
for crossbridge formation with actin to generate muscle contraction.

www.eduwaves360.com | Telegram : @eduwaves360


E - Calmodulin

Explanation Why

Calmodulin binds to calcium that is released during smooth-muscle activation to activate myosin
light-chain kinase, which phosphorylates the head of the myosin light chain. This process allows
actin-myosin crossbridges to form in order to generate muscle contraction. However, calmodulin is
not involved in the detachment of actin-myosin crossbridges.

F - Tropomyosin

Explanation Why

Tropomyosin blocks the myosin binding site on actin during muscle relaxation. Tropomyosin is
displaced by troponin C, which is activated by the calcium released during muscle activation. This
process allows actin-myosin crossbridges to form in order to generate muscle contraction. However,
tropomyosin is not involved in the detachment of actin-myosin crossbridges.

G - ADP

Explanation Why

ADP and inorganic phosphate (Pi) are the reaction products of the hydrolysis of ATP. This process
primes myosin for crossbridge formation with actin to generate muscle contraction. However, ADP
is not involved in the detachment of actin-myosin crossbridges.

www.eduwaves360.com | Telegram : @eduwaves360


Question # 8

An investigator is examining tissue samples from various muscle tissues throughout the body. She
notices that biopsies collected from a specific site have a high density of sarcoplasmic reticulum,
mitochondria, and myoglobin; they also stain poorly for ATPase. Electron microscopy of the biopsy
specimens shows that the myocytes do not have gap junctions. These biopsy findings are most likely to
be seen in a specimen obtained from which of the following sites?

Answer Image

A Ventricular myocardium

B Semispinalis muscle

C Glandular myoepithelium

D Myometrium

E Lateral rectus muscle

www.eduwaves360.com | Telegram : @eduwaves360


Hint

The presence of sarcoplasmic reticulum and myoglobin, in addition to the absence of gap junctions,
indicates a skeletal myocyte. Poor ATPase staining (due to low myosin ATPase activity) and a high
density of mitochondria and myoglobin (an oxygen transporter) are typical in a type 1 skeletal muscle
fiber (red fiber).

www.eduwaves360.com | Telegram : @eduwaves360


Correct Answer

A - Ventricular myocardium

Explanation Why

Ventricular myocardium (cardiac muscle) has a high level of mitochondria and myoglobin because it
predominantly uses aerobic respiration for energy production but in contrast to the findings described
above, cardiac myocytes would have gap junctions because fast, coordinated communication
between myocytes is essential for ensuring synchronized contraction in the heart.

B - Semispinalis muscle

Explanation Why

Semispinalis muscle is composed of skeletal muscle with type 1 fibers (red, slow-twitching fibers).
Type 1 fibers are mainly located in high-endurance muscles (e.g., postural muscles), and their
metabolic energy demands are predominantly obtained via aerobic respiration, which explains their
high density of mitochondria and myoglobin; myoglobin also gives type 1 fibers their red
appearance. The low myosin ATPase activity of type 1 fibers means that they are slow switching,
because the speed of sarcomere shortening depends on how quickly myosin can hydrolyze ATP.
Since skeletal myocytes do not have gap junctions, each cell must be stimulated individually via its
motor endplate.

C - Glandular myoepithelium

Explanation Why

Glandular myoepithelium is the thin layer of smooth muscle fibers above the basement membrane
and beneath the luminal cells of exocrine glands. Glandular myoepithelium is a multiunit smooth
muscle, which does not typically contain gap junctions. However, smooth muscle fibers typically do
not have myoglobin or sarcoplasmic reticulum.

www.eduwaves360.com | Telegram : @eduwaves360


D - Myometrium

Explanation Why

Myometrium is the smooth muscle layer in the uterus. This layer consists of single-unit smooth
muscle fibers, which, unlike the findings seen here, possess gap junctions to facilitate coordinated
contraction. In addition, smooth muscle fibers typically do not have myoglobin or sarcoplasmic
reticulum.

E - Lateral rectus muscle

Explanation Why

The lateral rectus muscle, an extraocular muscle, predominantly consists of type 2 skeletal muscle
fibers (white, fast-twitching fibers). Unlike the findings seen here, however, type 2 fibers have high
myosin ATPase activity because they need to hydrolyze ATP quickly and rapidly repeat the muscle
contraction cycle. Also type 2 fibers primarily generate energy by anerobic glycolysis and have a low
density of cell components that are necessary for aerobic respiration (e.g., mitochondria, myoglobin).

www.eduwaves360.com | Telegram : @eduwaves360


Question # 9

A 56-year-old postmenopausal woman comes to the physician because of a 6-month history of


worsening pain and swelling in her left knee. She has a history of peptic ulcer disease for which she
takes cimetidine. Examination shows palpable crepitus and limited range of motion of the left knee.
Which of the following is the most appropriate pharmacotherapy for this patient’s symptoms?

Answer Image

A Ketorolac

B Diclofenac

C Acetylsalicylic acid

D Celecoxib

E Ibuprofen

F Meloxicam

G Indomethacin

www.eduwaves360.com | Telegram : @eduwaves360


Hint

Cyclooxygenase-1 and cyclooxygenase-2 enzymes (COX-1 and COX-2) catalyze the synthesis of
prostaglandins, which decrease gastric acid secretion and increase the protective gastric mucus secretion.
As a result, COX-inhibitors, esp. COX-1 inhibitors can cause gastric ulcers and hemorrhage. They
should, therefore, be avoided in this patient with peptic ulcer disease.

www.eduwaves360.com | Telegram : @eduwaves360


Correct Answer

A - Ketorolac

Explanation Why

Ketorolac is used to treat moderate to severe pain (e.g., postoperative pain relief). However,
ketorolac is a nonselective NSAID that inhibits both COX-1 and COX-2. This inhibition disrupts the
production of protective gastric mucosal prostaglandin, increasing the risk for gastric and intestinal
ulcers and bleeding. Therefore, ketorolac is not a preferred agent in this patient with a history of
gastroduodenal ulcer (PUD).

B - Diclofenac

Explanation Why

Diclofenac is used to treat acute pain and inflammation in rheumatic or nonrheumatoid joint pain.
However, diclofenac is a nonselective NSAID that inhibits both COX-1 and COX-2. This inhibition
disrupts the production of protective gastric mucosal prostaglandin, increasing the risk for gastric and
intestinal ulcers and bleeding. Therefore, diclofenac is not a preferred agent in this patient with a
history of gastroduodenal ulcer (PUD).

C - Acetylsalicylic acid

Explanation Why

Acetylsalicylic acid (aspirin) is used to treat acute pain and inflammation in rheumatic or
nonrheumatoid joint pain. However, aspirin is a nonselective NSAID that inhibits both COX-1 and
COX-2. This inhibition disrupts the production of protective gastric mucosal prostaglandin,
increasing the risk for gastric and intestinal ulcers and bleeding. Therefore, aspirin is not a preferred
agent in this patient with a history of gastroduodenal ulcer (PUD).

www.eduwaves360.com | Telegram : @eduwaves360


D - Celecoxib

Explanation But

Active PUD or gastrointestinal bleeding are contraindications for celecoxib.

Explanation Why

Celecoxib is a selective NSAID and COX-2 inhibitor, which can be used to treat acute pain (equally
in rheumatic arthritis and nonrheumatoid joint pain). Compared to nonselective NSAIDs, COX-2
inhibitors have a reduced gastrointestinal toxicity, but nonetheless carry a minimal risk of
gastrointestinal side effects. Therefore, close supervision of this patient is recommended due to a
residual risk for gastrointestinal bleeds (especially in PUD). Unlike nonselective NSAIDs, selective
COX-2 inhibitors do not have an effect on platelet aggregation, since platelets only possess COX-1.

E - Ibuprofen

Explanation Why

Ibuprofen is used to treat acute pain and inflammation in rheumatic or nonrheumatoid joint pain.
However, ibuprofen is a nonselective NSAID that inhibits both COX-1 and COX-2. This inhibition
disrupts the production of protective gastric mucosal prostaglandin, increasing the risk for gastric and
intestinal ulcers and bleeding. Therefore, ibuprofen is not a preferred agent in this patient with a
history of gastroduodenal ulcer (PUD).

F - Meloxicam

Explanation Why

Meloxicam is used to treat acute pain and inflammation in rheumatic or non-rheumatoid joint pain.
Although meloxicam possesses relatively COX-2 selectivity, it still inhibits COX-1. This inhibition
disrupts the production of protective gastric mucosal prostaglandin, increasing the risk for gastric and
intestinal ulcers and bleeding. Therefore, meloxicam is not a preferred agent in this patient with a

www.eduwaves360.com | Telegram : @eduwaves360


history of gastroduodenal ulcer (PUD).

G - Indomethacin

Explanation Why

Indomethacin is used to treat acute pain and inflammation in rheumatic or nonrheumatoid joint pain.
However, indomethacin is a nonselective NSAID that inhibits both COX-1 and COX-2. This
inhibition disrupts the production of protective gastric mucosal prostaglandin, increasing the risk for
gastric and intestinal ulcers and bleeding. Therefore, indomethacin is not a preferred agent in this
patient with a history of gastroduodenal ulcer (PUD).

www.eduwaves360.com | Telegram : @eduwaves360


Question # 10

A 59-year-old man comes to the emergency department because of excruciating left knee pain for
4 days. He underwent a total knee arthroplasty of his left knee joint 4 months ago. He has hypertension
and osteoarthritis. Current medications include glucosamine, amlodipine, and meloxicam. His
temperature is 38.1°C (100.6°F), pulse is 97/min, and blood pressure is 118/71 mm Hg. Examination
shows a tender, swollen left knee joint; range of motion is limited by pain. Analysis of the synovial fluid
confirms septic arthritis, and the prosthesis is removed. Which of the following is the most likely causal
organism?

Answer Image

A Staphylococcus epidermidis

B Escherichia coli

C Staphylococcus aureus

D Pseudomonas aeruginosa

E Group B Streptococcus

www.eduwaves360.com | Telegram : @eduwaves360


Hint

This patient's presentation with a tender and swollen knee and low-grade fever is consistent with septic
arthritis. His history of arthroplasty makes a bacterial prosthetic joint infection most likely, even
4 months after the operation.

www.eduwaves360.com | Telegram : @eduwaves360


Correct Answer

A - Staphylococcus epidermidis

Explanation But

Treatment of prosthetic joint infections usually consists of removal of the prosthesis and debridement
of the bone and surrounding tissue, followed by IV antibiotics (e.g., oxacillin or ceftriaxone) for 6–8
weeks. Reimplantation of the prosthesis should be performed after repeat negative synovial fluid
cultures or after completion of antimicrobial therapy if there are no clinical signs of infection.

Explanation Why

Staphylococcus epidermidis has the ability to form a biofilm. It can therefore grow on the surface of
foreign material such as prosthetic joints, prosthetic cardiac valves, IV catheters, and pacemakers.
The patient underwent an arthroplasty in his left knee 4 months ago and is now presenting with
septic arthritis, which makes a foreign body infection very likely. S. epidermidis is the most common
cause of prosthetic infections 3–12 months after surgery. If the patient's infection had developed
either < 3 months or > 12 months after surgery, Staphylococcus aureus would be the most likely
causal organism.

B - Escherichia coli

Explanation Why

Escherichia coli is a possible cause of septic arthritis in immunodeficient patients. However, this
patient has no history of immunodeficiency, making this answer unlikely.

C - Staphylococcus aureus

Explanation Why

Staphylococcus aureus is the most common cause of septic arthritis. However, patients with joint

www.eduwaves360.com | Telegram : @eduwaves360


infection due to S. aureus are usually acutely ill with high fever, chills, and onset of symptoms within
3 months after arthroplasty (when the infection is acquired during implantation) or after 12 months
from surgery (due to hematogenous infection). Given the fact that this patient presents 4 months after
surgery, an infection with a less virulent pathogen is more likely.

D - Pseudomonas aeruginosa

Explanation Why

Pseudomonas aeruginosa is a common cause of septic arthritis in immunodeficient patients. This


patient has no history of conditions that would increase the risk of P. aeruginosa infection (e.g.,
malignancy, HIV infection, diabetes mellitus), which makes this answer unlikely.

E - Group B Streptococcus

Explanation Why

Streptococcus agalactiae, a group B Streptococcus, does not commonly cause septic arthritis, making
this answer unlikely.

www.eduwaves360.com | Telegram : @eduwaves360


Question # 11

A 26-year-old man comes to the physician because of a 1-week history of left-sided chest pain. The pain
is worse when he takes deep breaths. Over the past 6 weeks, he had been training daily for an upcoming
hockey tournament. He does not smoke cigarettes or drink alcohol but has used cocaine once. His
temperature is 37.1°C (98.7°F), pulse is 75/min, and blood pressure is 128/85 mm Hg. Physical
examination shows tenderness to palpation of the left chest. An x-ray of the chest is shown. Which of
the following is the most appropriate initial pharmacotherapy?

Answer Image

A Nitroglycerin

B Alprazolam

C Alteplase

www.eduwaves360.com | Telegram : @eduwaves360


Answer Image

D Heparin

E Naproxen

www.eduwaves360.com | Telegram : @eduwaves360


Hint

This young man is presenting with a pleuritic and reproducible chest pain. His pretest probability for
acute coronary syndrome is quite low. Given his vigorous physical activity, physical exam findings, and
normal chest x-ray, a musculoskeletal cause of chest pain is most likely.

www.eduwaves360.com | Telegram : @eduwaves360


Correct Answer

A - Nitroglycerin

Explanation Why

Nitroglycerin is used in the treatment of angina pectoris. Unlike in this patient, anginal chest pain is a
poorly localized, worsens with exercise, and diminishes with rest. Localized tenderness is not
typically seen in this condition. Angina is most commonly seen in individuals with coronary heart
disease (CAD). This patient's young age and lack of risk factors for CAD (e.g., smoking, HTN, DM),
make angina an unlikely etiology of his pain.

B - Alprazolam

Explanation Why

Alprazolam is a benzodiazepine that can be used in the management of cocaine-induced chest pain.
Although this patient admits to having tried cocaine, cocaine-induced chest pain typically has an
acute onset with a duration of < 1 week. It is caused by potentiation of sympathetic response and
increased release of catecholamines, resulting in increased myocardial oxygen demand with
decreased myocardial perfusion. It typically does not cause localized tenderness with palpation, as
seen in this patient. An alternative etiology is the more likely cause of this patient's chest pain.

C - Alteplase

Explanation Why

Alteplase is a thrombolytic agent used in the management of highly unstable patients with pulmonary
embolism and in those with acute ischemic stroke. This patient's presentation is inconsistent with
stroke. Although pulmonary embolism may cause sharp, pleuritic chest pain, this patient's young age,
duration of pain, reproducibility on palpation, and lack of risk factors for VTE (e.g., immobilization,
malignancy, smoking), make PE an unlikely etiology of his pain. His chest x-ray lacks radiographic
signs of PE (e.g., Hampton's hump or Westermark sign). Furthermore, even in the setting of acute
PE, this patient would not be a candidate for thrombolytic therapy, given his stable vital signs.

www.eduwaves360.com | Telegram : @eduwaves360


D - Heparin

Explanation Why

Heparin is an anticoagulant used in the management of acute myocardial infarction (MI) and
pulmonary embolism (PE). Although pulmonary embolism may cause sharp, pleuritic chest pain, this
patient's young age, normal vital signs, duration of pain, reproducibility on palpation, and lack of risk
factors for VTE (e.g., immobilization, malignancy, smoking), make PE an unlikely etiology of his
pain. His chest x-ray also lacks radiographic signs of PE (e.g., Hampton's hump or Westermark sign).
The nature and reproducibility of this patient's chest pain, and lack of risk factors for CAD, also
make acute MI an unlikely etiology of his symptoms.

E - Naproxen

Explanation Why

The patient's symptoms are suggestive of costochondritis, a musculoskeletal etiology of chest pain
that responds well to NSAIDs. Alternative treatment modalities include hot/cold compresses and
limiting painful activities. This condition is a common cause of chest pain in young patients with a
history of vigorous physical exercise, as seen in this case.

www.eduwaves360.com | Telegram : @eduwaves360


Question # 12

An investigator is studying mechanisms of oxygen storage in skeletal muscle. Biopsy specimens of the
soleus muscle are obtained from resting marathon runners and sprinters. Compared to muscle tissue
obtained from the sprinters, muscle tissue from the marathon runners is more likely to show an increased
concentration of a protein that corresponds to which of the following oxygen dissociation curves?

Answer Image

A A

B B

C C

www.eduwaves360.com | Telegram : @eduwaves360


Answer Image

D D

E E

F F

www.eduwaves360.com | Telegram : @eduwaves360


Hint

Endurance training (e.g., marathon running) increases the proportion of slow twitch fibers in muscle
tissue (type I muscle fibers).

www.eduwaves360.com | Telegram : @eduwaves360


Correct Answer

A-A

Explanation Why

Figure A shows an oxygen dissociation curve for hemoglobin that has been shifted to the left.
Factors that shift the O2 Hb dissociation curve to the left include increased serum pH, decreased
temperature, decreased serum pCO2, and a decreased serum concentration of 2,3-BPG. However,
hemoglobin is the major oxygen transport protein in the body and is found in erythrocytes, not
myocytes. Myocytes contain a different protein that is primarily used to store, not transport, oxygen.

B-B

Explanation Why

Figure B shows a normal (i.e., unshifted) oxygen dissociation curve for hemoglobin. Hemoglobin is
the major oxygen transport protein in the body and is found in erythrocytes, not myocytes. Myocytes
contain a different protein that is primarily used to store, not transport, oxygen.

C-C

Explanation Why

Figure C shows an oxygen dissociation curve for hemoglobin that has been shifted to the right.
Factors that shift the oxygen dissociation curve for hemoglobin to the right include decreased serum
pH, increased temperature, increased serum pCO2, and an increased serum concentration of
2,3-BPG. However, hemoglobin is the major oxygen transport protein in the body and is found in
erythrocytes, not myocytes. Myocytes contain a different protein that is primarily used to store, not
transport, oxygen.

www.eduwaves360.com | Telegram : @eduwaves360


D-D

Image

Explanation Why

Figure D shows an oxygen dissociation curve for myoglobin, the predominant oxygen storage
protein found in muscle tissue. Myoglobin contains a single heme moiety, and its oxygen
dissociation curve does not exhibit the positive cooperativity seen in hemoglobin. The shape of the
oxygen dissociation curve for myoglobin is therefore hyperbolic (indicating a very high affinity for
O2), unlike the sigmoidal shape of hemoglobin's oxygen dissociation curve (figures A–C).
Myoglobin and mitochondria are found in high concentrations in slow twitch muscle fibers (type I
muscle), which become more prevalent in muscle tissue in response to endurance training (e.g.,
marathon running) in order to provide oxygen for sustained oxidative phosphorylation. In contrast,
sprinters have a higher proportion of fast twitch muscle fibers (type II muscle), which have lower
concentrations of myoglobin and mitochondria and are better suited to performing the anaerobic
glycolysis required for the rapid generation of energy.

www.eduwaves360.com | Telegram : @eduwaves360


E-E

Explanation Why

Figure E shows a hyperbolic oxygen dissociation curve that begins at a PaO2 of approximately 28
mm Hg. This curve does not correspond to any human oxygen-binding protein.

F-F

Explanation Why

Figure F shows a hyperbolic oxygen dissociation curve that begins at a PaO2 of approximately 40
mm Hg. This curve does not correspond to any human oxygen-binding protein.

www.eduwaves360.com | Telegram : @eduwaves360


Question # 13

A 7-year-old boy is brought to the physician for evaluation of developmental delay and intellectual
disability. He has been admitted to the hospital twice in the past 6 months because of a cerebral venous
thrombosis and a pulmonary embolism, respectively. He is at 10th percentile for weight and
95th percentile for height. Physical examination shows bilateral downward and inward subluxation of
the lenses. He has a high-arched palate and kyphosis. Laboratory studies show increased serum
concentration of 5-methyltetrahydrofolate. Which of the following additional findings is most likely in
this patient's serum?

Answer Image

A Decreased cysteine concentration

Increased S-adenosylhomocysteine
B
concentration

C Decreased methionine concentration

D Increased propionyl-CoA concentration

E Decreased cystathionine concentration

www.eduwaves360.com | Telegram : @eduwaves360


Hint

This patient presents with intellectual disability, a history of thromboembolic events, Marfanoid habitus
(tall stature, high-arched palate), and lens subluxation, all of which are concerning for the diagnosis of
homocystinuria. Homocystinuria can be caused by a deficiency in one of two different enzymes. This
patient's elevated serum 5-methyltetrahydrofolate confirms one of these two deficiencies.

www.eduwaves360.com | Telegram : @eduwaves360


Correct Answer

A - Decreased cysteine concentration

Explanation Why

Decreased cysteine concentrations may be seen in patients with homocystinuria secondary to a


deficiency to cystathionine synthase. Although patients with cystathionine synthase deficiency would
present similarly to this patient, his homocystinuria is likely instead secondary to methionine
synthase deficiency. This is indicated by his elevated serum levels of 5-methyltetrahydrofolate.
Cysteine levels would be normal in patients with methionine synthase deficiency.

B - Increased S-adenosylhomocysteine concentration

Explanation Why

An increased s-adenosylhomocysteine concentration would not be expected in patients with


homocysteinuria secondary to a defect in methionine synthase. S-adenosylhomocysteine is a
downstream metabolite in this metabolic pathway, meaning a decreased (not increased) serum
concentration would be expected in this case.

www.eduwaves360.com | Telegram : @eduwaves360


C - Decreased methionine concentration

Image

Explanation Why

Decreased methionine concentration is characteristic of homocystinuria secondary to a deficiency in


methionine synthase. Note that homocystinuria can also result from cystathionine synthase
deficiency, however, this would cause elevated rather than decreased serum levels of methionine.
Homocystinuria secondary to methionine synthase deficiency also results in elevated serum
5-methyltetrahydrofolate, as seen in this patient. Alternatively, this is not true in patients with
cystathionine synthase deficiency.

D - Increased propionyl-CoA concentration

Explanation Why

Increased propionyl-CoA concentration would not be expected in homocystinuria secondary to a

www.eduwaves360.com | Telegram : @eduwaves360


defect in methionine synthase, as seen in this patient's case. Propionyl-CoA is a downstream
metabolite in this metabolic pathway, meaning a decreased (not increased) serum concentration
would be expected in this case.

E - Decreased cystathionine concentration

Explanation Why

Decreased cystathionine concentrations may be seen in patients with homocystinuria secondary to a


deficiency to cystathionine synthase. Although patients with cystathionine synthase deficiency would
present similarly to this patient, his homocystinuria is likely instead secondary to methionine
synthase deficiency. This is indicated by his elevated serum levels of 5-methyltetrahydrofolate.
Cystathionine levels would be normal in patients with methionine synthase deficiency.

www.eduwaves360.com | Telegram : @eduwaves360


Question # 14

A 58-year-old woman comes to the physician for evaluation of worsening fatigue for 1 week. She also
has a 1-year history of hand pain and stiffness. Four months ago, she started a new medication for these
symptoms. Medications used prior to that included ibuprofen, prednisone, and hydroxychloroquine.
Examination shows a subcutaneous nodule on her left elbow and old joint destruction with Boutonniere
deformity. Her hemoglobin concentration is 10.1 g/dL, leukocyte count is 3400/mm3, and platelet count
is 101,000/mm3. Methylmalonic acid levels are normal. Which of the following could have prevented
this patient's laboratory abnormalities?

Answer Image

A Vitamin B6

B Vitamin B12

C Amifostine

D 2-Mercaptoethanesulfonate

E Leucovorin

www.eduwaves360.com | Telegram : @eduwaves360


Hint

This patient presents with typical clinical findings of rheumatoid arthritis (pain and stiffness of the
hands, subcutaneous nodules, joint destruction with boutonniere deformity). Her fatigue and signs of
myelosuppression in laboratory studies (leukocytopenia, thrombocytopenia, anemia) indicate that she
was started on treatment with methotrexate (MTX).

www.eduwaves360.com | Telegram : @eduwaves360


Correct Answer

A - Vitamin B6

Explanation Why

Vitamin B6 should be added to treatment regimens that include isoniazid (INH), a drug most
commonly used to treat tuberculosis. While symptomatic Vitamin B6 deficiency is uncommon in the
general population, it is commonly seen in association with INH, as INH competes with pyridoxine
in neurotransmitter synthesis. In advanced cases, patients with chronic pyridoxine deficiency may
present with sensory peripheral neuropathy, stomatitis, glossitis, and convulsions, none of which are
present in this patient. This patient also lacks features of tuberculosis, such as weight loss, night
sweats, and productive cough, to indicate INH treatment.

B - Vitamin B12

Explanation Why

Vitamin B12 should be given to individuals with vitamin B12 deficiency. This patient has some
features of vitamin B12 deficiency, including anemia, thrombocytopenia, and leukocytopenia.
However, she lacks other typical findings such as glossitis and neurological deficits (peripheral
neuropathy, loss of vibratory sensation). Methylmalonic acid levels are also elevated in patients with
vitamin B12 deficiency. Moreover, she does not have a history of malabsorption or malnutrition (e.g.,
chronic alcoholism) to support a cause of vitamin B12 deficiency.

C - Amifostine

Explanation Why

Amifostine acts as a free-radical scavenger. It should be added to treatment regimens that include
platinum-based agents (e.g., cisplatin) to prevent nephrotoxic side effects. This patient, however,
does not present with signs of impaired kidney function (e.g., oliguria). Moreover, platinum-based
agents are not used in the treatment of rheumatoid arthritis, but rather to treat certain malignancies

www.eduwaves360.com | Telegram : @eduwaves360


(e.g., lung cancer, ovarian cancer, and urothelial carcinoma).

D - 2-Mercaptoethanesulfonate

Explanation Why

2-Mercaptoethanesulfonate (mesna) deactivates acrolein and increases the urinary excretion of


cysteine. It should be added to treatment regimens involving cyclophosphamide to prevent
hemorrhagic cystitis. Some cases of rheumatoid arthritis can be treated with cyclophosphamide.
However, this patient does not have typical findings of hemorrhagic cystitis such as hematuria and
symptoms of bladder irritation (e.g., increased urinary frequency and urgency).

E - Leucovorin

Explanation But

Other side effects of MTX include mucositis, hair loss, susceptibility to infection, hepatotoxicity,
nephrotoxicity, macrocytic anemia, pulmonary toxicity, and pulmonary fibrosis.

Explanation Why

Typical side effects of methotrexate (MTX) include myelosuppression, causing (macrocytic) anemia,
leukocytopenia, and thrombocytopenia, as well as constitutional symptoms (e.g., fatigue, fever),
which are seen in this patient. These side effects can be reduced by administering salvage therapy in
the form of folic acid or folinic acid (leucovorin). Leucovorin should be administered as prophylactic
therapy within 24–48 hours of starting high-dose treatment with MTX.

www.eduwaves360.com | Telegram : @eduwaves360


Question # 15

A 65-year-old woman comes to the physician because of a 4-month history of generalized bone pain.
She has a 25-year history of Crohn disease with multiple hospitalizations for acute exacerbations,
including a bowel obstruction last year. Current medications include mesalamine and bisoprolol. She is
165 cm (5 ft 6 in) tall and weighs 53 kg (117 lb); her BMI is 19.5 kg/m2. Examination shows a soft
abdomen and pale conjunctivae. Neurological examination shows decreased vibratory sensation over her
lower extremities. Muscle strength is 4/5 in the distal groups and 3/5 in proximal groups of the lower
limbs. The pelvic bones are tender to palpation. An x-ray of the pelvis shows thin cortices and multiple
radiolucent bands that are perpendicular to the cortex and surrounded by a thin sclerotic margin. Which
of the following laboratory findings are most likely present in this patient?

Calcium Phosphate Alkaline phosphatase Parathyroid hormone

A Normal normal normal normal

B Normal normal ↑ normal

C ↓ ↓ ↑ ↑

D ↑ ↓ ↑ ↑

E ↓ ↑ ↑ ↑

F ↑ ↑ ↑ ↓

G ↓ ↑ normal ↓

Answer Image

A A

www.eduwaves360.com | Telegram : @eduwaves360


Answer Image

B B

C C

D D

E E

F F

G G

www.eduwaves360.com | Telegram : @eduwaves360


Hint

This patient's history of Crohn disease combined with her physical exam findings of pale conjunctiva
and decreased vibratory sensation suggests malabsorption as the underlying cause for her bone pain.
Bone pain combined with thin cortices and Looser zones (transverse radiolucent bands) on x-ray
indicates osteomalacia.

www.eduwaves360.com | Telegram : @eduwaves360


Correct Answer

A-A

Explanation Why

Normal calcium, phosphorus, PTH, and ALP levels in an elderly female patient with bone pain are
characteristic of osteoporosis. Osteoporosis would result in cortical thinning and increased
radiolucency due to decreased bone density. However, bone sclerosis would not be expected, and the
bone would be diffusely radiolucent, in contrast to the osteolytic lesions seen in this patient, which
have a distinctive transverse band-like appearance on x-ray. Moreover, osteoporosis would not
explain the myopathic weakness seen here.

B-B

Image

www.eduwaves360.com | Telegram : @eduwaves360


Explanation Why

High serum ALP and normal calcium, phosphorus, and PTH levels in an elderly patient with bone
pain are characteristic of Paget disease of the bone (PDB). However, while PDB lesions can have a
mixed lytic (radiolucent) and sclerotic appearance, they typically appear as lytic lesions within a
background of diffuse sclerosis. Furthermore, a thin cortex would not be seen in PDB.

C-C

Explanation But

Chronic inflammation of the ileum in patients with Crohn disease can cause impaired reabsorption of
bile acids, resulting in fat malabsorption; unabsorbed fats can trap fat-soluble vitamins (e.g., vitamin
D) in the intestinal lumen, causing a deficiency.

Explanation Why

Severe vitamin D deficiency can lead to hypocalcemia and hypophosphatemia by decreasing the
amount of calcium and phosphate absorption in the intestine. As long as calcium levels are low,
increased PTH secretion occurs in an attempt to maintain normal serum calcium levels (secondary
hyperparathyroidism). PTH releases calcium stored in bone into serum by increasing bone turnover,
which the elevated ALP levels (a marker of bone turnover) indicate here. Increased PTH levels also
worsen hypophosphatemia by decreasing renal phosphate reabsorption.

D-D

Explanation Why

Decreased phosphate and increased calcium, ALP, and PTH levels are characteristic of primary
hyperparathyroidism, which typically occurs in elderly women and causes bone pain. A pelvic x-ray
in a patient with primary hyperparathyroidism can show cortical thinning as well as focal osteolytic
lesions. However, the osteolytic lesions in patients with primary hyperparathyroidism are usually
round/polygonal circumscribed lesions rather than the distinctive transverse band-like lesions seen
here.

www.eduwaves360.com | Telegram : @eduwaves360


E-E

Explanation Why

Bone pain with decreased calcium and increased phosphate, ALP, and PTH levels can be seen in
patients with secondary hyperparathyroidism due to ESRD (renal osteodystrophy), as well as in
patients with pseudohypoparathyroidism (PHP). Also, patients with PHP or renal osteodystrophy can
have muscle weakness (due to myopathy) and radiological features of osteomalacia such as Looser
zones, as seen here. However, renal osteodystrophy would not occur until the very late stage of CKD
(i.e., when renal replacement therapy is required). PHP, on the other hand, would typically manifest
during childhood with features of Albright hereditary osteodystrophy (e.g., short stature, round face,
obesity, short fourth metacarpals, soft tissue calcifications, dental hypoplasia).

F-F

Explanation Why

Osteolytic metastases, which appear as radiolucent lesions, can result in increased calcium,
phosphorus, and ALP levels, together with decreased PTH levels. Bone metastases should be
considered in any elderly patient with bone pain even if the primary tumor is not clinically evident.
Hypercalcemia could also explain muscle weakness in this patient. However, the transverse band-like
appearance of the radiolucent lesions in this woman suggests a different diagnosis, as osteolytic
metastases usually appear as ill-defined, round lesions on x-ray.

G-G

Explanation But

Rarely (< 1% of cases), primary hypoparathyroidism occurs as a result of autoimmune disease or


infiltrative diseases of the parathyroid glands (e.g., sarcoidosis, hemochromatosis, Wilson disease,
metastases).

www.eduwaves360.com | Telegram : @eduwaves360


Explanation Why

Decreased calcium and PTH levels with normal serum ALP levels and increased phosphate are
consistent with primary hypoparathyroidism, which can result in decreased muscle strength.
However, features of bone disease are usually absent in patients with primary hypoparathyroidism.
Moreover, in adults, primary hypoparathyroidism is typically caused by iatrogenic damage to the
parathyroid glands during neck surgery (e.g., thyroidectomy) or neck irradiation.

www.eduwaves360.com | Telegram : @eduwaves360


Question # 16

A 38-year-old man comes to the physician because of a 6-month history of chest discomfort and
progressive dyspnea. He cannot do daily chores without feeling out of breath. He has a history of an
X-linked recessive disorder that causes progressive proximal muscle weakness and gait abnormalities.
Physical examination shows a waddling gait and weak patellar reflexes. Cardiovascular examination
shows a holosystolic murmur, displaced point of maximal impulse, and bilateral pitting edema of the
ankle. Laboratory studies show elevated levels of brain natriuretic peptide. Which of the following is the
most likely underlying cause of this patient's muscle weakness?

Answer Image

Impaired connection of cytoskeletal


A actin filaments to membrane-bound
dystroglycan

Defective lysine-hydroxylysine
B
crosslinking of tropocollagen

Reduced nicotinic acetylcholine


C
receptor density on sarcolemma

Interruption of microtubule
D depolymerization through
stabilization of GDP-tubulin

www.eduwaves360.com | Telegram : @eduwaves360


Answer Image

Cell–mediated cytotoxicity against


E skeletal muscle antigens in the
endomysium

Increased number of CTG repeats in


F
the DMPK gene

www.eduwaves360.com | Telegram : @eduwaves360


Hint

Signs of global heart failure caused by dilated cardiomyopathy include progressive dyspnea, a
holosystolic murmur, displaced point of maximal impulse, pitting edema, and elevated levels of brain
natriuretic peptide (BNP). In combination with a history of genetic disease involving muscle weakness,
these signs are consistent with Becker muscular dystrophy (BMD).

www.eduwaves360.com | Telegram : @eduwaves360


Correct Answer

A-

Impaired connection of cytoskeletal actin filaments to membrane-bound


dystroglycan

Explanation Why

The mutated dystrophin gene that causes BMD is the underlying cause of the impaired connection of
the cytoskeletal actin filaments to membrane-bound dystroglycan. Dystrophin normally provides
muscle fibers with strength and stiffness; its dysfunction accounts for the pelvic girdle muscle
weakness and ensuing gait abnormalities that characterize this genetic disease. Cardiac involvement
can also occur in BMD, although this typically manifests with features of heart failure following the
onset of gait abnormalities.

B - Defective lysine-hydroxylysine crosslinking of tropocollagen

Explanation Why

In Ehlers-Danlos syndrome (EDS), defective lysine-hydroxylysine crosslinking of tropocollagen


leads to unstable collagen fibrils. The condition usually causes hyperextensible skin and joint
hypermobility. Cardiac manifestations are also common, especially mitral valve prolapse (MVP) or
dilation of the aortic root with possible subsequent aortic regurgitation and, ultimately, heart failure.
Unlike this patient's findings, a midsystolic click with a late systolic murmur would be expected for
MVP, while a decrescendo diastolic murmur would be expected for aortic regurgitation.
Furthermore, progressive proximal muscle weakness and gait abnormalities, as seen in this patient,
are not typical features of EDS.

www.eduwaves360.com | Telegram : @eduwaves360


C - Reduced nicotinic acetylcholine receptor density on sarcolemma

Image

Explanation Why

Reduced nicotinic acetylcholine receptor density on the sarcolemma occurs in myasthenia gravis. In
this autoimmune disease, these receptors are also blocked by autoantibodies. Myasthenia gravis
classically manifests with ptosis and diplopia, which worsen with prolonged muscle use and improve
with the administration of edrophonium. However, general proximal muscle weakness and heart
failure are not common features of myasthenia gravis. In addition, myasthenia gravis is not inherited
in an X-linked fashion.

www.eduwaves360.com | Telegram : @eduwaves360


D-

Interruption of microtubule depolymerization through stabilization of GDP-


tubulin

Explanation Why

Interruption of microtubule depolymerization through the stabilization of GDP-tubulin is the


mechanism of action of a group of chemotherapeutic agents called taxanes. Taxanes, e.g., paclitaxel,
have significant side effects, including myelosuppression and peripheral neuropathy. However,
progressive muscle weakness and heart failure would not be expected. Furthermore, because this
patient has no history of cancer, there is no indication for treatment with a taxane.

E-

Cell–mediated cytotoxicity against skeletal muscle antigens in the endomysium

Explanation Why

Cell-mediated cytotoxicity against skeletal muscle antigens in the endomysium causes chronic
inflammation of skeletal muscle, as seen in inflammatory myopathies such as polymyositis. Patients
with polymyositis classically present with proximal muscle weakness (i.e., of the shoulder and pelvic
girdle muscles). Advanced polymyositis may manifest with dysphagia and aspiration caused by
oropharyngeal muscle involvement. However, cardiac muscle involvement and global heart failure,
as seen in this patient, are not associated with polymyositis. Furthermore, these conditions are not
inherited in an X-linked fashion.

F - Increased number of CTG repeats in the DMPK gene

Explanation Why

CTG repeats in the region of the DMPK gene cause a deficiency of myotonin-protein kinase in
skeletal and heart muscle tissue. CTG repeats in the DMPK gene are the underlying pathology of
myotonic dystrophy type 1 (MDT1), which manifests with myotonia, muscular atrophy, balding,
cataracts, and arrhythmias. However, this patient has none of these cardiovascular findings.

www.eduwaves360.com | Telegram : @eduwaves360


Moreover, MDT1 is an autosomal dominant condition.

www.eduwaves360.com | Telegram : @eduwaves360


Question # 17

A 41-year-old woman comes to the doctor because of gradually progressive weakness in her right hand
over the past few weeks. She goes to the gym to lift weights 5 times a week. With the dorsum of the
right hand on a flat surface, the patient is unable to move her thumb to touch a pen held 2 cm above the
interphalangeal joint of the thumb. An MRI of the right arm shows compression of a nerve that passes
through the pronator teres muscle. Based on the examination findings, loss of innervation of which of
the following muscles is most likely in this patient?

Answer Image

A Adductor pollicis

B Flexor pollicis longus

C First dorsal interosseus

D Abductor pollicis brevis

E Extensor pollicis brevis

F Abductor pollicis longus

www.eduwaves360.com | Telegram : @eduwaves360


Hint

This patient presents with an inability to abduct the thumb (pen test). This finding indicates weakness of
a muscle that is innervated by the median nerve, which runs through the two heads of the pronator teres
muscle.

www.eduwaves360.com | Telegram : @eduwaves360


Correct Answer

A - Adductor pollicis

Explanation Why

The adductor pollicis is responsible for thumb adduction at the first carpometacarpal joint and helps
oppose the thumb. This muscle is innervated by the ulnar nerve, which does not pass through the
pronator teres muscle. As the adductor pollicis helps to oppose and adduct the thumb, damage to the
ulnar nerve would lead to weakness of these functions, which would be tested by Froment sign. This
patient presents with weakness of thumb abduction, which is inconsistent with damage to the
adductor pollicis.

B - Flexor pollicis longus

Explanation Why

The flexor pollicis longus is responsible for flexion of the metacarpophalangeal and interphalangeal
joints. This muscle is innervated by the anterior interosseus nerve, which is a branch of the median
nerve that passes through the pronator teres muscle. Damage to the median nerve would lead to
weakness in flexing the interphalangeal joint of the thumb against resistance. This patient presents
with weakness in thumb abduction, which is inconsistent with damage to the flexor pollicis longus.

C - First dorsal interosseus

Explanation Why

The first dorsal interosseus muscle is responsible for abduction of the index finger and facilitates
adduction of the thumb. Like all other dorsal interosseus muscles, this muscle is innervated by the
ulnar nerve, which runs between the two heads of flexor carpi ulnaris, but does not pass through the
pronator teres muscle. Damage to the ulnar nerve would lead to weakness of abduction of the index,
middle, and ring fingers, which would be tested by adducting these fingers against resistance.

www.eduwaves360.com | Telegram : @eduwaves360


D - Abductor pollicis brevis

Explanation Why

This patient shows signs of weakness of the abductor pollicis brevis. The abductor pollicis brevis is a
muscle of the thenar eminence that arises from the flexor retinaculum and the scaphoid bone, before
inserting into the proximal phalanx and the capsule of the first metacarpophalangeal joint. This
muscle is responsible for thumb abduction, extension, and opposition. Damage to the median nerve
leads to a weakness of these functions, especially abduction, which can be assessed by the pen test.
Repetitive activities such as weight-lifting can cause median nerve injury by direct compression or
entrapment.

E - Extensor pollicis brevis

Explanation Why

The extensor pollicis brevis is supplied by the posterior interosseous nerve, a branch of the radial
nerve. This muscle extends the thumb at the metacarpophalangeal and interphalangeal joints.
Damage to the radial nerve would lead to weakness of this function, which would be tested by
extending the interphalangeal joint against resistance. This patient's weak thumb abduction, and
positive pen test, are inconsistent with weakness of the extensor pollicis brevis.

F - Abductor pollicis longus

Explanation Why

The abductor pollicis longus is responsible for thumb abduction at the first metacarpophalangeal
joint and the carpometacarpal joint. The abductor pollicis longus also helps with extension and
rotation of the thumb. This muscle is innervated by the posterior interosseous nerve, which is a
branch of the radial nerve. The posterior interosseous nerve descends along the interosseous
membrane, anterior to the extensor pollicis longus muscle, but it does not pass through the pronator
teres muscle. Furthermore, damage to the posterior interosseous nerve would not lead to an inability
to abduct the thumb in the pen test.

www.eduwaves360.com | Telegram : @eduwaves360


www.eduwaves360.com | Telegram : @eduwaves360
Question # 18

A 21-year-old woman is brought to the emergency department following a motor vehicle collision. She
has significant pain and weakness in her right arm and hand. Physical examination shows multiple
ecchymoses and tenderness in the right upper extremity. She is able to make a fist, but there is marked
decrease in grip strength. An x-ray of the right upper extremity shows a midshaft humerus fracture.
Which of the following structures is most likely injured?

Answer Image

A Median nerve

B Radial nerve

C Radial artery

D Brachial artery

www.eduwaves360.com | Telegram : @eduwaves360


Answer Image

E Ulnar nerve

F Musculocutaneous nerve

www.eduwaves360.com | Telegram : @eduwaves360


Hint

This patient most likely cannot extend her hand at the wrist joint.

www.eduwaves360.com | Telegram : @eduwaves360


Correct Answer

A - Median nerve

Explanation Why

A median nerve palsy would result in decreased grip strength, as seen here, but the patient would
also be unable to make a fist because the muscles that control thumb opposition (thenar muscles) and
flexion of the index and middle fingers (flexor digitorum superficialis, lateral belly of the flexor
digitorum profundus) would also be paralyzed. In addition, median nerve palsy is typically
associated with supracondylar fractures rather than a mid-shaft fracture of the humerus.

B - Radial nerve

Image

www.eduwaves360.com | Telegram : @eduwaves360


Explanation Why

Radial nerve injury is the most common complication associated with humerus mid-shaft fractures
due to the radial nerve's proximity to the humerus shaft. It is typically associated with wrist drop,
decreased finger extensor strength, and loss of sensation over the dorsal hand and forearm. Grip
strength can also be decreased through impaired wrist extension, as seen in this patient, but the
ability to make a fist is not affected.

C - Radial artery

Image

Explanation Why

The radial artery is unlikely to be injured by a mid-shaft fracture of the humerus because it originates
below the cubital fossa as a terminal branch of the brachial artery.

www.eduwaves360.com | Telegram : @eduwaves360


D - Brachial artery

Explanation Why

Brachial artery injury is a possible complication of humerus fractures, but is more likely to be injured
in a supracondylar fracture (brachial artery runs along the anteromedial aspect of the humerus) than a
mid-shaft fracture as seen in this patient. Acute limb ischemia due to brachial artery injury can cause
distal weakness (e.g., decreased grip strength). However, paralysis due to acute limb ischemia is
usually associated with paresthesias, which are not reported by this patient. A different structure is
more likely to be injured in this patient.

E - Ulnar nerve

Explanation Why

An ulnar nerve palsy would result in decreased grip strength, as seen here, but the patient would also
be unable to make a fist because the muscles that control flexion of the little and ring fingers (medial
belly of the flexor digitorum profundus) would also be paralyzed. In addition, ulnar nerve palsy is
typically associated with fractures of the medial epicondyle of the humerus rather than a mid-shaft
fracture of the humerus.

F - Musculocutaneous nerve

Explanation Why

Isolated musculocutaneous nerve injury is rare and typically occurs as a result of trauma from chest
compression or shoulder dislocation. Although such injuries can occur in motor vehicle accidents,
they would manifest with impaired elbow flexion, not with decreased grip strength.

www.eduwaves360.com | Telegram : @eduwaves360


Question # 19

A 45-year-old man comes to the physician because of mild left-sided abdominal pain when lifting heavy
objects that has developed over the past 6 months. He has had a weight loss of 41 kg (90 lb) in the past
year. Physical examination shows a soft bulge inferior and lateral to the navel on the left. A CT scan of
the abdomen of a patient with the same condition is shown. Which of the following muscles is located
directly medial to the pathology seen on the CT scan?

Answer Image

A Psoas major

B Internal oblique

C External oblique

www.eduwaves360.com | Telegram : @eduwaves360


Answer Image

D Rectus abdominis

E Transverse abdominal

www.eduwaves360.com | Telegram : @eduwaves360


Hint

The CT scan shows a left-sided Spigelian hernia. Rapid weight loss is a risk factor for this condition.

www.eduwaves360.com | Telegram : @eduwaves360


Correct Answer

A - Psoas major

Explanation Why

The psoas major is not a muscle of the anterior abdominal wall and would not be located medial to
this defect. The psoas major originates from the lumbar vertebra and pelvic brim and inserts into the
lesser trochanter of the femur.

B - Internal oblique

Explanation Why

The internal oblique aponeurosis is part of the linea semilunaris. This muscle originates laterally
from the iliac crest and lumbar fascia and continues medially to the linea semilunaris. In the case of a
Spigelian hernia, the hernia sac protrudes directly through the internal oblique aponeurosis, which
means the internal oblique muscle would be lateral to the hernia.

C - External oblique

Explanation Why

The external oblique muscle lies superficial and lateral to the defect on this patient's CT scan. The
medial aponeurosis of this muscle contributes to the most superficial part of the linea semilunaris and
helps to reinforce the anterior portion of the rectus sheath. The structure medial to the hernia is a
different muscle.

www.eduwaves360.com | Telegram : @eduwaves360


D - Rectus abdominis

Image

Explanation Why

The rectus abdominis muscle lies directly medial to the hernia sac on this CT scan. Spigelian hernias
occur through defects in the linea semilunaris of the anterior abdominal wall. The linea semilunaris is
a curved, vertical line at the lateral edge of each rectus abdominis muscle that is composed of the
aponeurotic fibers of the internal oblique muscle as they merge with the rectus sheath. Spigelian
hernias are therefore bounded medially by the rectus abdominis. These hernias are occasionally
palpable as a subcutaneous bulge. However, they are most commonly intraperitoneal and may not be
evident on examination. Because the fascial defects are typically small, there is a high risk of bowel
strangulation; surgical repair is therefore indicated.

www.eduwaves360.com | Telegram : @eduwaves360


E - Transverse abdominal

Explanation Why

The transverse abdominal muscle is the deepest of the anterior abdominal wall muscles. It
contributes fibers to the most posterior part of the linea semilunaris, which is deep to the internal
oblique. The defect seen in this patient would occur through the aponeurotic fibers of the transverse
abdominal muscles, which means that the transverse abdominal muscles would not be medial to the
defect.

www.eduwaves360.com | Telegram : @eduwaves360


Question # 20

A 35-year-old woman comes to the physician because of progressive left flank pain over the past
2 weeks. She has a history of type 1 diabetes mellitus. Her temperature is 38°C (100.4°F). There is
tenderness to percussion along the left flank. Passive extension of the left hip is painful. Her leukocyte
count is 16,000/mm3. An axial CT scan is shown. The underlying pathology is most likely located in
which of the following anatomical structures?

Answer Image

A Left ovary

Quadratus
B lumborum
muscle

www.eduwaves360.com | Telegram : @eduwaves360


Answer Image

Iliacus
C
muscle

Erector
D spinae
muscles

Psoas
E major
muscle

F Colon

Left
G
kidney

Abdominal
H
aorta

www.eduwaves360.com | Telegram : @eduwaves360


Hint

The CT scan shows a left-sided, circular, isodense structure with clear borders. In combination with
fever, flank pain, and leukocytosis, these findings are suggestive of abscess formation.

www.eduwaves360.com | Telegram : @eduwaves360


Correct Answer

A - Left ovary

Explanation Why

Ovarian abscesses can be caused by an ascending infection or occur as a complication of pelvic


inflammatory disease. A left tubo-ovarian abscess typically manifests with fever, lower abdominal
pain, vaginal discharge, and leukocytosis; it does not typically cause pain during hip flexion. In
addition, the CT scan shows a fluid collection that involves a structure along the posterior abdominal
wall, not the left ovary.

B - Quadratus lumborum muscle

Explanation Why

The quadratus lumborum muscle enables lateral flexion and extension of the vertebral column as
well as stabilization of the diaphragm during inspiration. An infection or abscess that involves this
muscle group would not cause pain during passive hip extension (psoas sign). Furthermore, this
muscle is not clearly visible on the CT scan; instead, a fluid collection can be seen in a different
structure.

C - Iliacus muscle

Explanation Why

The iliacus muscle lies on the iliac fossa, lateral to the psoas major muscle. With its insertion into the
lesser trochanter of the femur, the iliacus enables hip flexion. On this patient's CT scan, the iliacus is
the thin, flat structure posterior to the fluid collection but immediately anterior to the iliac bone,
which indicates that the fluid collection is located within a different structure.

www.eduwaves360.com | Telegram : @eduwaves360


D - Erector spinae muscles

Explanation Why

The erector spinae muscles are intrinsic back muscles that extend the trunk. An infection or abscess
that involves this muscle group would not cause pain during passive hip extension (psoas sign). In
addition, the CT scan here shows an abscess that involves a different muscle group.

E - Psoas major muscle

Image

Explanation But

This patient's underlying diabetes is a risk factor for the formation of primary abscesses.

www.eduwaves360.com | Telegram : @eduwaves360


Explanation Why

This patient's subacute flank pain, positive psoas sign, fever, and leukocytosis are suggestive of an
abscess that involves the hip flexors (e.g., psoas major, iliacus, or iliopsoas). The CT scan indicates
an abscess within the left psoas major muscle. This muscle originates from the vertebral bodies,
intervertebral discs, and transverse processes of T12–L5. A psoas abscess can be classified as either a
primary abscess due to hematogenous spread of infection (e.g., bacteremia, endocarditis), or a
secondary abscess caused by local spread of infection (e.g., osteomyelitis, pyelonephritis, trauma).
Primary abscesses are more common than secondary abscesses.

F - Colon

Explanation Why

An abscess of the colon can occur as a complication of inflammatory bowel disease or diverticulitis.
A colonic abscess can cause abdominal pain, fever, leukocytosis, and bloody diarrhea, but it does not
typically cause pain during hip flexion. In addition, the CT scan shows a fluid collection in a
structure of the posterior abdominal wall, not the bowel.

G - Left kidney

Explanation Why

An abscess that involves the left kidney can cause left-sided flank pain, leukocytosis, and fever.
However, they also typically result in dysuria, which this patient does not have. In addition, risk
factors for the development of renal abscesses include acute pyelonephritis, recent procedures that
involve the kidney, and nephrolithiasis, none of which are seen in this patient. Finally, CT scan of an
abscess that involves the left kidney would show renal parenchyma surrounding the lesion (in the
case of a renal abscess), or a fluid collection contained within the capsule of the kidney but outside
the parenchyma (in the case of a perinephric abscess).

www.eduwaves360.com | Telegram : @eduwaves360


H - Abdominal aorta

Explanation Why

Aortitis can be difficult to diagnose because it commonly manifests in a nonspecific manner, with a
spectrum of symptoms that includes abdominal pain, fever, and weight loss. Infection or
inflammation of the abdominal aorta does not typically manifest with a psoas sign, flank pain, or
leukocytosis. CT scan of aortitis usually shows aortic dilation or other anomalies in the vessel.
However, this CT scan is taken from a slice distal to the branching of the aorta into the iliac arteries,
and it shows a fluid collection in a different structure.

www.eduwaves360.com | Telegram : @eduwaves360


Question # 21

A 36-year-old woman comes to the physician because of a 2-week history of progressively worsening
pain on the outer side of her left elbow. She does not recall any trauma to the area. The patient plays
badminton recreationally. Examination shows tenderness over the lateral surface of the left distal
humerus. The pain is reproduced by supinating the forearm against resistance. Which of the following is
the most likely underlying cause of this patient's pain?

Answer Image

A Excessive stress to bone

B Nerve compression at the elbow

C Bursal inflammation

www.eduwaves360.com | Telegram : @eduwaves360


Answer Image

D Repeated wrist extension

E Repeated wrist flexion

www.eduwaves360.com | Telegram : @eduwaves360


Hint

Progressively worsening pain near the lateral epicondyle in a racket-sports enthusiast suggests lateral
epicondylitis (tennis elbow).

www.eduwaves360.com | Telegram : @eduwaves360


Correct Answer

A - Excessive stress to bone

Image

Explanation Why

Excessive bone stress from submaximal loading is responsible for stress fractures. The most common
site for stress fractures near the elbow is the olecranon process, and individuals involved in racket
sports are at increased risk of developing this overuse injury. However, patients with olecranon stress
fractures typically present with pain over the posterior surface of the elbow and exacerbation of pain
with extension of the forearm against resistance. Given the pain on the lateral surface of the forearm
and exacerbation of pain with wrist extension against resistance seen here, a different diagnosis is
likely.

www.eduwaves360.com | Telegram : @eduwaves360


B - Nerve compression at the elbow

Explanation But

Compression of the posterior interosseous nerve within the radial tunnel occasionally results in pain
on the lateral surface of the elbow without motor weakness, sensory deficits, or paresthesias (radial
tunnel syndrome). In approximately 5% of patients, radial tunnel syndrome occurs in conjunction
with lateral epicondylitis. However, in radial tunnel syndrome, pain typically affects the dorsal aspect
of the proximal forearm rather than the distal humerus.

Explanation Why

Compression neuropathies that can occur at the elbow include cubital tunnel syndrome, pronator
teres syndrome, and posterior interosseous nerve syndrome. Although individuals who practice
racket sports are at increased risk of developing these conditions, a compression neuropathy would
typically result in distal weakness in the affected arm. Cubital tunnel syndrome and pronator teres
syndrome would also result in sensory loss and/or paresthesias.

www.eduwaves360.com | Telegram : @eduwaves360


C - Bursal inflammation

Image

Explanation Why

Repetitive elbow flexion and extension, which occurs while playing racket sports, can cause elbow
pain due to olecranon bursitis. However, the pain associated with olecranon bursitis would be
localized to the posterior surface of the elbow, and swelling would usually occur at the tip of the
elbow. The more likely cause of this patient's symptoms is lateral epicondylitis, which is not the
result of bursitis.

www.eduwaves360.com | Telegram : @eduwaves360


D - Repeated wrist extension

Image

Explanation Why

The lateral epicondyle is the common origin of the wrist extensors. Lateral epicondylitis is caused by
activities, e.g., racket sports such as tennis and badminton, that involve repetitive wrist extension
and/or repetitive forearm supination/pronation with the elbow nearly fully extended. In patients with
lateral epicondylitis, the pain can be exacerbated by resisted supination, resisted wrist extension, and
tight grasping of objects. The most sensitive imaging finding is thickening and edema of the
tendinous origin of the extensor carpi radialis brevis.

E - Repeated wrist flexion

Explanation Why

Repeated wrist flexion can cause medial epicondylitis, which can affect individuals involved in

www.eduwaves360.com | Telegram : @eduwaves360


racket sports and manifests with elbow pain. However, the pain would be present on the medial
surface of the elbow and it would be exacerbated by forearm pronation and/or wrist flexion.
Moreover, medial epicondylitis is 5–10 times less common than lateral epicondylitis.

www.eduwaves360.com | Telegram : @eduwaves360


Question # 22

A 56-year-old man is brought to the emergency department 30 minutes after falling from a height of
3 feet onto a sharp metal fence pole. He is unconscious. Physical examination shows a wound on the
upper margin of the right clavicle in the parasternal line that is 3-cm-deep. Which of the following is the
most likely result of this patient's injury?

Answer Image

A Rotator cuff tear due to supraspinatus muscle injury

B Pneumothorax due to pleural injury

Trapezius muscle paresis due to spinal accessory nerve


C
injury

D Hemathorax due to azygos vein injury

E Chylothorax due to thoracic duct injury

F Traumatic aneurysm due to internal carotid artery injury

www.eduwaves360.com | Telegram : @eduwaves360


Hint

This condition can also be caused by improper placement of a central venous catheter.

www.eduwaves360.com | Telegram : @eduwaves360


Correct Answer

A - Rotator cuff tear due to supraspinatus muscle injury

Explanation Why

The supraspinatus muscle is part of the rotator cuff, which stabilizes the shoulder joint. It runs from
the superior portion of the scapula to the greater tubercle of the humerus. Because this patient's
wound is on the upper margin of the right clavicle, injury of the supraspinatus muscle is unlikely.

B - Pneumothorax due to pleural injury

Image

Explanation Why

The pleural membrane covering the apex of the superior lobe of the lung extends approx. 2.5 cm
above the medial end of the clavicle. A penetrating injury to the supraclavicular parasternal region,

www.eduwaves360.com | Telegram : @eduwaves360


like the one sustained by this patient, can pierce the parietal pleural membrane, permitting air to enter
the pleural space and causing an ipsilateral pneumothorax.

C - Trapezius muscle paresis due to spinal accessory nerve injury

Explanation Why

The spinal accessory nerve exits the skull through the jugular foramen and sends a branch to the
sternocleidomastoid muscle 3 cm below the mastoid process before entering the posterior triangle of
the neck to innervate the trapezius muscle. Spinal accessory nerve injury resulting in trapezius (and
sternocleidomastoid) paralysis would be expected if there were an injury to the upper aspect of the
posterior triangle of the neck.

D - Hemathorax due to azygos vein injury

Explanation Why

Azygos vein injury would cause hemothorax. However, the azygos vein drains into the superior vena
cava at the level of the 4th thoracic vertebra (T4). It does not extend into the supraclavicular space
and would therefore not be susceptible to injury in a penetrating trauma to the right supraclavicular
space.

E - Chylothorax due to thoracic duct injury

Explanation Why

Injury to the thoracic duct would cause chylothorax. The thoracic duct extends into the left
supraclavicular space just before it drains into the venous system at the junction of the left subclavian
and internal jugular vein. As this patient has sustained an injury to the right supraclavicular space, his
thoracic duct would not be injured.

www.eduwaves360.com | Telegram : @eduwaves360


F - Traumatic aneurysm due to internal carotid artery injury

Explanation Why

The internal carotid artery (ICA) usually arises from the common carotid artery at the level of the
fourth cervical vertebra (C4). A traumatic aneurysm of the ICA would therefore be expected
secondary to a neck injury at or above the C4 level.

www.eduwaves360.com | Telegram : @eduwaves360


Question # 23

A 68-year-old man comes to the physician because of a 5-month history of undulating, dull pain in his
right thigh. Physical examination shows a tender, round mass located above the right knee on the
anterior aspect of the thigh. An x-ray of the right thigh shows sunburst pattern of osteolytic bone lesions
in combination with sclerotic bone formation and invasion of the surrounding tissue. Despite
limb-sparing attempts, the patient has to undergo amputation of the right leg. A photograph of a
cross-section of the affected leg is shown. Which of the following is the strongest predisposing factor for
this patient's condition?

Answer Image

A Paget disease of bone

www.eduwaves360.com | Telegram : @eduwaves360


Answer Image

B Gardner syndrome

C t(11;22) translocation

D Hyperparathyroidism

E FGF receptor mutation

F Tuberous sclerosis

www.eduwaves360.com | Telegram : @eduwaves360


Hint

An osteoid-producing bone tumor with a sunburst pattern on x-ray suggests osteosarcoma.

www.eduwaves360.com | Telegram : @eduwaves360


Correct Answer

A - Paget disease of bone

Image

Explanation But

Primary osteosarcomas typically occur in the long bones of individuals between 10–25 years of age.

Explanation Why

Paget disease of bone (PDB) is strongly associated with the development of osteosarcoma. Increased
RANK/RANKL activity leads to an increase in osteoclast activity and subsequently to an increase in
osteoblast activity, which induces remodeling of the bone. This results in weaker, disorganized bone
tissue that is predisposed to malignant degeneration (sarcomatous transformation). Other risk factors
for secondary osteosarcoma include Li Fraumeni syndrome, radiation injury, and extensive bone
infarcts.

www.eduwaves360.com | Telegram : @eduwaves360


B - Gardner syndrome

Explanation Why

Gardner syndrome is a subtype of familial adenomatous polyposis (FAP) that involves the formation
of extracolonic tumors such as osteomas. These benign bone tumors are frequently seen on the skull
and mandible but rarely in the metaphyses of long bones. Moreover, this patient has no other
symptoms of Gardner syndrome (hypertrophy of the retinal pigment epithelium, adrenal adenomas,
desmoid tumors, or cutaneous lesions).

C - t(11;22) translocation

Image

Explanation Why

A t(11;22) translocation results in the production of the fusion protein EWS-FLI-1, which
predisposes patients to Ewing sarcoma, a malignant bone tumor of neuroectodermal cells. However,

www.eduwaves360.com | Telegram : @eduwaves360


Ewing sarcoma tends to occur in the diaphysis, not the metaphyses of long bones. Moreover, x-rays
would classically show “onion-skinning” but not a sunburst pattern.

D - Hyperparathyroidism

Explanation Why

Severe hyperparathyroidism is associated with osteitis fibrosa cystica, which often involves the
formation of brown tumors. These tumors consist of osteoclast clusters within a mixture of fibrous
tissue and woven bone and are high in hemosiderin (giving the lesions a brown color). However, this
patient shows none of the typical signs of hyperparathyroidism (hypercalcemia, renal stones,
polyuria, constipation, or psychiatric disturbances).

E - FGF receptor mutation

Explanation Why

FGF receptor mutations, which produce activated fibroblast growth factor receptor 3 (FGFR3), are
the cause of achondroplasia. Overactive FGFR3 prevents endochondral ossification and prevents
longitudinal bone growth. This mutation is the most common cause of dwarfism. However, this
patient does not present with dwarfism.

F - Tuberous sclerosis

Explanation Why

Tuberous sclerosis is a strong risk factor for the development of chordomas. These slow-growing
malignant bone tumors tend to manifest in regions developmentally related to the notochord (e.g., the
skull, vertebral bodies, and sacrococcygeal region) and are not normally found in the metaphyses of
long bones. However, this patient has no signs of tuberous sclerosis (intellectual disability, seizures,
typical skin manifestations like ash-leaf spots or Shagreen patches).

www.eduwaves360.com | Telegram : @eduwaves360


Question # 24

A 17-year-old boy comes to the physician because of a 3-month history of pain in his right shoulder. He
reports that he has stopped playing for his high school football team because of persistent difficulty
lifting his right arm. Physical examination shows impaired active abduction of the right arm from 0 to 15
degrees. After passive abduction of the right arm to 15 degrees, the patient is able to raise his arm above
his head. The dysfunctional muscle in this patient is most likely to be innervated by which of the
following nerves?

Answer Image

A Suprascapular nerve

B Long thoracic nerve

C Axillary nerve

D Upper subscapular nerve

E Accessory nerve

www.eduwaves360.com | Telegram : @eduwaves360


Hint

The findings in this patient are most likely caused by impairment of the supraspinatus muscle.

www.eduwaves360.com | Telegram : @eduwaves360


Correct Answer

A - Suprascapular nerve

Image

Explanation Why

The suprascapular nerve innervates the supraspinatus muscle and infraspinatus muscle, which are
part of the rotator cuff. The supraspinatus muscle facilitates initial abduction of the arm. Impaired
abduction from 0 to 15 degrees, which is seen in this patient, is a typical finding in supraspinatus
injury.

B - Long thoracic nerve

Explanation Why

The long thoracic nerve innervates the serratus anterior muscle. Dysfunction of the serratus anterior

www.eduwaves360.com | Telegram : @eduwaves360


muscle results in impaired arm abduction. However, impaired abduction at angles greater than 100
degrees would be expected, while this patient only has impaired abduction from 0 to 15 degrees.
Moreover, this patient lacks other features of serratus anterior muscle paresis, such as a winged
scapula.

C - Axillary nerve

Explanation Why

The axillary nerve innervates the deltoid muscle, the long head of the triceps brachii muscle, and the
teres minor muscle. Dysfunction of the deltoid muscle results in impaired arm abduction. However,
impaired abduction would be expected at angles greater than 15 degrees (up to 100 degrees), while
this patient only has impaired abduction from 0 to 15 degrees.

D - Upper subscapular nerve

Explanation Why

The upper subscapular nerve innervates the upper part of the subscapularis muscle. Dysfunction of
the subscapularis muscle results in impaired internal rotation and adduction of the arm rather than
impaired abduction of the arm, which is seen in this patient.

E - Accessory nerve

Explanation Why

The accessory nerve innervates the trapezius muscle and sternocleidomastoid muscle. Dysfunction of
the trapezius muscle results in impaired arm abduction. However, impaired abduction at angles
greater than 90 degrees would be expected, while this patient only has impaired abduction from 0 to
15 degrees. Moreover, this patient lacks other features of trapezius muscle paresis, such as drooping
of the shoulder and lateral winging of the scapula.

www.eduwaves360.com | Telegram : @eduwaves360


Question # 25

A 3855-g (8-lb 8-oz) newborn is examined shortly after birth. She was delivered at 40 weeks' gestation
by cesarean delivery because of breech presentation. Pregnancy was otherwise uncomplicated. Physical
examination shows asymmetric thigh creases. The left leg is shorter than the right leg and positioned in
external rotation. Which of the following is the most likely underlying cause of this patient's findings?

Answer Image

A Avascular necrosis of the femoral head

B Fracture of the femoral neck

C Inflammation of the hip synovium

www.eduwaves360.com | Telegram : @eduwaves360


Answer Image

D Abnormal development of the acetabulum

E Traction apophysitis of the tibial tubercle

F Displacement of the femoral epiphysis

www.eduwaves360.com | Telegram : @eduwaves360


Hint

Further examination of this patient will likely reveal a palpable clunk when the left hip is abducted,
flexed, and lifted forward (Ortolani sign) as well as a similar clunk when posterior pressure is applied on
the flexed hip (Barlow sign). These are signs of hip instability.

www.eduwaves360.com | Telegram : @eduwaves360


Correct Answer

A - Avascular necrosis of the femoral head

Image

Explanation Why

Avascular necrosis of the femoral head is the pathophysiology underlying Legg-Calvé-Perthes


disease, which may be unilateral or bilateral. Affected patients generally present with hip pain
(occasionally projecting to the knee), limited internal rotation, and an antalgic gait, none of which is
seen in this patient (and which would be difficult to assess at this time). Legg-Calvé-Perthes disease
typically manifests in children 4–10 years of age and would not be expected in a newborn.

www.eduwaves360.com | Telegram : @eduwaves360


B - Fracture of the femoral neck

Image

Explanation Why

Patients with a fracture of the femoral neck classically present with a shortened and externally
rotated leg (which is seen in this patient). However, femoral neck fractures typically occur in the
elderly (> 70 years of age) and would not be expected in a newborn.

C - Inflammation of the hip synovium

Explanation Why

Inflammation of the hip synovium is seen in patients with transient synovitis (TS). Patients with TS
typically present with hip pain and a limp, neither of which is seen in this patient (and which would
be difficult to assess at this time). This inflammatory condition, which is usually self-limiting, most
often occurs after viral illnesses in children 3–10 years of age and would not be likely in a newborn.

www.eduwaves360.com | Telegram : @eduwaves360


Moreover, the patient here has signs of hip instability (i.e., leg length discrepancy, asymmetrical
gluteal folds), which are not consistent with TS.

D - Abnormal development of the acetabulum

Image

Explanation But

Most cases of (DDH) involve the left hip. Several studies suggest that this might be caused by the
left side of the fetus lying against the mother's sacrum in utero, which results in abduction of the hip.
Leg length discrepancy and/or asymmetrical gluteal folds are absent in patients with bilateral DDH.

Explanation Why

Abnormal development of the acetabulum and an abnormal relationship between this structure and
the femoral head is the underlying cause of developmental dysplasia of the hip (DDH). Depending
on the affected structures, DDH can cause hip instability (capsular laxity), subluxation of the femoral
head, hip dislocation, and/or acetabular dysplasia. The etiology is not yet fully understood, but a
breech presentation, female sex, firstborn status, and a family history of DDH are important risk
factors. Early detection of DDH is important, as the prognosis worsens with increasing age. Hip
ultrasound is the imaging modality of choice in all infants younger than 4 months. A harness is the

www.eduwaves360.com | Telegram : @eduwaves360


treatment of choice for infants with DDH who are less than 6 months old.

E - Traction apophysitis of the tibial tubercle

Explanation Why

Traction apophysitis of the tibial tubercle is characteristic of Osgood-Schlatter disease. Osgood-


Schlatter disease classically manifests in children 9–14 years of age with anterior knee pain and point
tenderness over the tibial tuberosity, which is not reported in this patient. Moreover, this newborn has
signs of hip instability (i.e., leg length discrepancy, asymmetrical gluteal folds), which are not
consistent with Osgood-Schlatter disease.

F - Displacement of the femoral epiphysis

Explanation Why

Displacement of the femoral epiphysis is seen in patients with slipped capital femoral epiphysis
(SCFE), which can manifest with hip instability and external rotation of the leg. However, SCFE
occurs most often in obese boys 10–16 years of age and would not be expected in a newborn.

www.eduwaves360.com | Telegram : @eduwaves360


Question # 26

A 25-year-old woman comes to the physician because of pain and weakness in her left leg that started
2 days ago while running a marathon. Physical examination shows impaired flexion of the left knee joint
and impaired extension of the left hip joint. An MRI of the left lower extremity shows injury to the long
head of the biceps femoris muscle and the semitendinosus muscle. Which of the following is the most
likely origin of the injured muscles?

Answer Image

A Superior pubic ramus

B Ischial tuberosity

www.eduwaves360.com | Telegram : @eduwaves360


Answer Image

C Anterior superior iliac spine

D Greater trochanter

E Linea aspera

F Medial tibial condyle

www.eduwaves360.com | Telegram : @eduwaves360


Hint

The inferior gemellus, adductor magnus, and semimembranosus muscles also attach to this anatomical
landmark.

www.eduwaves360.com | Telegram : @eduwaves360


Correct Answer

A - Superior pubic ramus

Image

Explanation Why

The superior pubic ramus is part of the pubis and forms the anterior part of the acetabulum. It is the
origin of the adductor longus muscle, which runs along the inner side of the thigh, inserting on the
linea aspera. The long head of the biceps femoris and the semitendinosus muscle do not originate
from the superior pubic ramus. Also, the adductor longus muscle performs hip adduction and flexion,
but not knee flexion or hip extension.

www.eduwaves360.com | Telegram : @eduwaves360


B - Ischial tuberosity

Image

Explanation Why

The long head of the biceps femoris and the semitendinosus muscle originate from the ischial
tuberosity. The muscle belly of the long head of the biceps femoris joins together with the short head
of the biceps femoris to form a conjoined tendon, which inserts primarily into the lateral aspect of the
fibular head.

www.eduwaves360.com | Telegram : @eduwaves360


C - Anterior superior iliac spine

Image

Explanation Why

The anterior superior iliac spine is an anatomic landmark on the ilium, which forms the lateral part of
the pelvic girdle. It is the origin of the sartorius muscle, which runs along the inner side of the thigh
and inserts on the medial upper side of the tibia. Although the sartorius facilitates knee flexion, it
does not allow for hip extension. Instead, it performs hip flexion. The long head of the biceps
femoris and the semitendinosus muscle do not originate from the anterior superior iliac spine.

D - Greater trochanter

Explanation Why

The greater trochanter lies on the lateral aspect of the junction between the neck and shaft of the
femur. It is the origin of the vastus lateralis muscle, which is one of the four heads of the quadriceps

www.eduwaves360.com | Telegram : @eduwaves360


femoris muscle, but it is not the origin of the long head of the biceps femoris or the semitendinosus
muscle. Also, the vastus lateralis performs knee extension but not knee flexion or hip extension.

E - Linea aspera

Explanation Why

The linea aspera is a bony ridge located on the posterior side of the femur shaft. Together with the
upper supracondylar line, the linea aspera forms the origin of the short head of the biceps femoris
muscle. It is not the origin of the long head or semitendinous muscle.

F - Medial tibial condyle

Explanation Why

The tibia forms the medial and lateral condyle on its upper end. The semimembranosus muscle
inserts on the medial tibial condyle. It is not the origin of the biceps femoris or semitendinosus
muscle.

www.eduwaves360.com | Telegram : @eduwaves360


Question # 27

A 24-year-old man comes to the physician because of chronic fatigue and generalized weakness after
exertion. His legs feel stiff after walking long distances and he has leg cramps after climbing stairs. His
symptoms are always relieved by rest. Urine dipstick shows 3+ blood and urinalysis is negative for
RBCs. Baseline venous lactate and serum ammonia levels are collected, after which a blood pressure
cuff is attached to the upper right arm. The patient is asked to continuously pump his right arm with the
cuff inflated and additional venous samples are collected at 2-minute intervals. Analysis of the venous
blood samples shows that, over time, serum ammonia levels increase and venous lactate levels remain
stable. A biopsy of the right gastrocnemius muscle will most likely show which of the following?

Answer Image

A Intrafascicular CD8+ lymphocytic infiltration

B Endomysial fibrosis with absent dystrophin

C Intermyofibrillar proliferation of mitochondria

D Subsarcolemmal acid–Schiff-positive deposits

E Perivascular CD4+ lymphocytic infiltrate

www.eduwaves360.com | Telegram : @eduwaves360


Hint

This patient's symptoms of generalized muscle weakness, myoglobinuria (positive blood but negative
RBCs), and exercise intolerance suggest a myopathy. The forearm exercise test finding of a flat venous
lactate curve despite rising ammonia levels is consistent with McArdle disease.

www.eduwaves360.com | Telegram : @eduwaves360


Correct Answer

A - Intrafascicular CD8+ lymphocytic infiltration

Explanation Why

An intrafascicular CD8+ lymphocytic infiltrate on biopsy is consistent with polymyositis. While


polymyositis manifests with muscle weakness, which is seen in this patient, it is typically proximal
and symmetrical in nature and is especially pronounced in the pelvic and shoulder girdles.
Furthermore, cramps are uncommon, and the muscle weakness is not profoundly exacerbated by
exertion, which is the case here.

B - Endomysial fibrosis with absent dystrophin

Explanation Why

Endomysial fibrosis with absent dystrophin is seen in individuals with muscular dystrophies, such as
Duchenne muscular dystrophy (DMD). Muscle fiber degeneration occurs secondary to altered
dystrophin protein, which is caused by underlying dystrophin gene mutations. Complete impairment
of dystrophin results in the disturbance of cellular signaling pathways, which promotes necrosis of
these cells. The necrotic cells are subsequently replaced by connective and fatty tissue. While
patients with a dystrophin gene mutation may present with muscle weakness, which is seen in this
patient, DMD appears significantly early in life, with most affected individuals using a wheelchair by
late childhood. Furthermore, classic features of DMD (e.g., Gower sign, calf pseudohypertrophy) are
not seen in this patient.

C - Intermyofibrillar proliferation of mitochondria

Explanation Why

Intermyofibrillar proliferation of mitochondria would appear on biopsy as ragged red fibers, which
are characteristic of mitochondrial myopathies. While there are several forms of mitochondrial
myopathies, patients typically present with exercise intolerance, which is seen here. However, the
forearm exercise test finding of flat venous lactate curve despite rising ammonia levels is

www.eduwaves360.com | Telegram : @eduwaves360


inconsistent with this diagnosis.

D - Subsarcolemmal acid–Schiff-positive deposits

Explanation Why

Subsarcolemmal acid-Schiff-positive deposits indicate an increased accumulation of glycogen in


muscle cells, which is seen in McArdle disease. The accumulation of glycogen in muscle cells is
caused by a deficiency of muscle phosphorylase (myophosphorylase), which is an enzyme necessary
for the rate-limiting step of glycogenolysis. The inability to break down glycogen leads to
generalized muscle weakness and exercise intolerance, which are both seen in this patient.
Furthermore, this inability to release glucose leads to a lack of anaerobic glycolysis, preventing a
normal rise in lactate during the forearm exercise test.

E - Perivascular CD4+ lymphocytic infiltrate

Explanation Why

A perivascular CD4+ lymphocytic infiltrate on biopsy is consistent with dermatomyositis.


Dermatomyositis is an inflammatory myopathy typically characterized by proximal and symmetrical
muscle weakness (especially of the pelvic and shoulder girdles) with skin involvement (e.g., Gottron
papules or heliotrope rash). These features are not seen in this patient.

www.eduwaves360.com | Telegram : @eduwaves360


Question # 28

A 62-year-old woman comes to the physician because of a 6-month history of progressive pain and
stiffness of the fingers of her right hand. The stiffness is worse at the end of the day. She recently retired
after working for 28 years as a typist at a data entry firm. Physical examination shows swelling,
joint-line tenderness, and decreased range of motion of the first metacarpophalangeal joint and the distal
interphalangeal joints of the right hand. Discrete, hard, mildly tender nodules are palpated over the 2nd
and 4th distal interphalangeal joints of the right hand. An x-ray of her right hand shows narrowing of the
interphalangeal joint spaces with subchondral sclerosis and osteophytes. Which of the following is the
most likely underlying mechanism of this patient's condition?

Answer Image

Monosodium urate crystal


A
precipitation in the joints

B Bacterial infection of the joint space

www.eduwaves360.com | Telegram : @eduwaves360


Answer Image

Autoimmune-mediated cartilage
C
erosion

Calcium pyrophosphate dihydrate


D
crystal precipitation in the joints

E Degenerative disease of the joints

www.eduwaves360.com | Telegram : @eduwaves360


Hint

This patient presents with asymmetric, polyarticular arthritis and Heberden nodes (tender nodules over
the DIP joints). The osteophytes seen on imaging are consistent with a specific diagnosis.

www.eduwaves360.com | Telegram : @eduwaves360


Correct Answer

A - Monosodium urate crystal precipitation in the joints

Image

Explanation Why

Monosodium urate crystal precipitation in the joints causes gout, which is associated with
hyperuricemia and classically manifests as acute pain and inflammation affecting a single joint of the
lower extremity (especially the first metatarsophalangeal joint). Risk factors for gout include thiazide
diuretic use and alcohol consumption, neither of which is seen in this patient. The involvement of
multiple finger joints and absence of inflammation (e.g., erythema, warmth) seen in this patient make
gout very unlikely.

www.eduwaves360.com | Telegram : @eduwaves360


B - Bacterial infection of the joint space

Image

Explanation Why

Bacterial infection of the joint space is characteristic of septic arthritis, which can also cause pain,
joint-line tenderness, swelling, and decreased range of motion. Septic arthritis is usually acute,
asymmetric, monoarticular, and accompanied by signs of inflammation (e.g., erythema, warmth in
the affected joint, leukocytosis). The polyarticular and chronic nature of this woman's symptoms,
along with the absence of fever and risk factors (e.g., IV drug use), make septic arthritis unlikely in
this patient.

www.eduwaves360.com | Telegram : @eduwaves360


C - Autoimmune-mediated cartilage erosion

Image

Explanation Why

Autoimmune-mediated inflammation of the joints, as seen in rheumatoid arthritis (RA), results in the
destruction and erosion of joint cartilage. RA commonly involves the hand joints, but it is usually
symmetric, does not affect the DIP joints, and is worse with inactivity (> 30 minutes of morning
stiffness). The subchondral sclerosis and osteophytes seen on this patient's imaging are also atypical
for RA, which is instead characterized by joint space erosions and cysts on imaging.

www.eduwaves360.com | Telegram : @eduwaves360


D - Calcium pyrophosphate dihydrate crystal precipitation in the joints

Image

Explanation Why

Calcium pyrophosphate dihydrate crystal deposition disease (e.g., pseudogout) typically manifests as
recurrent episodes of acute or subacute inflammatory arthritis, usually affecting a solitary large joint
(e.g., the knee). This patient lacks signs of inflammation (e.g., erythema, warmth of the joint) and
chondrocalcinosis on x-ray, making this diagnosis unlikely.

www.eduwaves360.com | Telegram : @eduwaves360


E - Degenerative disease of the joints

Image

Explanation Why

Osteoarthritis (OA) is a form of degenerative arthritis characterized by pain and stiffness that
worsens with activity. In this patient, it is likely related to her 28-year history of working as a typist.
While OA can affect any joint, it frequently affects the knees and hand joints (most commonly the
DIP and PIP). The Heberden nodes seen in this patient are a characteristic sign of OA, which can
also manifest with Bouchard nodes. Typical imaging findings include joint space narrowing,
subchondral sclerosis, and osteophyte formation.

www.eduwaves360.com | Telegram : @eduwaves360


Question # 29

A previously healthy 61-year-old man comes to the physician because of bilateral knee pain for the past
year. The pain is worse with movement and is relieved with rest. Physical examination shows crepitus,
pain, and decreased range of motion with flexion and extension of both knees. There is no warmth,
redness, or swelling. X-rays of both knees show irregular joint space narrowing, osteophytes, and
subchondral cysts. Which of the following is the most appropriate pharmacotherapy?

Answer Image

A Prednisone

B Methotrexate

C Naproxen

D Allopurinol

E Celecoxib

F Infliximab

www.eduwaves360.com | Telegram : @eduwaves360


Hint

This patient's presentation (chronic knee pain on exertion that is relieved with rest), physical
examination findings (crepitus, decreased range of motion), and x-ray findings are consistent with
osteoarthritis.

www.eduwaves360.com | Telegram : @eduwaves360


Correct Answer

A - Prednisone

Explanation Why

Oral prednisone can be used to treat inflammatory arthritis, such as rheumatoid arthritis. This patient
lacks any signs of inflammatory arthritis (e.g., erythema, warmth of the affected joint), and the
chronicity and bilateral nature of his symptoms make osteoarthritis much more likely. In patients
with severe pain due to osteoarthritis, intra-articular glucocorticoids can be used but the systemic
administration of glucocorticoids is not recommended.

B - Methotrexate

Explanation Why

Methotrexate is a DMARD that reduces inflammation, prevents joint destruction, and slows disease
progression in individuals with rheumatoid arthritis. This patient's history and x-ray findings
(subchondral cysts, osteophytes, and joint space narrowing) suggest osteoarthritis, not rheumatoid
arthritis. Because osteoarthritis is not an inflammatory process, methotrexate would have no
therapeutic benefit.

www.eduwaves360.com | Telegram : @eduwaves360


C - Naproxen

Image

Explanation Why

NSAIDs such as naproxen are used as first-line pharmacotherapy for patients with osteoarthritis.
Because of the degenerative nature of osteoarthritis, management should be focused on treating pain
as well as maintaining and improving joint mobility and function. Other medications used for the
treatment of osteoarthritis include acetaminophen, topical capsaicin, and intra-articular
glucocorticoids. Glucocorticoid injections are usually only indicated for acute, severe pain in which
NSAIDs have not been effective.

D - Allopurinol

Explanation Why

Allopurinol is a reversible xanthine oxidase inhibitor that prevents the conversion of hypoxanthine

www.eduwaves360.com | Telegram : @eduwaves360


and xanthine into uric acid. Allopurinol is used as a maintenance therapy for patients with gout,
which is associated with hyperuricemia and typically has an acute presentation with asymmetric joint
swelling and pain (classically affecting the big toe). This man's subacute and bilateral symptoms are
consistent with osteoarthritis, for which allopurinol is not effective.

E - Celecoxib

Explanation Why

Celecoxib is a selective COX-2 inhibitor that is most commonly used as analgesic therapy in patients
with gastric or gastroduodenal ulcers because of the drug's reversible and selective inhibition of
cyclooxygenase 2 without affecting cyclooxygenase 1. Although celecoxib can also be used to treat
osteoarthritis, it is less effective and more expensive than other medications, and it has an increased
risk for cardiovascular side effects. Therefore, celecoxib is only indicated in patients with
gastrointestinal concerns, which this patient does not report.

F - Infliximab

Explanation Why

Infliximab is a biological agent used for the treatment of refractory inflammatory arthritis, such as
rheumatoid arthritis, psoriatic arthritis, and ankylosing spondylitis. This patient does not have signs
of local inflammation such as erythema, swelling, and warmth in the affected joints, making
inflammatory arthritis extremely unlikely. Infliximab is not indicated for the treatment of
osteoarthritis.

www.eduwaves360.com | Telegram : @eduwaves360


Question # 30

A 71-year-old man with type 2 diabetes mellitus comes to the physician because of a 9-month history of
pain and stiffness in the right knee. He reports that the stiffness lasts approximately 10 minutes after
waking up and that the pain is worse in the evening. There is no history of trauma. He is 175 cm
(5 ft 9 in) tall and weighs 102 kg (225 lb); BMI is 33 kg/m2. Examination of the right knee shows
tenderness in the anteromedial joint line and crepitus during knee movement. Laboratory studies show
an erythrocyte sedimentation rate of 15 mm/h and a serum uric acid concentration of 6.9 mg/dL. Which
of the following is the most likely finding on imaging of the right knee?

Answer Image

Osteophytes and narrowing of the


A
joint-space

Marginal bony erosions and


B opacification of periarticular soft
tissue

www.eduwaves360.com | Telegram : @eduwaves360


Answer Image

Periarticular osteopenia and pannus


C
formation

Loculated epiphyseal cyst with


D
thinning of the overlying cortex

Bony ankylosis and bone


E
proliferation at the entheses

www.eduwaves360.com | Telegram : @eduwaves360


Answer Image

Calcification of the synovium and


F
cartilage

www.eduwaves360.com | Telegram : @eduwaves360


Hint

Pain that is worse in the evening (due to ambulation during the day), brief morning stiffness
(< 30 minutes), a normal ESR, involvement of the medial knee compartment, and the presence of
crepitus with joint movement suggest non-inflammatory degenerative arthritis (osteoarthritis) as the
cause of this patient's chronic monoarticular pain. Advanced age and obesity are risk factors for primary
osteoarthritis.

www.eduwaves360.com | Telegram : @eduwaves360


Correct Answer

A - Osteophytes and narrowing of the joint-space

Image

Explanation But

In patients with OA of the hand, these osteophytes can be palpated as bony spurs (Heberden nodes,
Bouchard nodes) on examination.

Explanation Why

Osteoarthritis (OA) is caused by articular cartilage degeneration, which results in narrowing of the
joint space. Unlike inflammatory degenerative arthritides, which cause uniform joint space
narrowing, OA is associated with focal joint space narrowing that is most prominent in regions
where stress is concentrated (e.g., medial compartment of the knee, superolateral aspect of the hip).
Subsequently, reactive bone formation and trabecular compression cause subchondral sclerosis, and
entry of synovial fluid into the bone forms subchondral cysts. To limit damage to the bone, remnants
of the degenerating cartilage undergo endochondral ossification. This results in the formation of
osteophytes at the margin of the joint space.

www.eduwaves360.com | Telegram : @eduwaves360


B - Marginal bony erosions and opacification of periarticular soft tissue

Image

Explanation Why

Marginal bony erosions and opacification of periarticular soft tissue (gouty tophi) are findings of
chronic gout, which can occasionally involve the knee and result in chronic joint pain due to
secondary osteoarthritis. However, the patient would have hyperuricemia, unlike the normal uric acid
levels seen here, and there would be a past history of recurrent attacks of acute inflammatory gout
preceding the onset of joint pain. Osteoarthritis unrelated to gout would not be associated with
marginal erosions or periarticular soft tissue opacification.

www.eduwaves360.com | Telegram : @eduwaves360


C - Periarticular osteopenia and pannus formation

Image

Explanation Why

Periarticular osteopenia and pannus formation are classic findings in rheumatoid arthritis (RA),
which can cause joint pain and early morning stiffness. However, morning stiffness associated with
RA typically lasts longer than 30 minutes and the pain, which improves with movement, is usually
less in the evening. Furthermore, RA typically causes an elevated ESR and manifests more
commonly as oligoarthritis of small joints of the hand rather than monoarthritis of a large joint such
as the knee. Periarticular osteopenia and pannus formation are not seen with osteoarthritis.

www.eduwaves360.com | Telegram : @eduwaves360


D - Loculated epiphyseal cyst with thinning of the overlying cortex

Image

Explanation Why

A large, loculated epiphyseal cyst (soap-bubble appearance) with thinning of the overlying cortex
describes the radiographic appearance of a giant cell tumor (osteoclastoma), which often affects the
knee joint. However, an osteoclastoma typically occurs in younger patients (20–40 years) and it
would not be expected to cause joint line tenderness or crepitus. Patients with osteoarthritis would
develop subchondral cysts, but such cysts are neither loculated nor associated with thinning of the
overlying bone cortex.

www.eduwaves360.com | Telegram : @eduwaves360


E - Bony ankylosis and bone proliferation at the entheses

Image

Explanation Why

Bony ankylosis and bone proliferation at the entheses (enthesophytes) are characteristic of certain
seronegative arthritides (e.g., psoriatic arthritis, ankylosing spondylitis). However, while psoriatic
arthritis typically affects small joints of the hands and would be associated with a history of scaly
cutaneous plaques, ankylosing spondylitis would affect the spine. In patients with osteoarthritis, bone
proliferation occurs within the joint capsule at the ends of joint lines rather than at entheses, and
bony ankylosis would not be expected.

www.eduwaves360.com | Telegram : @eduwaves360


F - Calcification of the synovium and cartilage

Image

Explanation Why

Calcification of the synovium and cartilage (chondrocalcinosis) is most commonly caused by


calcium pyrophosphate dihydrate (CPPD) disease, which typically occurs in elderly patients and
usually involves the knee. Chronic CPPD can manifest with features of osteoarthritis (pyrophosphate
arthropathy), but it more commonly affects the patellofemoral joint and the lateral knee
compartment, unlike here, where there is involvement of the medial knee compartment. Also, a
patient with osteoarthritis due to CPPD would have had a past history of recurrent attacks of acute
CPPD. Osteoarthritis unrelated to CPPD would not be associated with calcification of the synovium
or cartilage.

www.eduwaves360.com | Telegram : @eduwaves360


Question # 31

A 67-year-old man comes to the physician because of worsening lower back pain for 6 weeks. He
reports that the pain is most intense with movement and that it sometimes occurs at night. Over the past
3 months, he has noticed a weakened urinary stream. He has not seen any blood in his urine. His only
daily medication is ibuprofen. Examination shows no spinal deformities. Palpation of the lumbar spinal
process elicits tenderness. Muscle strength is normal. Which of the following is the most likely cause of
this patient’s back pain?

Answer Image

A Malignancy

B Lumbar spinal stenosis

C Osteoporosis

D Spinal epidural abscess

E Disc herniation

F Lumbar strain

www.eduwaves360.com | Telegram : @eduwaves360


Hint

The most important risk factor for this patient's underlying disease is advanced age (> 50 years).

www.eduwaves360.com | Telegram : @eduwaves360


Correct Answer

A - Malignancy

Explanation But

In addition to treating the underlying malignancy, therapy options include pain management and
local radiation.

Explanation Why

Subacute progressive lumbar back pain and a weakened urinary stream in a patient > 50 years of age
should raise suspicion for advanced-stage prostate cancer with bony metastasis. Prostate cancer is
frequently associated with skeletal metastases, with the spine being the most common location.
Accordingly, some patients present with lower back pain as the initial manifestation. Red flags for
metastatic bone disease include a history of cancer, advanced age, unexplained weight loss, and bone
pain that does not improve after one month. Besides pain, which commonly occurs at night and
exacerbates with movement, patients with metastatic bone disease can present with pathologic
fractures, hypercalcemia, and neurological deficits due to spinal cord compression.

B - Lumbar spinal stenosis

Explanation Why

Lower back pain is a common manifestation of lumbar spinal stenosis (LSS), a condition that mostly
affects individuals > 60 years of age. LSS typically causes bilateral radiating pain to the buttocks
and/or legs that is exacerbated by activities involving lumbar extension (e.g., standing, walking
downhill) and relieved by sitting or lying down (i.e., lumbar flexion), which has not been reported by
this patient. Furthermore, he does not have typical symptoms of LSS, such as cramping, numbness,
weakness, or tingling in the legs. While LSS can rarely lead to urinary retention symptoms secondary
to cauda equina syndrome, this patient's normal sensorimotor function makes such a diagnosis
unlikely.

www.eduwaves360.com | Telegram : @eduwaves360


C - Osteoporosis

Explanation Why

Individuals with osteoporosis have an increased susceptibility to pathological vertebral compression


fractures. These fractures may manifest with lower back pain and point tenderness, both of which are
seen here. However, a more acute presentation would be expected in a fracture. Moreover,
osteoporosis most commonly affects postmenopausal women, individuals > 70 years of age, and
individuals with risk factors such as long-term steroid therapy. While vertebral compression fractures
can rarely lead to urinary retention symptoms secondary to cauda equina syndrome, this patient's
normal sensorimotor function makes such a diagnosis unlikely.

D - Spinal epidural abscess

Explanation Why

Subacute lower back pain worsened by palpation, which is seen here, is a typical manifestation of a
spinal epidural abscess (SEA). However, SEA is commonly associated with further symptoms, such
as fever, malaise, and, eventually, neurologic deficits (secondary to spinal cord compression).
Moreover, this patient has no risk factors for this condition (e.g., IV drug use, recent invasive spinal
procedures, immunosuppression). While SEA can rarely lead to urinary retention symptoms
secondary to cauda equina syndrome, this patient's normal sensorimotor function makes such a
diagnosis unlikely.

E - Disc herniation

Explanation Why

Lumbar disc herniation can manifest with severe lower back pain, which is seen in this patient.
However, disc herniation typically manifests with an acute stabbing pain that radiates to the legs and
is associated with neurological deficits, such as paresthesia, loss of deep tendon reflexes, and muscle
weakness, none of which are seen here. Moreover, disc herniation is usually triggered by physical
exertion (e.g., bending, heavy lifting), which this patient has not reported. While a large
posteromedial disc herniation can lead to urinary retention symptoms secondary to cauda equina

www.eduwaves360.com | Telegram : @eduwaves360


syndrome, this patient's normal sensorimotor function makes such a diagnosis unlikely.

F - Lumbar strain

Explanation Why

Lumbar muscle strain is the most common cause of lower back pain. It can manifest with lower back
pain that is most intense with movement, reproducible on palpation, and not associated with sensory
or motor dysfunction, as seen here. Nocturnal pain and a weakened urinary stream, however, are not
associated with lumbar strain. Lack of improvement after 6 weeks of therapy with NSAIDs should
further raise suspicion for a nonmuscular cause of the pain. Moreover, lumbar muscle strain is
usually triggered by physical exertion (e.g., heavy lifting), which this patient has not reported.

www.eduwaves360.com | Telegram : @eduwaves360


Question # 32

A 9-year-old boy is brought to the physician for evaluation of short stature. He is at the 5th percentile for
height, 65th percentile for weight, and 95th percentile for head circumference. Examination shows
midface retrusion, a bulging forehead, and flattening of the nose. The extremities are disproportionately
short. He was adopted and does not know his biological parents. The patient’s condition is an example
of which of the following genetic phenomena?

Answer Image

A Anticipation

B Variable expressivity

C Imprinting

D Allelic heterogeneity

E Complete penetrance

F Codominance

www.eduwaves360.com | Telegram : @eduwaves360


Hint

This patient's short stature, distinctive craniofacial features (midface retrusion, prominent forehead, nose
flattening), and disproportionate extremity length indicate achondroplasia.

www.eduwaves360.com | Telegram : @eduwaves360


Correct Answer

A - Anticipation

Explanation Why

Anticipation describes a phenomenon in which a genetic disease increases in severity over several
generations or manifests earlier with each generation. It is seen in conditions caused by trinucleotide
repeat expansions (e.g., Huntington disease, myotonic dystrophy, fragile X syndrome), in which the
number of repeats can increase with each generation, resulting in a more severe or earlier phenotypic
expression. Anticipation, however, does not occur in achondroplasia.

B - Variable expressivity

Explanation Why

Variable expressivity describes a phenomenon in which clinical or phenotypic features differ among
individuals that carry the same altered genotype. Many genetic diseases show some variability in
expressivity of a phenotype, with manifestations ranging from mild to life-threatening (e.g., Marfan
syndrome). Factors influencing expressivity include other genetic phenomena and epigenetic or
environmental factors. However, all patients with achondroplasia typically show the same degree of
expressivity.

C - Imprinting

Explanation Why

Imprinting describes an epigenetic mechanism of gene regulation in which either the paternal or
maternal allele of a gene is inactivated (imprinted) via DNA methylation while the other allele is
expressed. This mechanism plays a role in the genetics of Angelman syndrome, in which the
paternally inherited UBE3A gene is imprinted (silenced) while the corresponding maternal allele is
deleted or mutated. Imprinting also plays a role in Prader-Willi syndrome, in which genetic
information on the maternal chromosome 15 is imprinted (silenced) while the corresponding allele
on the paternal allele is deleted or mutated. However, achondroplasia is not caused by imprinting.

www.eduwaves360.com | Telegram : @eduwaves360


D - Allelic heterogeneity

Explanation Why

Allelic heterogeneity describes a phenomenon in which different mutations within a given gene can
lead to the same disease. Depending on the type of mutation, the functional consequences can range
from mild to significant, resulting in a large variety of phenotypic expressions. Examples include β-
thalassemia and G6PD deficiency. Achondroplasia, however, is always caused by a gain-of-function
mutation in the fibroblast growth factor receptor 3 gene and is not associated with allelic
heterogeneity.

E - Complete penetrance

Image

www.eduwaves360.com | Telegram : @eduwaves360


Explanation Why

Achondroplasia is an autosomal dominant disorder with complete penetrance, meaning that all
individuals who carry the altered genotype (i.e., a gain-of-function mutation in the fibroblast growth
factor receptor 3 gene) will have the disease. Complete penetrance typically occurs when the
phenotype is mostly determined by a single gene, such as in achondroplasia. Further examples of
genetic conditions with full penetrance include Huntington disease and neurofibromatosis type I.

F - Codominance

Explanation Why

Codominance describes a phenomenon in which both alleles of a gene contribute to phenotypic


expression. For example, in the ABO blood group system, the blood groups A and B are codominant.
However, achondroplasia is an autosomal dominant disease in which only the mutant allele
determines the phenotype.

www.eduwaves360.com | Telegram : @eduwaves360


Question # 33

A 10-year-old boy is brought to the physician by his parents for a follow-up examination. He has had a
short stature since birth and underwent diagnostic testing. Genetic analyses showed a gain of function
mutation in the fibroblast growth factor receptor 3 (FGFR3) gene. He has met all developmental
milestones. He is at the 10th percentile for height and 90th percentile for weight. Which of the following
additional findings is most likely on physical examination?

Answer Image

A Absent collar bones

B Blue sclerae

C Long extremities

D Small genitalia

E Frontal bossing

www.eduwaves360.com | Telegram : @eduwaves360


Hint

Constitutive activation of FGFR3 impairs chondrocyte proliferation in the growth plate and is the cause
of achondroplasia.

www.eduwaves360.com | Telegram : @eduwaves360


Correct Answer

A - Absent collar bones

Explanation Why

This patient's short stature since birth might raise concern for cleidocranial dysplasia (CCD), which
is caused by an autosomal dominant mutation in a transcription factor involved in osteoblast
development and is characterized by absent collar bones. CCD typically manifests with mildly
disproportionate short stature (short limb length relative to the trunk), other skeletal abnormalities
(e.g., delayed closure of cranial sutures, pelvic abnormalities, scoliosis) that often lead to gross motor
delay, dental abnormalities (e.g., supernumerary teeth, unerupted permanent teeth), and characteristic
facial dysmorphism (wide face, prominent forehead and chin, maxillary hypoplasia).

B - Blue sclerae

Explanation Why

This patient's short stature since birth should raise concern for osteogenesis imperfecta (OI), which is
caused by an autosomal dominant mutation in type 1 collagen genes (COL1A1 or COL1A2) and can
manifest with blue sclera. Further symptoms include progressive hearing loss, brittle bones and
recurrent pathological fractures, and dentinogenesis imperfecta (brittle, opalescent teeth), none of
which is seen in this patient. OI is managed with IV bisphosphonates to decrease the risk of
fractures.

C - Long extremities

Explanation Why

Disproportionately long extremities are characteristic of Marfan syndrome (caused by a mutation in


fibrillin-1) and homocystinuria (caused by mutations in enzymes of homocysteine metabolism). Both
conditions are associated with tall stature rather than the short stature seen here. Other common
features of both Marfan syndrome and homocystinuria include pectus deformity, arachnodactyly,
joint hyperlaxity, skin hyperelasticity, and scoliosis, which are absent in this patient.

www.eduwaves360.com | Telegram : @eduwaves360


D - Small genitalia

Explanation Why

This patient's short stature since birth may raise concern for Laron syndrome, which is caused by
loss-of-function mutations in the growth hormone (GH) receptor gene and can manifest with small
genitalia. Laron syndrome typically manifests with disproportionate short stature (average-sized
torso with short extremities), distinctive facial dysmorphism (saddle nose, prominent forehead,
microcephaly), delayed gross motor developmental milestones and metabolic abnormalities (e.g.,
osteopenia, obesity, dyslipidemia, hypoglycemia).

E - Frontal bossing

Image

www.eduwaves360.com | Telegram : @eduwaves360


Explanation But

Treatment of achondroplasia involves early administration of growth hormone (1–6 years) and
surgical corrections for skeletal complications.

Explanation Why

Achondroplasia causes disproportionate short stature (average-sized torso with short extremities),
characteristic facial dysmorphism (frontal bossing, macrocephaly, midface retrusion, nose
flattening), and spinal abnormalities (e.g., scoliosis, thoracolumbar kyphosis). Patients with
achondroplasia have normal intelligence and typically meet all developmental milestones, as seen
here. Approximately 80% of achondroplasia cases are due to sporadic FGFR3 mutations and
approximately 20% of cases are caused by autosomal dominant inheritance with full penetrance. The
probability of new mutations increases with paternal age at the time of conception.

www.eduwaves360.com | Telegram : @eduwaves360


Question # 34

A 68-year-old woman is brought to the emergency room because of pain in her right shoulder after
falling down the stairs in her apartment. She has a history of postmenopausal osteoporosis. Her current
medications are alendronate, vitamin D, and calcium supplementation. Physical examination shows
shortening of the right arm as well as tenderness and swelling over the right shoulder. The range of
motion of the right arm is limited by pain. An x-ray of the right shoulder shows a fracture of the surgical
neck of the humerus. Which of the following movements is most likely to be impaired in this patient?

Answer Image

A Wrist extension

B Elbow flexion

www.eduwaves360.com | Telegram : @eduwaves360


Answer Image

C Arm abduction

D Thumb adduction

E Wrist flexion

www.eduwaves360.com | Telegram : @eduwaves360


Hint

Examination would likely also show flattening of the deltoid prominence.

www.eduwaves360.com | Telegram : @eduwaves360


Correct Answer

A - Wrist extension

Image

Explanation Why

Impaired wrist extension and subsequent wrist drop due to weakness of the dorsal forearm muscles is
a finding of proximal radial nerve injury. The radial nerve originates from the posterior cord of the
brachial plexus and travels adjacent to the deep brachial artery in the radial groove, where it is most
susceptible to injury. It is most likely to be damaged because of a mid-shaft and distal humerus
fracture, whereas this patient has a fracture of the proximal humerus. Injury to the radial nerve within
the radial groove typically also results in sensory deficits over the posterior forearm and dorsal
aspect of the thumb, second and third digit, and lateral fourth digit.

www.eduwaves360.com | Telegram : @eduwaves360


B - Elbow flexion

Image

Explanation Why

Impaired elbow flexion due to weakness of the brachialis and coracobrachialis muscles is a sign of
musculocutaneous nerve injury. This nerve arises from the lateral cord of the brachial plexus and
travels downwards between the biceps brachii and the brachialis muscles. Because it is largely
protected by the biceps, injury due to mid-shaft humerus fractures is rare. This patient's x-ray,
however, shows a fracture of the proximal humerus. Excessive lateral flexion of the neck (e.g.,
trauma, football injuries) can lead to Erb palsy with injury to the musculocutaneous nerve. Further
symptoms of musculocutaneous nerve injury include impaired forearm supination and sensory
deficits over the lateral forearm.

www.eduwaves360.com | Telegram : @eduwaves360


C - Arm abduction

Image

Explanation But

Further common causes of axillary nerve palsy include anterior shoulder dislocation and iatrogenic
injury (e.g., during shoulder reconstruction procedures, rotator cuff surgery).

Explanation Why

Impaired arm abduction over 15 degrees due to weakness of the deltoid muscle is a classic finding of
axillary nerve injury. The axillary nerve originates from the posterior cord of the brachial plexus and
carries fibers from C5–6 through the quadrangular space to innervate the deltoid. Due to its
proximity to the surgical neck of the humerus, the nerve can become injured by a proximal humerus
fracture, as seen in this patient. Further symptoms of axillary nerve injury include impaired external
rotation of the arm (due to paralysis of the teres minor muscle) and sensory loss over the lateral
shoulder.

www.eduwaves360.com | Telegram : @eduwaves360


D - Thumb adduction

Image

Explanation Why

Impaired thumb adduction due to weakness of the adductor pollicis muscle is a sign of proximal
ulnar nerve injury (Froment sign). The ulnar nerve originates from the medial cord of the brachial
plexus and runs along the posteromedial aspect of the humerus until passing through the cubital
tunnel, where it is most superficial and susceptible to injury. It is most likely to be damaged due to a
distal humerus fracture at the medial epicondyle, whereas this patient has a fracture of the proximal
humerus. Further symptoms of proximal ulnar nerve injury include an ulnar claw and sensory
deficits over the fourth and fifth digit.

www.eduwaves360.com | Telegram : @eduwaves360


E - Wrist flexion

Image

Explanation Why

Impaired wrist flexion due to weakness of the ventral forearm muscles is a sign of proximal median
nerve injury. The median nerve originates from the lateral and medial cord of the brachial plexus and
crosses the elbow anteriorly with the brachial artery, where the nerve is most at risk for injury. It is
most likely to be damaged due to a supracondylar humerus fracture, whereas this patient has a
fracture of the proximal humerus. Further symptoms of proximal median nerve injury include the
“hand of benediction” and sensory deficits over the palmar aspect of the first three digits and the
radial side of the fourth digit.

www.eduwaves360.com | Telegram : @eduwaves360


Question # 35

A 62-year-old man comes to the physician for hematemesis and progressive heartburn over the past
5 days. Ten days ago, he was started on a medication to treat a condition that causes hearing difficulties
and pain of the lower legs. He has no other history of serious illness. He has smoked 1 pack of cigarettes
daily for the past 20 years. Physical examination shows bowing of the tibias. Upper endoscopy shows
inflammation of the mucosa and a 1-cm punched-out ulcer in the distal esophagus. Which of the
following drugs is the most likely cause of the patient's current condition?

Answer Image

A Calcium citrate

B Denosumab

C Risedronate

D Calcitonin

E Prednisolone

F Acetaminophen

www.eduwaves360.com | Telegram : @eduwaves360


Hint

This patient's hearing difficulties, bone pain, and tibial bowing raise suspicion for Paget disease of bone.
Heartburn, hematemesis, and an esophageal ulcer occurring after treatment was initiated indicate
drug-induced esophagitis.

www.eduwaves360.com | Telegram : @eduwaves360


Correct Answer

A - Calcium citrate

Explanation Why

Calcium citrate may be given as a supplement to prevent hypocalcemia and secondary


hyperparathyroidism in Paget disease of bone. However, calcium citrate is not associated with
esophagitis.

B - Denosumab

Explanation Why

Denosumab inhibits osteoclasts and reduces bone resorption. It is administered as a subcutaneous


injection to treat osteoporosis. Denosumab is not currently recommended for use in Paget disease of
bone, and it is not associated with esophagitis.

C - Risedronate

Explanation Why

This patient's drug-induced esophagitis is most likely due to oral bisphosphonate therapy (e.g.,
risedronate). Bisphosphonates, which inhibit osteoclast-mediated bone resorption, are the first-line
treatment for Paget disease of the bone. Oral bisphosphonates may cause erosive esophagitis and
esophageal cancer, especially in patients who do not maintain an upright posture for 30–60 minutes
after ingestion. Other side effects of bisphosphonates (both IV and oral) include aseptic osteonecrosis
of the jaw, atypical femoral fractures, hypocalcemia, and hypophosphatemia. IV bisphosphonates
(e.g., zoledronate) may cause an acute-phase reaction with flu-like symptoms 24–72 hours after
administration.

www.eduwaves360.com | Telegram : @eduwaves360


D - Calcitonin

Explanation Why

Calcitonin reduces bone resorption and disease activity in Paget disease of bone. Because it is less
effective than the first-line treatment, it is only used if the first-line therapy is poorly tolerated.
Calcitonin is administered as a subcutaneous injection and does not cause esophagitis.

E - Prednisolone

Explanation Why

Oral prednisolone can cause drug-induced esophagitis, peptic ulcers, and gastrointestinal
hemorrhage. However, prednisolone should be avoided in patients with Paget disease of bone
because prolonged use may lead to osteoporosis and further increase the risk of fragility fractures.
Therefore, it is unlikely that this patient was treated with prednisolone.

F - Acetaminophen

Explanation Why

Acetaminophen can cause drug-induced esophagitis and may be used as an adjunct to help relieve
bone pain in Paget disease of bone. However, acetaminophen is not the first-line therapy for this
condition, and the preferred treatment option for this patient is associated with a higher risk of
esophagitis than acetaminophen.

www.eduwaves360.com | Telegram : @eduwaves360


Question # 36

A 54-year-old woman comes to the emergency department because of drooping on the left side of her
face since that morning. She also reports difficulty closing her eyes and chewing. During the neurologic
examination, the physician asks the patient to open her jaw against resistance. Which of the following
muscles is most likely activated in this movement?

Answer Image

A Palatopharyngeus

B Lateral pterygoid

C Masseter

D Orbicularis oris

www.eduwaves360.com | Telegram : @eduwaves360


Answer Image

E Hyoglossus

F Buccinator

www.eduwaves360.com | Telegram : @eduwaves360


Hint

The muscle responsible for lowering the jaw has two heads and is innervated by the mandibular division
of the trigeminal nerve.

www.eduwaves360.com | Telegram : @eduwaves360


Correct Answer

A - Palatopharyngeus

Image

Explanation Why

The palatopharyngeus muscle, which receives motor innervation from the spinal accessory nerve, is
one of the five paired muscles of the soft palate. It closes off the nasopharynx from the oropharynx
during swallowing by tensing the soft palate and pulling the pharyngeal walls superiorly, anteriorly,
and medially. It plays a role in preventing aspiration during swallowing but it is not a primary muscle
of mastication and is not responsible for the movement of the mandible.

www.eduwaves360.com | Telegram : @eduwaves360


B - Lateral pterygoid

Image

Explanation Why

The temporalis, medial pterygoid, lateral pterygoid, and masseter are the four primary muscles of
mastication, which are responsible for the movement of the mandible during the process of chewing.
The lateral pterygoid is the only muscle that lowers the mandible (opens the jaw), while the other
three muscles function to close the jaw. Together with the medial pterygoid muscle, the lateral
pterygoid also functions to move the mandible from side to side (contraction of pterygoids causes
deviation to contralateral side). All of the muscles of mastication are innervated by the mandibular
division of the trigeminal nerve (V3).

www.eduwaves360.com | Telegram : @eduwaves360


C - Masseter

Explanation Why

The masseter is innervated by the mandibular division of the trigeminal nerve and is one of the four
muscles of mastication. However, it closes the jaw (elevates the mandible); it would not be activated
when the patient opens her jaw (depressing the mandible).

D - Orbicularis oris

Explanation Why

The orbicularis oris muscle is a complex, multi-layered muscle that encircles the mouth and is
innervated by the buccal branch of the facial nerve. It facilitates the precise movements of the lips
during speech and facial expression and allows food to be held in the mouth during mastication by
enabling pursing of the lips and puckering lip edges. However, it is not a primary muscle of
mastication and is not responsible for the movement of the mandible.

www.eduwaves360.com | Telegram : @eduwaves360


E - Hyoglossus

Image

Explanation Why

The hyoglossus muscle, which is innervated by the hypoglossal nerve, is an extrinsic tongue muscle
that depresses and retracts the tongue. It is used during speech and facilitates tongue movement
during mastication and swallowing food. It is, however, not a primary muscle of mastication and is
not responsible for the movement of the mandible.

F - Buccinator

Explanation Why

The buccinator muscle, which lies under the cheeks and is innervated by the buccal branch of the
facial nerve, is a muscle of facial expression. It also functions to pull the cheeks inwards against the
teeth during chewing and acts as a valve to prevent air from entering the parotid duct. However, it is

www.eduwaves360.com | Telegram : @eduwaves360


not a primary muscle of mastication and is not responsible for the movement of the mandible.

www.eduwaves360.com | Telegram : @eduwaves360


Question # 37

A 66-year-old man comes to the physician for a follow-up examination after a below-knee amputation of
the right lower leg. Three weeks ago, he had an acute arterial embolism that could not be revascularized
in time to save the leg. He now reports episodic hot, shooting, and tingling pain in the right lower leg
that began shortly after the amputation. He has type 2 diabetes mellitus. He has smoked one pack of
cigarettes daily for the past 30 years. His current medications are metformin and prophylactic
subcutaneous heparin. His temperature is 37.1°C (98.8°F), pulse is 78/min, and blood pressure is
135/88 mm Hg. Physical examination shows a slightly erythematous stump with clean sutures. The skin
overlying the stump is warm and well-perfused. Muscle strength and sensation are normal throughout
the remaining extremity and the left lower extremity. Which of the following is the most likely
diagnosis?

Answer Image

A Phantom limb pain

B Foreign body reaction

C Reinfarction

D Diabetic neuropathy

E Osteomyelitis

www.eduwaves360.com | Telegram : @eduwaves360


Hint

This patient has a common postoperative complication that develops in 50–85% of patients who have
undergone limb amputation.

www.eduwaves360.com | Telegram : @eduwaves360


Correct Answer

A - Phantom limb pain

Explanation Why

Phantom limb pain is a condition characterized by episodic burning, tingling, shooting, itching, and/
or squeezing pain in an amputated limb. This condition typically develops within days to weeks after
surgery, as seen in this patient. Other postoperative complications (e.g., infection, ischemia) must be
ruled out to make a diagnosis of phantom limb pain. Treatment options include mirror therapy,
transcutaneous electrical nerve stimulation, NMDA receptor antagonists, and other adjuvant
therapies (e.g., tricyclic antidepressants, anticonvulsants).

B - Foreign body reaction

Explanation Why

Foreign body reactions can manifest with extremity pain and develop as a postoperative
complication. However, this patient does not have other typical manifestations of this condition,
including swelling, point tenderness, wound drainage, and failure of a wound to fully close or
opening of a closed wound.

C - Reinfarction

Explanation Why

This patient, who has diabetes mellitus, a 30 pack-year smoking history, and a history of a previous
infarction, is at increased risk for reinfarction, which can manifest with extremity pain. However, he
does not have other typical manifestations of infarction, including numbness, pallor, paralysis,
coolness to the touch, and poor perfusion of the stump. In addition, he is receiving thrombosis
prophylaxis, which decreases the risk of reinfarction.

www.eduwaves360.com | Telegram : @eduwaves360


D - Diabetic neuropathy

Explanation Why

Diabetic neuropathy is a long-term complication of diabetes mellitus that can cause burning
extremity pain due to damage to nerve fibers. However, diabetic neuropathy typically develops more
chronically with symmetric pain and loss of sensation in the distal lower extremities; in contrast, this
patient has unilateral pain that has developed across only 3 weeks, and his sensation is intact in the
left lower extremity.

E - Osteomyelitis

Explanation Why

Osteomyelitis can manifest with extremity pain and develop as a postoperative complication of
amputation; it is more common in patients with diabetes mellitus and patients who smoke. However,
the pain associated with osteomyelitis is typically dull in quality, not hot, shooting, and tingling.
Moreover, this patient does not have other typical manifestations of osteomyelitis, including fever,
chills, and localized swelling; the erythema, warmth, and clean sutures on examination are
appropriate postoperative findings.

www.eduwaves360.com | Telegram : @eduwaves360


Question # 38

A 53-year-old multiparous woman is scheduled to undergo elective sling surgery for treatment of stress
incontinence. She has frequent loss of small amounts of urine when she coughs or laughs, despite
attempts at conservative treatment. The physician inserts trocars in the obturator foramen bilaterally to
make the incision and passes a mesh around the pubic bones and underneath the urethra to form a sling.
During the procedure, the physician accidentally injures a nerve in the obturator foramen. The function
of which of the following muscles is most likely to be affected following the procedure?

Answer Image

A Obturator internus

B Adductor longus

C Tensor fascia latae

www.eduwaves360.com | Telegram : @eduwaves360


Answer Image

D Transversus abdominis

E Rectus femoris

F Semitendinosus

www.eduwaves360.com | Telegram : @eduwaves360


Hint

The nerve passing through the obturator foramen is the obturator nerve, which is at risk of being injured
during pelvic surgeries or as a result of pelvic trauma.

www.eduwaves360.com | Telegram : @eduwaves360


Correct Answer

A - Obturator internus

Explanation Why

The obturator internus muscle is supplied directly from the sacral plexus roots L5, S1, and S2.
Although the muscle itself is dissected during midurethral sling surgery, the roots innervating this
muscle pass through the greater sciatic foramen, not the obturator foramen, and are rarely damaged.

B - Adductor longus

Image

Explanation Why

The obturator nerve innervates the muscles of the medial compartment of the thigh, including the
adductor longus, adductor magnus, adductor brevis, gracilis, external obturator, and, occasionally,

www.eduwaves360.com | Telegram : @eduwaves360


the pectineus muscle. It is frequently damaged during midurethral sling surgeries, as well as by
pelvic ring fractures and obturator hernias. Damage to this nerve can cause sensory deficits along the
medial thigh and weakness of hip adduction.

C - Tensor fascia latae

Image

Explanation Why

The tensor fascia latae muscle is innervated by the superior gluteal nerve, which passes through the
greater sciatic foramen above the piriformis muscle and lies in the superomedial gluteal region. The
superior gluteal nerve does not pass through the obturator foramen and is not typically injured in
midurethral sling surgery. Instead, this nerve is typically injured by gluteal intramuscular injections.

www.eduwaves360.com | Telegram : @eduwaves360


D - Transversus abdominis

Image

Explanation Why

The transversus abdominis muscle is innervated by the iliohypogastric nerve, which passes over the
quadratus lumborum muscle of the posterior abdominal wall and is at risk of injury during abdominal
surgery. Midurethral sling surgery is performed in the pelvic region and does not typically endanger
the iliohypogastric nerve.

www.eduwaves360.com | Telegram : @eduwaves360


E - Rectus femoris

Image

Explanation Why

The rectus femoris muscle, which is part of the quadriceps femoris muscle, is innervated by the
femoral nerve. The femoral nerve lies within the psoas major muscle of the posterior abdominal wall
and enters the thigh by passing underneath the inguinal ligament. It can be injured during femoral
vascular access procedures, abdominal surgery, and pelvic surgery. The nerve that was injured during
this patient's pelvic procedure lies in the obturator foramen, which does not contain the femoral
nerve.

www.eduwaves360.com | Telegram : @eduwaves360


F - Semitendinosus

Image

Explanation Why

The semitendinosus muscle is innervated by the sciatic nerve, which passes through the greater
sciatic foramen below the piriformis muscle and lies in the gluteal region. The sciatic nerve does not
pass through the obturator foramen and is not typically injured during midurethral sling surgery.
Instead, this nerve is typically injured during hip joint surgeries and by gluteal intramuscular
injections.

www.eduwaves360.com | Telegram : @eduwaves360


Question # 39

A 28-year-old man comes to the physician because of a 1-week history of weakness in the fingers of his
right hand. One week ago, he experienced sudden pain in his right forearm during weight training. He
has no history of serious illness. Physical examination shows impaired flexion of the proximal
interphalangeal joints, while flexion of the distal interphalangeal joints is intact. Which of the following
muscles is most likely injured?

Answer Image

A Palmaris longus

B Flexor carpi radialis

C Flexor carpi ulnaris

D Flexor digitorum superficialis

www.eduwaves360.com | Telegram : @eduwaves360


Answer Image

E Flexor digitorum profundus

www.eduwaves360.com | Telegram : @eduwaves360


Hint

The affected muscle splits into four tendons to attach to the middle phalanges of the second to the fifth
digit.

www.eduwaves360.com | Telegram : @eduwaves360


Correct Answer

A - Palmaris longus

Explanation Why

The palmaris longus muscle is a weak flexor of the wrist. This patient presents with impaired flexion
of the proximal interphalangeal joints; this movement would not be affected by palmaris longus
muscle injury.

B - Flexor carpi radialis

Explanation Why

The flexor carpi radialis muscle enables flexion of the forearm and abduction and flexion of the
wrist. This patient presents with impaired flexion of the proximal interphalangeal joints; this
movement would not be affected by flexor carpi radialis muscle injury.

C - Flexor carpi ulnaris

Explanation Why

The flexor carpi ulnaris muscle enables flexion and adduction of the wrist. This patient presents with
impaired flexion of the proximal interphalangeal joints; this movement would not be affected by
flexor carpi ulnaris muscle injury.

www.eduwaves360.com | Telegram : @eduwaves360


D - Flexor digitorum superficialis

Image

Explanation Why

The flexor digitorum superficialis muscle, which originates from the medial epicondyle of the
humerus and proximal shaft of the radius and inserts into the middle phalanges of the fingers,
enables flexion of the wrist, metacarpophalangeal joints, and proximal interphalangeal joints.
Isometric weight training can cause rupture of this muscle, as seen here. Flexion of the distal
interphalangeal joints is a function of the flexor digitorum profundus muscle and is unaffected by
flexor digitorum superficialis muscle injury.

www.eduwaves360.com | Telegram : @eduwaves360


E - Flexor digitorum profundus

Image

Explanation Why

The flexor digitorum profundus muscle enables flexion of the wrist, metacarpophalangeal joints, and
distal interphalangeal joints. This patient presents with impaired flexion of the proximal
interphalangeal joints; this movement would not be affected by flexor digitorum profundus muscle
injury. Moreover, flexion of the distal interphalangeal joints is intact.

www.eduwaves360.com | Telegram : @eduwaves360


Question # 40

An 18-year-old man comes to the physician because of severe left shoulder pain after a basketball
match. During the game, the patient sustained an injury to the posterior part of his outstretched arm after
being blocked by a defender. Examination shows no gross deformity of the left shoulder. Palpation of
the shoulder elicits mild tenderness. Internal rotation of the arm against resistance shows weakness.
These findings are most specific for injury to which of the following muscles?

Answer Image

A Supraspinatus

B Deltoid

C Subscapularis

D Infraspinatus

www.eduwaves360.com | Telegram : @eduwaves360


Answer Image

E Teres minor

www.eduwaves360.com | Telegram : @eduwaves360


Hint

The affected muscle's function can also be tested with the lift-off test.

www.eduwaves360.com | Telegram : @eduwaves360


Correct Answer

A - Supraspinatus

Explanation Why

The supraspinatus is a muscle of the rotator cuff that abducts the arm from 0°–15°. Its function can
be tested with the Jobe test. Although the supraspinatus muscle/tendon is the most common site of a
rotator cuff injury, this patient's weakness of internal rotation suggests that a different muscle is
affected.

B - Deltoid

Image

Explanation Why

The deltoid muscle forms the contour of the shoulder and primarily abducts the shoulder from

www.eduwaves360.com | Telegram : @eduwaves360


15°–100°. Although this muscle also assists in internal rotation of the arm, this movement is more
specific for a different muscle.

C - Subscapularis

Image

Explanation Why

The subscapularis is a muscle of the rotator cuff and the main internal rotator of the shoulder. The
muscle originates at the costal surface of the scapula and inserts into the lesser tubercle of the
humerus. Innervation is provided by the lower and upper subscapular nerves. This patient's
presentation suggests a traumatic subscapularis tendon tear, for which surgical repair is usually
indicated.

www.eduwaves360.com | Telegram : @eduwaves360


D - Infraspinatus

Explanation Why

The infraspinatus is a muscle of the rotator cuff and the main external rotator of the shoulder. Its
function can be tested with the infraspinatus test. This patient's weakness of internal rotation suggests
an injury of a different muscle.

E - Teres minor

Explanation Why

The teres minor is a muscle of the rotator cuff that adducts and externally rotates the shoulder. This
patient's weakness of internal rotation suggests an injury of a different muscle.

www.eduwaves360.com | Telegram : @eduwaves360


www.eduwaves360.com | Telegram : @eduwaves360
Join us on Telegram :

Click here : @eduwaves360

Unlocked the Medical premiums

Click here : www.eduwaves360.com

Medical Courses : https://t.me/usmle_study_materials_2

Discussion Group : @usmle_discussion_group

www.eduwaves360.com | Telegram : @eduwaves360


Question # 1

A 34-year-old woman comes to the physician because of swelling of her wrist for the past few months.
She has no pain, numbness, or weakness in her hand or wrist. The swelling has not changed in size and
there is no history of trauma. She has no history of serious illness and her only medication is an oral
contraceptive. Physical examination shows a 2.5-cm firm, nonerythematous, transilluminant mass on the
dorsum of the right wrist over the extensor carpi radialis brevis tendon. The mass is nontender, cool to
the touch, and fluctuant. Histological examination of the contents of the swelling is most likely to show
which of the following?

Answer Image

Spindled fibroblasts in collagenous


A
matrix

B Paucicellular connective tissue

C Neutrophil-rich synovial fluid

www.eduwaves360.com | Telegram : @eduwaves360


Answer Image

D Different forms of hemoglobin

E Encapsulated adipocytes

www.eduwaves360.com | Telegram : @eduwaves360


Hint

This patient's fluctuant mass on the dorsum of her right wrist is consistent with a ganglion cyst.

www.eduwaves360.com | Telegram : @eduwaves360


Correct Answer

A - Spindled fibroblasts in collagenous matrix

Explanation Why

Spindled fibroblasts in a collagenous matrix are characteristic of a tendon sheath fibroma, which can
arise in the wrist and present as a nontender, nonerythematous mass like the one seen here. However,
tendon sheath fibromas would not be fluctuant on palpation and would not transilluminate.

B - Paucicellular connective tissue

Image

Explanation Why

Ganglion cysts develop as a result of trauma, cystic or myxoid connective tissue degeneration, and/or
synovial herniation. Fine-needle aspiration will show paucicellular connective tissue (typically

www.eduwaves360.com | Telegram : @eduwaves360


mucin). Ganglion cysts are the most common type of hand swelling and can be asymptomatic or
cause joint pain and nerve compression.

C - Neutrophil-rich synovial fluid

Image

Explanation Why

Neutrophil-rich synovial fluid can be seen in inflammatory (i.e., crystal or rheumatoid arthritis) or
septic arthritis. However, inflammatory and septic arthritis are associated with warm, tender, and
erythematous joint swelling rather than an asymptomatic mass like the one seen here.

www.eduwaves360.com | Telegram : @eduwaves360


D - Different forms of hemoglobin

Image

Explanation Why

An aspirate with different forms of hemoglobin (e.g., oxyhemoglobin, deoxyhemoglobin,


methemoglobin) would raise suspicion for a hematoma or hemarthrosis, which can present as a
fluctuant swelling like the one seen here. However, hematomas and hemarthrosis do not
transilluminate and they present acutely or subacutely, unlike this patient who has had swelling for
the past few months. In addition, both hematomas and hemarthrosis usually occur following an
injury whereas this patient does not have a history of trauma.

www.eduwaves360.com | Telegram : @eduwaves360


E - Encapsulated adipocytes

Image

Explanation Why

Encapsulated adipocytes are characteristic of a lipoma, which can arise at any location, including the
wrist, and present as a nontender, erythematous, fluctuant mass like the one seen here. Lipomas do
not, however, transilluminate.

www.eduwaves360.com | Telegram : @eduwaves360


Question # 2

A 53-year-old man comes to the physician for a follow-up examination after an open fracture of the right
tibia. Three months ago, he went on an expedition to Egypt, where he fell down while running away
from a snake. He was treated with open reduction and internal fixation. He reports that, since his fall, his
leather hat no longer fits. He has also had progressive hearing loss over the past year. He works as a
professor of archeology. He takes ibuprofen for bone pain. Physical examination shows bowing of the
right leg. Which of the following changes in bone architecture is the most likely underlying cause of this
patient's symptoms?

Answer Image

Deposition of lamellar bone interspersed with


A
woven bone

Subperiosteal bone resorption and cystic


B
degeneration

Accumulation of inadequately mineralized


C
osteoid

Formation of multiple punched-out lytic lesions


D
in bony cortex

Loss of cortical bone mass and thinning of


E
trabeculae

www.eduwaves360.com | Telegram : @eduwaves360


Answer Image

Overgrowth of cortical bone and reduced marrow


F
space

www.eduwaves360.com | Telegram : @eduwaves360


Hint

This patient presents with bone pain and deformity (“saber shin”), apparent enlargement of the skull
(with features of nerve compression, indicated by the hearing loss), and a history that suggests a
pathological fracture. These findings indicate Paget disease of bone. Indiana Jones is getting old.

www.eduwaves360.com | Telegram : @eduwaves360


Correct Answer

A - Deposition of lamellar bone interspersed with woven bone

Image

Explanation Why

Deposition of lamellar bone interspersed with disorganized, woven bone creates a mosaic pattern that
is typical for Paget disease of bone (PDB). These changes in bone architecture are due to an
accelerated rate of bone remodeling, resulting in poor-quality bone that can easily be deformed by
stress (e.g., weight-bearing) and that is highly susceptible to fracture (classically chalk stick fractures
of long bones). Furthermore, enhanced bone remodeling increases bone volume (e.g., of the skull),
leading to complications such as cranial nerve impingement, impaired hearing, and headache. Oral
bisphosphonates are the first-line treatment for PDB.

www.eduwaves360.com | Telegram : @eduwaves360


B - Subperiosteal bone resorption and cystic degeneration

Explanation Why

Subperiosteal bone resorption and cystic degeneration are seen in hyperparathyroidism. In this
condition, pathologically elevated parathyroid hormone levels excessively stimulate osteoclasts,
causing bone catabolism, which can lead to a pathological fracture, bone pain, and, in advanced
cases, bone deformities. However, hearing loss and skull enlargement, as seen in this patient, are not
typical manifestations of hyperparathyroidism. Symptoms of hypercalcemia such as nausea,
constipation, or nephrolithiasis would be expected instead.

C - Accumulation of inadequately mineralized osteoid

Explanation Why

Accumulation of inadequately mineralized osteoid is the pathophysiology of osteomalacia (in adults)


and rickets (in children). Osteomalacia is most commonly caused by a vitamin D deficiency due to
malnutrition, malabsorption, or chronic kidney disease. Typical symptoms include bone pain,
susceptibility to pathologic fractures, and bone deformities such as bowing of the legs, all of which
are seen here. However, this patient has no risk factors for vitamin D deficiency, and symptoms such
as increased skull size and hearing loss would not be expected in osteomalacia.

D - Formation of multiple punched-out lytic lesions in bony cortex

Explanation Why

Multiple punched-out lytic lesions in the bony cortex may occur due to multiple myeloma, bone
metastases, or osteomyelitis, and they can predispose to pathological fractures. Multiple myeloma,
bone metastases, and osteomyelitis can also manifest with bone pain. However, this patient has no
other features of multiple myeloma (e.g., anemia, renal failure, hypercalcemia), bone metastases
(e.g., weight loss, night sweats), or osteomyelitis (e.g., fever). In addition, skull enlargement, hearing
loss, or bone deformity would not be typical of any of these conditions.

www.eduwaves360.com | Telegram : @eduwaves360


E - Loss of cortical bone mass and thinning of trabeculae

Explanation Why

Loss of cortical bone mass and thinning of trabecular bone are the histological hallmarks of
osteoporosis, which can manifest with pathological fractures in diaphyseal bones, as seen in this
patient. Although advanced age is a predisposition for osteoporosis, no other risk factors such as
malnutrition, long-term glucocorticoid therapy, and tobacco use are seen here. Moreover,
osteoporosis would not explain this patient's hearing loss, enlargement of the skull, bone pain, and
bone deformity.

F - Overgrowth of cortical bone and reduced marrow space

Explanation Why

Overgrowth of cortical bone and reduced marrow space are seen in osteopetrosis. Pathological
fractures are a common feature in osteopetrosis due to defective osteoclast activity, which results in a
thickened cortical bone that breaks easily. Narrowing of cranial nerve foramina can lead to cranial
nerve palsies, resulting in symptoms such as hearing loss. However, osteopetrosis typically manifests
by childhood and usually manifests along with pancytopenia (due to decreased bone marrow space)
and hepatosplenomegaly (due to extramedullary hematopoiesis). Moreover, osteopetrosis is not
consistent with this patient's enlargement of the skull, bone pain, and bone deformity.

www.eduwaves360.com | Telegram : @eduwaves360


Question # 3

A 55-year-old man comes to the physician because of a 7-month history of dull hip pain that worsens at
night. Treatment with over-the-counter ibuprofen has not lessened the pain. He has not had any trauma
to the hip. Examination shows swelling and marked tenderness to palpation over the right hip. An x-ray
of the right hip shows an ill-defined lytic lesion with spiculated calcifications and soft tissue infiltration
in the proximal femur. A photomicrograph of a biopsy specimen of the lesion is shown. Which of the
following is the most likely diagnosis?

Answer Image

A Osteoblastoma

www.eduwaves360.com | Telegram : @eduwaves360


Answer Image

B Osteoid osteoma

C Osteochondroma

D Osteosarcoma

E Chondrosarcoma

www.eduwaves360.com | Telegram : @eduwaves360


Answer Image

F Ewing sarcoma

www.eduwaves360.com | Telegram : @eduwaves360


Hint

The photomicrograph shows atypical, irregularly shaped lobules of cartilage separated by fibrous bands.

www.eduwaves360.com | Telegram : @eduwaves360


Correct Answer

A - Osteoblastoma

Image

Explanation Why

Osteoblastoma is a benign bone tumor that can manifest with progressive, dull pain that is
unresponsive to NSAID therapy. However, osteoblastomas develop more commonly in the vertebrae
than the pelvis. In addition, osteoblastomas usually develop in children and young adults (peak
incidence of 10–20 years of age). Moreover, x-ray findings typically show a bony mass with a
radiolucent core, which is not seen here. Infiltration of the adjacent soft tissue, as seen in this patient,
would not be expected.

www.eduwaves360.com | Telegram : @eduwaves360


B - Osteoid osteoma

Explanation Why

Osteoid osteoma is a benign bone tumor that can manifest with progressive pain that worsens at
night. However, these tumors typically develop in the cortices of long bones rather than the pelvis,
and the associated pain is usually relieved by NSAID therapy. In addition, osteoid osteomas usually
develop in children and young adults (peak incidence of 10–30 years of age). Moreover, x-ray
findings typically show a bony mass with a radiolucent core, which is not seen here. Infiltration of
the adjacent soft tissue, as seen in this patient, would not be expected.

C - Osteochondroma

Image

Explanation Why

Osteochondroma is a benign bone tumor that arises from the growth plate. Although this tumor can

www.eduwaves360.com | Telegram : @eduwaves360


cause pain, it is typically asymptomatic. Because osteochondromas most commonly develop during
periods of rapid bone growth, they are more common in children and adolescents (peak incidence of
10–20 years of age) than adults. In addition, they usually develop in the metaphyses of long bones
rather than the pelvis. Moreover, x-ray findings typically show a pedunculated or sessile lesion on
the surface of bone metaphyses, neither of which is seen here. Infiltration of the adjacent soft tissue,
as seen in this patient, would not be expected.

D - Osteosarcoma

Image

Explanation Why

Osteosarcoma is a malignant bone-forming tumor that arises from osteoblasts, and it may manifest
with progressive, dull pain. However, osteosarcomas develop more commonly in the metaphyses of
long bones than the pelvis. In addition, x-ray findings typically show a sclerotic lesion and/or a
periosteal reaction, which can result in a Codman triangle and/or sunburst appearance. Moreover,
osteosarcomas typically develop during adolescence (peak incidence of 10–20 years of age) or more
advanced age (70–80 years). Finally, histopathologic findings in osteosarcoma typically show
pleomorphic osteoblasts and woven bone matrix, which are not seen here.

www.eduwaves360.com | Telegram : @eduwaves360


E - Chondrosarcoma

Image

Explanation But

Osteochondroma rarely develops into chondrosarcoma.

Explanation Why

Chondrosarcoma is a malignant tumor that arises from chondrocytes. It is most common in men ≥ 50
years of age and is usually a slow-growing tumor. Common sites of involvement include the pelvis,
ribs, proximal femur, and proximal humerus. Manifestations include insidious, dull, deep pain that
worsens at night and localized swelling with tenderness to palpation, as seen in this patient. X-ray
findings typically show a moth-eaten (lytic) appearance with popcorn-like (spiculated) calcifications.
The diagnosis is confirmed by biopsy, which typically shows irregularly shaped lobules of cartilage,
fibrous bands, and calcium deposits. Surgery, which may be curative depending on the grade,
extension, and location of the tumor, is the treatment of choice.

www.eduwaves360.com | Telegram : @eduwaves360


F - Ewing sarcoma

Image

Explanation Why

Ewing sarcoma is an aggressive, malignant bone tumor that arises from neuroectodermal cells. It is
commonly located on the pelvis, resulting in progressive pelvic pain that worsens at night. However,
Ewing sarcoma more commonly develops in children and adolescents (peak incidence of 10–15
years of age) than adults. In addition, x-ray findings usually show periosteum with an onion skin
appearance. Moreover, histopathologic findings in Ewing sarcoma typically show anaplastic, small
blue cells that resemble lymphocytes, which are not seen here.

www.eduwaves360.com | Telegram : @eduwaves360


Question # 4

A 31-year-old man comes to the physician because of a 1-month history of pain in his right arm and
numbness and tingling in his right hand. One month ago, he quit his job as a painter because the
symptoms worsened whenever he tried to reach above his head. He has no history of serious illness and
takes no medications. He does not smoke and drinks two beers daily. Vital signs are within normal
limits. Physical examination shows decreased sensation to fine touch in the 4th and 5th fingers of his
right hand. Radial pulse intensity decreases when the patient's right arm is abducted and externally
rotated. Which of the following is the most likely cause of this patient's symptoms?

Answer Image

Carpal tunnel
A
syndrome

B Cervical rib

C Polymyositis

www.eduwaves360.com | Telegram : @eduwaves360


Answer Image

Upper trunk
D
compression

Cervical disc
E
protrusion

www.eduwaves360.com | Telegram : @eduwaves360


Hint

Physical examination shows a positive Adson test (i.e., decreased pulse intensity with arm abduction and
external rotation).

www.eduwaves360.com | Telegram : @eduwaves360


Correct Answer

A - Carpal tunnel syndrome

Image

Explanation Why

Carpal tunnel syndrome (CTS) results in median nerve compression and often presents with
paresthesia of the fingers, which can worsen with prolonged overhead hand elevation (positive hand
elevation test). However, this paresthesia would be present in the 1st, 2nd, and 3rd digits rather than
the 4th and 5th digits. Moreover, this patient's arm pain and the positive Adson test are not consistent
with CTS.

www.eduwaves360.com | Telegram : @eduwaves360


B - Cervical rib

Image

Explanation Why

A cervical rib is a common cause of thoracic outlet syndrome, which occurs because of compression
of the lower trunk of the brachial plexus (derived from the C8–T1 nerve roots) and/or subclavian
vessels in the area of the scalene triangle. Compression of the lower trunk of the brachial plexus can
cause neuropathic pain, paresthesias, and sensory loss in the C8 and T1 dermatomes (i.e., 4th and 5th
fingers, medial aspect of the arm), as seen here, and possibly weakness of the intrinsic muscles of the
hand. The symptoms in the case of a cervical rib are usually exacerbated by overhead arm elevation.
Compression of the subclavian vessels is less common; subclavian vein compression results in
swelling of the arm, and subclavian artery compression can cause acute or chronic limb ischemia
and, possibly, subclavian steal syndrome. A positive Adson test, as seen here, is suggestive of a
cervical rib. Other common causes of thoracic outlet syndrome are Pancoast tumor, muscular
anomalies (e.g., supernumerary scalene muscles, neck muscle hypertrophy), and trauma (e.g.,
clavicle fracture).

www.eduwaves360.com | Telegram : @eduwaves360


C - Polymyositis

Explanation Why

Patients with polymyositis have progressive symmetric proximal muscle weakness and report
difficulty reaching for objects above their head rather than worsening pain and paresthesia when
performing this action, as seen here. Moreover, this patient's positive Adson test is not consistent
with polymyositis.

D - Upper trunk compression

Image

Explanation Why

Compression of the upper trunk of the brachial plexus (Erb palsy) can also result in pain and
paresthesia. However, the paresthesia would occur in the lateral aspects of the arm, forearm, and/or
hand (C5 and C6 dermatomes), not the 4th and 5th (medial) digits. In addition, patients with Erb

www.eduwaves360.com | Telegram : @eduwaves360


palsy typically present with an internally rotated and adducted arm, extended forearm, and a flexed
wrist (“waiter's tip posture”), all of which are absent in this patient.

E - Cervical disc protrusion

Image

Explanation Why

Protrusion of the C7/T1 intervertebral disc can compress the C8 spinal root and cause pain and
paresthesia in the 4th and 5th digits (C8 dermatome). However, in the case of cervical spinal root
compression (radiculopathy), patients usually have neck pain and the paresthesia is relieved by
overhead arm elevation, unlike the exacerbation experienced in this patient reaching above the head.
Parasthesias in cervical disc protrusion are instead exacerbated by extending the neck to the side and
applying downward pressure to the head (Spurling maneuver). Moreover, a positive Adson test is not
consistent with a cervical disc protrusion.

www.eduwaves360.com | Telegram : @eduwaves360


Question # 5

An 18-year-old man comes to the physician because of a 4-week history of pain in his right foot that
increases with physical activity and improves with rest. He is a military recruit who started his basic
combat training 6 weeks ago. Before he started military training, he did not exercise regularly. The
patient's older brother is a sergeant in the army and the patient has been using his brother's old boots. He
has no history of major medical illness and takes no medications. He is 178 cm (5 ft 10 in) tall and
weighs 60 kg (132 lb); BMI is 19 kg/m2. Physical examination shows mild swelling, erythema, and
tenderness to palpation over the right forefoot. An x-ray of the right foot is shown. Which of the
following is the most likely diagnosis?

Answer Image

A Morton neuroma

www.eduwaves360.com | Telegram : @eduwaves360


Answer Image

B Hallux valgus

C Hammer toe

D Ligament sprain

E Stress fracture

F Metatarsalgia

www.eduwaves360.com | Telegram : @eduwaves360


Hint

Although this patient's x-ray shows a pathological finding, patients with this condition often have
normal x-ray findings for the first 2–3 weeks after the onset of symptoms.

www.eduwaves360.com | Telegram : @eduwaves360


Correct Answer

A - Morton neuroma

Explanation Why

Morton neuroma, which typically occurs in the intermetatarsal spaces between the second, third, and
fourth metatarsal heads, can cause swelling in the foot and neuropathic pain that is aggravated by
physical activity and relieved with rest, as seen here. However, erythema is not expected. Moreover,
x-ray findings in Morton neuroma are typically normal or show splaying of the metatarsals due to a
widening of the intermetatarsal space; this patient's x-ray findings indicate a different diagnosis.

B - Hallux valgus

Image

www.eduwaves360.com | Telegram : @eduwaves360


Explanation Why

A bunion, or hallux valgus, can be caused by ill-fitting footwear and manifest with foot pain and
erythema, which is seen here. However, an x-ray of the foot in hallux vagus typically shows a lateral
deviation of the great toe at the metatarsophalangeal joint, which is not seen here.

C - Hammer toe

Image

Explanation Why

Hammer toe is a common deformity that can occur due to ill-fitting footwear and manifest with foot
pain with exercise, which is seen here. However, hammer toe manifests as hyperextension of the
metatarsophalangeal joint and flexion of the proximal interphalangeal joint, and it is usually not
associated with swelling or erythema. In addition, the x-ray findings show no obvious deformity of
these joints of the lesser toes.

www.eduwaves360.com | Telegram : @eduwaves360


D - Ligament sprain

Explanation Why

A ligament sprain in the forefoot can manifest with swelling, erythema, and pain that increases with
physical activity and improves with rest, all of which are seen here. However, an isolated ligament
sprain does not show pathological findings on x-ray, as seen in this patient.

E - Stress fracture

Explanation Why

This patient's x-ray shows a fracture of the neck of the second metatarsal and callus formation. The
metatarsal bones are common sites for stress fractures, which are typically caused by repetitive, high-
intensity activity and/or by a sudden increase in the intensity of physical activity, as seen with this
army recruit. Such stress to the bone can increase the level of bone resorption to a degree that
overwhelms the bone's ability to adapt to the stress, resulting in multiple microfractures that coalesce
over time to form a larger break in the bone cortex. Risk factors for stress fractures include ill-fitting
footwear (this patient is using his brother's old boots that may not fit him properly), decreased bone
density (e.g., osteoporosis), and severe caloric restriction (e.g., in anorexia nervosa). Other important
sites for stress fractures are the tibia (most common site), calcaneum, and navicular bone.

F - Metatarsalgia

Explanation Why

Metatarsalgia, which is pain along the plantar surface of the metatarsal heads, can worsen with
physical activity, which is seen here. However, metatarsalgia is diagnosed clinically and would not
show pathological findings on x-ray, as seen in this patient.

www.eduwaves360.com | Telegram : @eduwaves360


Question # 6

A 56-year-old woman comes to the physician because of a 1-month history of increasing pain, swelling,
and stiffness in both ankles. The pain and stiffness worsen with movement. She has no history of serious
illnesses and takes no medications. She does not smoke or drink alcohol. Her temperature is
36.8ºC (98.2ºF). Physical examination shows digital clubbing. The ankle joints are swollen and tender to
palpation; range of motion is limited by pain. An x-ray of the lower extremities shows periosteal
thickening of the distal tibial diaphysis in both extremities. Which of the following is the most likely
underlying cause of this patient's symptoms?

Answer Image

A Occult malignancy

B Accelerated bone turnover

C Left-to-right cardiac shunt

www.eduwaves360.com | Telegram : @eduwaves360


Answer Image

D Nutritional deficiency

E Autoimmune joint disease

F Chronic degenerative disease

www.eduwaves360.com | Telegram : @eduwaves360


Hint

This patient has hypertrophic osteoarthropathy, which is due to distal hyperemia and proliferation of
connective tissue in the distal parts of extremities, especially the fingers.

www.eduwaves360.com | Telegram : @eduwaves360


Correct Answer

A - Occult malignancy

Image

Explanation But

In rare cases, HOA occurs as a result of a hereditary disorder (primary HOA).

Explanation Why

Hypertrophic osteoarthropathy (HOA) is most commonly associated with adenocarcinoma of the


lung. Patients with HOA present with arthritis, digital clubbing, and periostitis of the distal diaphysis
of long bones, as seen here. Some affected individuals also develop thickening of the skin
(pachyderma). The exact pathophysiology of this condition is not entirely understood but the release
of certain growth factors like VEGF and PDGF from the tumor is thought to play a role. Other
causes include chronic pulmonary conditions (e.g., bronchiectasis, cystic fibrosis) and, less
commonly, hepatic and gastrointestinal disease (e.g., cirrhosis, inflammatory bowel disease). HOA in
a previously healthy patient should raise suspicion for an occult pulmonary malignancy and a low-
dose CT scan (LDCT) of the chest should be obtained. Adenocarcinoma of the lung can occur in
nonsmokers, like this patient, and is more common among female than male individuals.

www.eduwaves360.com | Telegram : @eduwaves360


B - Accelerated bone turnover

Image

Explanation Why

Accelerated bone turnover is the cause of Paget disease of bone (PDB), which often affects the long
bones of the lower extremities (e.g., tibia). PDB can manifest with bone pain, swelling overlying the
affected region, and periosteal thickening, all of which are seen here. However, PDB typically does
not lead to arthritis, and in addition to a periosteal reaction, an x-ray would show lytic and/or
sclerotic lesions in the bone cortex and, possibly, anterior bowing of the tibia, neither of which are
seen here. This patient's condition is not the result of accelerated bone turnover.

www.eduwaves360.com | Telegram : @eduwaves360


C - Left-to-right cardiac shunt

Explanation Why

A right-to-left cardiac or pulmonary shunt (e.g., pulmonary arteriovenous malformation) rather than
a left-to-right cardiac shunt (e.g., patent foramen ovale) can cause hypertrophic osteoarthropathy. A
right-to-left shunt bypasses the filtering effect of pulmonary circulation on platelet clumps, which
then accumulate peripherally and produce growth factors (e.g., PDGF) that can cause osseous
hypertrophy.

D - Nutritional deficiency

Image

Explanation Why

Vitamin D deficiency can cause bone pain but does not cause joint swelling and stiffness or digital
clubbing, which are seen here. Moreover, x-ray findings of osteomalacia include decreased bone

www.eduwaves360.com | Telegram : @eduwaves360


lucency and Looser zones, not periosteal thickening. This patient's condition is not the result of a
nutritional deficiency.

E - Autoimmune joint disease

Image

Explanation Why

Rheumatoid arthritis (RA) is an autoimmune joint disease that occasionally affects the ankle joints,
and it manifests with joint pain, swelling, and decreased range of motion, all of which are seen in this
patient. However, the stiffness and pain associated with RA typically improve rather than worsen
with activity. Moreover, RA does not cause digital clubbing or periosteal thickening; instead, finger
deformities (e.g., swan neck deformity, Boutonniere deformity) and different x-ray findings (e.g.,
concentric joint space narrowing, subcortical cysts, para-articular osteopenia) would be expected.
This patient's arthralgia is not the result of an autoimmune process.

www.eduwaves360.com | Telegram : @eduwaves360


F - Chronic degenerative disease

Image

Explanation Why

Osteoarthritis is a chronic degenerative disease that can affect the ankle joints, and it manifests with
joint stiffness, restricted range of motion, and pain that worsens with activity, all of which are seen
here. However, osteoarthritis does not cause digital clubbing or periosteal thickening. Instead, an x-
ray would show irregular joint space narrowing, osteophytes, subchondral sclerosis, and/or
subchondral cysts. This patient's condition is not the result of a chronic degenerative process.

www.eduwaves360.com | Telegram : @eduwaves360


www.eduwaves360.com | Telegram : @eduwaves360
Join us on Telegram :

Click here : @eduwaves360

Unlocked the Medical premiums

Click here : www.eduwaves360.com

Medical Courses : https://t.me/usmle_study_materials_2

Discussion Group : @usmle_discussion_group

www.eduwaves360.com | Telegram : @eduwaves360

You might also like